.
.
Math Central - mathcentral.uregina.ca
Quandaries & Queries
Q & Q
. .
topic card  

Topic:

si

list of
topics
. .
start over

1680 items are filed under this topic.
 
Page
1/1
Calculating the area (acreage) of a four sided lot 2007-07-18
From A property owner:
I have a real estate property and the lot size is something I need to find out. I know the lengths of the four sides, but it isn't a rectangle, it is an odd shape. How do I find the acreage?
Answered by Stephen La Rocque.
Lot size / acreage (more than three sides) 2007-05-11
From Martha:
What is the lot size of the following dimensions equivalent to acre. 141.85 x 52.55 x 217.63 x 38.89 x 68.08
Answered by Penny Nom and Stephen La Rocque.
The area of a lot 2022-12-31
From Brian:
To calculate lot size, do you use arc length or chord length? Lot Survey shows: Length 1 = 120.0' Length 2 = 120.0' Width 1 = 61.0' Width 2 = ARC Length 125.58' / CHORD Length 124.10' / Radius = 236.0'
Answered by Harley.
The sides of a 5-pointed star inscribed in a circle 2021-09-18
From Scott:
Is there a calculation for the length of each side of a five-pointed star? I am not worried about area but if I have a circle with a 7.5 foot radius and place a star inside it, how can I measure the lines creating the star?
Answered by Harley Weston.
The dimensions of a rectangle 2021-09-09
From Ash:
I need to know how to figure out the dimensions of a rectangle. I know that the area is 30ft and the perimeter is 26ft. I'm 15 and I have no idea what I'm supposed to do.
Answered by Harley Weston.
Division in different bases 2021-02-06
From Promise:
Simplify the following 302 in base6 divided by 5 in base6
Answered by Penny Nom.
Simplify 2021-02-05
From Leslie:
To simplify improper fractions, do you convert it to a mixed number or leave as an improper fraction?

My daughter had a quiz in which she was supposed to simplify her answers. So, for example, on one question the answer she got was 11/10 and on another question she came up with 17/12. The teacher marked these as incorrect because they were mixed numbers. Her teacher wrote on her paper, "**name**, I even gave this back to you and told you to go through and simplify your answers!" We (her father and I) feel that the teacher wanted her to convert her answers, not simplify them but I haven't been able to find a definitive answer online. So, does simplifying improper fractions mean converting them to mixed numbers? Thank you for your help!


Answered by Penny Nom.
Divisibility by 8 2020-11-10
From Sariyah:
Of the positive integers between 1000 and 10000 that are divisible by 8, how many have a 5 in the hundreds place?
Answered by Harley Weston.
The dimensions of a rectangle 2020-11-02
From Chie:
A rectangular piece of property has an area of 987m² and a perimeter of 136m. Find its LENGTH and WIDTH.
Answered by Penny Nom.
Two equations with fractions 2020-10-22
From mia:
I am supposed to look for x and y. my teacher didn't explain very well.
please help : ((
2x - y = 5
x/4 + y/3= 2

Answered by Penny Nom.
A confusing problem 2020-10-01
From Kgaugelo:
a triangle has the vertices A(-4;-3),B(x;y)and C(2;1). Determine the coordinates of B if the equation of the line BC is y=5x-9
Answered by Harley Weston.
9 games with 3 possible outcomes 2020-09-13
From Daniel:
Hi
I’d like to know how many different outcomes would be in a Soccer week played if there is 9 different games 2 teams and 3 outcomes Win, tie or lose

Thank you

Answered by Penny Nom.
A window problem 2020-08-18
From Richard:
Hello,
I was hoping your math specialists could help me with some formulas. I have shapes with specific known variables need to calculate others.  

Example:
We make a straight legged arch, This shape has a width a overall height and a leg height, The leg height is always less then the overall, And the top is arched. We have the width, Height, and leg size.

  Need to calculate the length of the curve and sq ft of the shape.

Answered by Harley Weston.
Filling holes with stone dust 2020-07-30
From Zach:
I’m looking for amount of stone dust needed for thirteen holes with a 3ft depth and 12in diameter with a 4x4 post. I would need the measurement in KG.
Answered by Harley Weston.
Choosing 4 balls from 9 2020-07-22
From sandiso:
What is the probability associated with each possible random sample of size 4 from the population consisting of these nine balls
Answered by Penny Nom.
A word problem involving linear equations 2020-06-22
From Allie:
The length of a rectangular swimming pool is p m and its width is q m. Its given tht the length of the swimming pool is twice its with. if the perimeter of the swimming pool is 150 m, find the values of p and q this is in the chapter linear equations
Answered by Penny Nom.
1 square cm is 100 square mm 2020-06-19
From macie:
explain why the conversion factor for cm squared to mm squared is 100 ,and not 10
Answered by Penny Nom.
A triangular garden 2020-05-24
From yoonji:
the 3 sides of a triangular garden measure 200 ft., 250 ft., and 300 ft., respectively. the garden is to be divided by a line bisecting the longest side and drawn from the opposite vertex. what is the length of the bisecting line?
Answered by Penny Nom.
An angle i a triangle 2020-05-16
From Ogunjobi:
Two goal post are 8m apart a footballer is 34 m from one post and 38m from the other within what angle must he kick the ball if he is to score
Answered by Penny Nom.
Running in opposite directions around a circular track 2020-05-11
From Alessandra:
2. Alan and Brian are two runners. Alan runs at 6.6 m/s and Brian runs at 8.4 m/s. They start from the same place on a circular 400 m track and run in opposite directions round and round the track.
(a) How far has Brian run when they pass each other for the first time?
(b) After they pass each other for the second time, how much further does Brian need to run in order to complete two laps (800 m)?

Answered by Penny Nom.
A system of equations. 2020-04-27
From solomon:
xy + x =28
x + y +4

Answered by Penny Nom.
The missing diameter of a truncated cone 2020-04-17
From Brandon:
Need a formula to find missing large Dia of a truncated cone where the small dia =.19" Height= 2" and taper =16deg
Answered by Harley Weston.
A hemisphere and a cylinder 2020-03-23
From Monalizsha:
Wat is the volume and the surface area of the hemisphere and cylinder (both are together) The height is 12cm of the cylinder and the diameter is 6cm
Answered by Penny Nom.
The side length of a hexagon 2020-02-05
From Rob:
I have a hexagon that is 8 feet wide how long would the sides be?
Answered by Penny Nom.
A system of inequalities 2020-02-01
From Isaac:
Austin has x nickels and y dimes, having a maximum of 15 coins worth a minimum of $1 combined. No more than 4 of the coins are nickels and no less than 11 of the coins are dimes. Solve this system of inequalities graphically and determine one possible solution.
Answered by Penny Nom.
Filling a 48 cubic meter tank 2020-01-19
From kamohelo:
An empty tank for storing water from a borehole has a volume of 480m3 if it is filled by a pump that pump water at a rate of 16L/s how many hours will it take the pump to fill this tank
Answered by Penny Nom.
What do you call a 43-sided polygon? 2020-01-06
From Alniko:
What do you call a 43-sided polygon and what is its interior angle's total measure?
Answered by Penny Nom.
Sigma notation 2019-12-04
From Kenneth:

What would be used for the top and bottom indices for the sigma sign in my example shown below?
If the amounts were 1 + 2 + 3, the top index would be 3 and the bottom would be i = 1, but if the amounts are 1 + 2 + 4, or 1 + 0.05 + 1/3, what would the indices be since these amounts are in an uneven pattern.

I thank you for your reply and explanation


Answered by Harley Weston.
A hexagonal planter 2019-11-19
From Colleen:
I need to build a hexagon planter around a 32” square box. How long is each side of the hexagon?
Answered by Penny Nom.
Increasing your average mark 2019-11-01
From Elena:
After 9 weeks Mikayla has an average ark of 5 out of 10 in the weekly spelling tests. What is the minimum number of extra weeks now required to raise her average to 7?
Answered by Penny Nom.
Simultaneous equations 2019-10-16
From deepak:
This is too complicated please help me
8/x - 10/y = 1 and x+y=9

Answered by Penny Nom.
400 in 7.5 billion 2019-08-28
From Emma:
What is a better way to express 400 in 7.5 billion? My daughter has a rare condition, I want to know the answer to say “she is 1 in a ...”
Answered by Penny Nom.
Six digit numbers from 4 digits 2019-07-30
From Tab:
How many six digit combinations can be made from the numbers zero, two, five, eight? With repetition of numbers
Answered by Penny Nom.
Price per square meter to price per square inch 2019-07-09
From Al:
If something costs $2.5 square meter, how much would it cost in square inches? How do i convert costs per square inch, into square meters?
Answered by Harley Weston.
How many cubes are in a 3x3x3 cube? 2019-06-24
From Darren:
Dear Sir/Madam, How many cubes of different sizes (eg. 1x1x1, 2x2x2, 3x3x3) are there in total in say, a 3x3x3 cube? I have trouble figuring this out.

Yours faithfully,
Darren

Answered by Penny Nom.
Yards, feet and gallons 2019-06-12
From Ralph:
How many gallons of water in a 10 yard by 10 yard container 2 1/2 feet deep?
Answered by Penny Nom.
Conversions, feet, kilometers and acres 2019-06-10
From Erika:
A lake has 113 feet deep and an area of 115 acres, what is the volume of the lake in cubic kilometer?
Answered by Harley Weston.
A repeating decimal divided by a whole number 2019-05-01
From Emily:
How do you divide repeating decimals by whole numbers?
Answered by Penny Nom.
(3d) Whether line segment intersects triangle 2019-04-13
From rj:
Well hello,

Lets say I have a triangle,

Not any ordinary triangle, but a triangle that is represented in 3 dimensions. I also have a line segment.

Lets say that this triangle has points A(0,0,0), B(1,0,0), C(0,1,1)

And the line segment has points D(0,1/2,0), E(1,1/2,1)

In what way can I find if the line segment and triangle intersect? Their intersection point?

I don't prefer matrices, because they don't make any sense, but ... if that's the only way to solve this, then do what's necessary.

Thank you in advance.

Answered by Penny Nom.
The area of a lot 2019-04-06
From Robert:
Can you help me determine the rough square footage of this lot please(attached)? I think it is nearly 7000 square feet. I am sure I need the degrees of of the corners but I don’t have that. An estimate would be all I need. Thank you for you help. Rob
Answered by Harley Weston.
Salary Plus Commission 2019-04-05
From Herschel:
Fire Fighting Equipment pays salespeople as follows: $452 per week plus a commission of .9% on sales between $15,000 and $25000, with 1.1% paid on sales in excess of $25,000. Find the gross earnings for an salesperson whose Total Sales are $28,400. (No commission is paid on the first $15,000 of sales)
Answered by Penny Nom.
Four consecutive positive integers 2019-03-12
From Nabila:
The product of four consecutive positive integers in 24024
What is their sum?

Answered by Penny Nom.
Converting a base six number to base three 2019-03-10
From Nina:
Change 253 in base 6 to a number in base 3
Answered by Penny Nom.
Random assignments 2019-03-02
From Ninoshka:
Liu and Michi plan to sign up for a drawing class next term. Drawing is offered during the first 4 periods of the day, and students are assigned randomly to class.
What is the probability that Liu and Michi will have drawing together?

Answered by Penny Nom.
Running around a circular track in opposite directions 2019-02-24
From carmi:
how many times will lita and rose pass each other in 15 minutes if lita and rose start jogging on a 110 meters circular track. they begin at the same point but in opposite direction. lita at 8/3 meters per second and rose at 7/3 meters per second.
Answered by Penny Nom.
Composition of functions 2019-02-24
From Joshua:
Let g(x)=(x)^(2)+x-1 Find such a function f such that (fog)(x)=(x)^(4)+2(x)^(3)-3(x)^(2)-4x+6
Answered by p.
The Polynomial Remainder Theorem 2019-02-23
From pasandi:
f(x) is a quadratic polynomial. when f(x) is divided by (x-1),(x-2) & (x+2) the remainders respectively are -1, 4 and 2 how to find the f(x) in a question like this?
Answered by Penny Nom.
An arithmetic sequence 2019-02-23
From Dalal:
If x+1 and -x+17 are the second and sixth term of a sequence with a common difference of 5, what's the value of x.
Answered by Penny Nom.
Margin with commission as part of selling price 2019-02-13
From Anni:
I need to find selling price to put us at 24% margin.
15% commission is based on selling price and is included in our cost.
I'm trying to do it excel and keep getting a circular reference.
Total cost= cost + commission (15% of selling price)
Selling price=total cost + 24% margin

Answered by Penny Nom.
Fractions with roots 2019-02-06
From DEBMALYA:
5/√2+√3 –1/√2–√3
Answered by Penny Nom.
An arithmetic sequence 2019-01-31
From sara:
if the 6th term of an arithmetic sequence is 8 and the 11th term is -2, what is the first term?
Answered by Penny Nom.
One quarter of a quarter cake 2019-01-30
From Tre:
1/4 of a birthday cake was left over from a party. The next day, it is shared among 4 people. How big a piece of the original cake did each person get?
Answered by Penny Nom.
The sum of three consecutive even integers 2019-01-27
From Pauleen:
Translate this into algebraic expression
The sum of three consecutive even integers, if x is the first even integer

Answered by Penny Nom.
(2018^2019)-(2019^2018) 2019-01-23
From Nazrul:
Find the remainder when (2018^2019)-(2019^2018) is divided by 4? Please show me the process.
Answered by Harley Weston.
A triangle made from 12 matchsticks 2019-01-20
From Palesa:
How many different types of triangles can be made with a perimeter of 12 matches?
Answered by Penny Nom.
7 spheres on a hexagonal tray 2019-01-14
From herm:
what is the length of each side of a hexagonal tray, with the height of each side 0.75 inch, to hold seven spheres, each with a diameter of 3.00 inches? The spheres are placed such that each side of the hexagon is touched by one sphere at its midpoint (and the seventh sphere is place in the center of the "ring" of the other six spheres.
Answered by Harley Weston.
A binomial expansion 2019-01-05
From shifali:
if the binomial expansion of (m-nx)^3 is 1+9x+-- find the values of m & n
Answered by Penny Nom.
A quadrilateral inside a square 2019-01-02
From Swetha:
In 2*2 square ABCD, E is the mid point of the side AD. F is a point in BE.CF is perpendicular to BE. Find the area of the quadrilateral CDEF
Answered by Penny Nom.
Solve for x in Y=1.5x + log (1+x) 2018-12-23
From Asadollah:
Please restate below expression in terms of y

Y=1.5x + log (1+x)

Thanks

Answered by Penny Nom.
Ten games with two possible outcomes for each game 2018-12-22
From Jin:
I'm trying to make a list to view all the possibility.
An example is if I have 2 games there's two outcomes (home/away)
I will use number to represent the teams. 0-1,2-3
All possible out comes are 0-2,0-3,1-2,1-3 right?

MY QUESTION If I have 10 games each have 2 outcomes (home/away) to win
What is all possible outcome.
0-1,2-3,4-5,6-7,8-9,10-11,12-13,14-15,16-17,18-19
Could u please provide me with a list of all possible out comes

Thanks you

Answered by Penny Nom.
Simultaneous equations with fractions 2018-12-14
From zaheer:
solve simultaneous equations and give answer in fractional form
3x - 2 = 4y +5/3
y + 7 = 2x + 4
would really appreciate some help on this please

Answered by Penny Nom.
Divisibility by 3 2018-11-20
From Ray:
There is a rule that a number is divisible by 3 if the sum of its digits are divisible by 3 (for example, 81=8+1=9 {divisible by 3} and 33=3+3=6 {again, divisible by 3}) I know this works but I don't know why! Please help.
Answered by Penny Nom.
Dividing by a fraction 2018-11-02
From Fadely:
If 8 divided by 4/2 can be expressed as 8 X 2/4, how can 8 X 2/4 be converted back to 8 divided by 4/2?
Answered by Penny Nom.
The area of a trapezoid 2018-09-24
From An other:
AB-272 feet: AC 690 feet; CD-330 Feet; DA-669Feet Calculate the area in feet and Acres.
Answered by Penny Nom.
Splitting a commission 2018-09-13
From Beth:
If Sales Prices is $170975 and commission is 6% but listing agent gets 60% of the 6% and selling agent gens 40% of the 6% what is the actual percentage that each would actually receive?
Answered by Penny Nom.
A 3500 acre ranch 2018-08-30
From Dee:
I am in the middle of writing the description of the size of a ranch in my story. It is 3500 acres. How does that equate into mileage? The main ranch house is at one end, and the foreman's ranch house at the other end.
Answered by Penny Nom.
Two spinners 2018-08-13
From Atina:
A spinner has four equal sectors and a number is written on each sector; 1, 2, 3 and 4. A two-digit number is formed by spinning two times. The number on the first spinning makes the first digit and the number on the second spinning makes the second digit. For example, 2 on the first spinning and 1 on the second spinning make the number 21.
(a) Give the sample space S for the experiment.
(b) Consider the following events : E = odd number; F = number smaller than 35; G = prime number. Give the subset of outcomes in S that defines each of the events E, F, and G.
(c) Describe the following events in terms of E, F, and G and find the probabilities for the events.
• getting an even integer less than 35.
• getting an odd number or an prime.
• getting an even number greater than or equal to 35 that is a prime number.
• an odd number smaller than 35 that is not a prime number.
(d) Are E and F mutually exclusive events? Give a reason for your answer.

Answered by Penny Nom.
Linear feet and cubic feet 2018-07-16
From Dave:
How do I convert: 1 foot wide x 15 inches high x 19 inches deep

We are trying to figure out how to convert the linear footage into cubic footage the above are the figures represent just 1 linear feet of several thousand

Any thing helps,

Thanks in advance

Dave

Answered by Penny Nom.
The domain of f(g(x)) 2018-06-18
From Joshua:
What are the restrictions of the domain of f(g(x))?
Answered by Penny Nom.
The inverse square law 2018-06-05
From Amy:

Question about inverse square law ;

Hi, I'm trying to understand some nuances about this law and
have been reading about it a lot online and trying out some homework
for personal interest (hobby ) - not school related.

I understand that the simple formula is ; 1/d^2

I was wondering about what appears to be an oddity to me, that
I came across when I was working with a test example I found at this
link ;

http://www.softschools.com/formulas/physics/inverse_square_law_formula/82/

The problem that caught my interest was this one on that page ;

"1) If a bright flashlight has a light intensity of 15.0 candela at a distance 1.00 m from the lens, what is the intensity of the flashlight 100.0 m from the lens?"

So I have a few different questions about this, but the most important
one is, what does it mean if we replace the "1" in ; 1/d^2 with a different value such as a ratio in the form of a decimal value, so instead we have something like this ;

0.75/d^2

???

Does this ratio represent a curve then?

I am thinking that the "1" in the normal formula represents a straight line

Some of the links I was researching ;

http://wisptools.net/book.php?c=3&s=2

http://www.softschools.com/formulas/physics/inverse_square_law_formula/82/

https://www.nde-ed.org/GeneralResources/Formula/RTFormula/InverseSquare/InverseSquareLaw.htm


Answered by Penny Nom.
Monthly Pension 2018-05-28
From Kenneth:

Hello:

I have a question regarding how the units cancel in the following:

A company uses the following calculation to determine the monthly payment for its employees as they retire:

1.1% * average 5 years highest salary * years of service

If an employee worked 33 years and his average 5 year salary is $30,705.17, what is his monthly payment?

I used the following to determine his monthly payment, but I am not sure how the units cancel especially the years.

1.1% * $30,705.17 * 33 years divided by 12 months/year

I determined his payment as $928.83 per month, but the year units do not cancel unless the 1.1% is per year.

Is the correct calculation 1.1%/year * $30,705.17 * 33 years divided by 12 months/year so that the units cancel properly?

The $\$30,705.17$ is an average for 5 years of working. It is not $30,705.17 per year.

I thank you for your assistance.


Answered by Penny Nom.
What is the diameter of the observable universe? 2018-05-27
From peter:
The diameter of the observable universe is calculated to be 92 billion light years. What would that distance be in miles given a light year is 6 trillion miles?
Answered by Penny Nom.
Making a cylinder from a metal plate 2018-05-18
From Rahul:
I want to make a cylinder which inside diameter is 600mm and thickness is 5mm and height is 800mm...so my question is that how much plate I need to make this happen
Answered by Penny Nom.
mg to ml 2018-05-17
From Carolynne:
I need to convert 12 mg to ml
density is 12.5mg/5ml
I can give dose of 1mg per pound . My dropper is in ml.
How much in ml can I give

Answered by Penny Nom.
Two expressions for the perimeter of a a rectangle 2018-05-13
From Slayde:
A rectangular house has one side 4 metres longer than the other. Create two different expressions for its perimeter. The expressions should be given in factorised and expanded form.
Answered by Penny Nom.
More on the curvature of the Earth 2018-04-23
From will:
the formula for figuring the earth's curve goes against logic, looking at a fixed point and backup 1mi. the point drops 8" then 16" in the next mi. and 32" in the third mi. why shouldn't it be 24" why is the 8" per mi. squared can you tell me in laymens terms why this is it goes against logic it would seem the correct wat would be to add up 8" per mile as you back up from the fixed point 8"- 16"- 24"- 32" not 8/16/32/64"
Answered by Harley Weston.
The sides of a circle 2018-04-10
From Reid:
Hello,
I recently was wondering about whether or not a circle has an infinite number of sides, and I ended up searching it on your website. I saw that you guys found the question to be too ambiguous or something of that nature, and I thought about your process of reasoning involving vertices and such. I soon realized that I may have come up with the solution to the question, but I want to confirm it with you guys. Allow me to explain:
A circle, unlike any other typical 2 dimensional polygon, can sustain an infinite number of straight lines coming in contact with only one point on it. A square, for example, cannot. A square only has 4 locations that can sustain a such a line, each of those being its corners. The flat edge of a square cannot support a tangential line, because the line would either be crossing the edge of the square or coming into contact with multiple points along its edge.
This concept is consistent in every 2 dimensional polygon: pentagons sustain 5 locations for tangential lines, hexagons 6, nonagons 9, etc. The reason a circle has an infinite number of sides is simply the fact that it must have an infinite number of "corners", assuming it can be defined as a polygon like any of the shapes I described above.
Corners can only exist assuming there is two sides coming into contact with one another. If there is an infinite number of corners in the circle, which is apparent due to the above reasoning, there must be an infinite number of sides coming into contact with each other. Thus, a circle is a polygon that consists of an infinite number of sides coming into contact with each other.
Please review this reasoning and let me know if it is solid.
Thank you!

Answered by Penny Nom.
A side of a square 2018-04-06
From Angie:
I am dyslexic and have problems with spelling and math I'm trying to figure 4.610 Acres Square trying to figure out how many feet or yards of one side if that makes sense to you thank you I'd really appreciate the help
Answered by Penny Nom.
Base 6 to base 4 2018-03-25
From Malik:
Hi Sir..!
I just want to know the proper example with a brief explanation of conversion of base 6 into base 4. kindly reply me as soon as possible.. Thank you

Answered by Penny Nom.
Solve sinX=0.703X for X 2018-03-13
From PARAM:
sinX=0.703X
Answered by Penny Nom.
0.366 x cos square (02 degree 17 mins 27 seconds) 2018-03-12
From michael:
what is 0.366 x cos square (02 degree 17 mins 27 seconds)
what is 0.366 x cos square (88 degree 26 mins 45 seconds)

Answered by Penny Nom.
The dimensions of a rectangle 2018-03-03
From Mia:
A wire of length 36cm is bent to form a rectangle. Find the dimensions of the rectangle.
Answered by Penny Nom.
nC0 + nC1 + nC2 + .... + nCn = 2^n 2018-02-19
From bristal:
(QQ) Prove, nC0 + nC1 + nC2 + .... + nCn = 2^n.
Answered by Penny Nom.
A footballer angle 2018-02-14
From Kim:
Two goal posts are 8m apart. A footballer is 34m from one post and 38m from the other. Within what angle must he kick the ball if he is to score a goal.
Answered by Penny Nom.
Related rates 2018-02-11
From angelo:
hi admin please help me answer this question. thank you! At a certain instant of time, the angle A of a triangle ABC is 60 degrees and increasing at the rate of 5degrees per second, the side AB is 10cm and increasing at the rate of 1cm per second, and side AC is 16cm and decreasing at the rate of 1/2 cm per second. Find the rate of change of side AB?
Answered by Penny Nom.
Similar rectangles 2018-01-31
From Kathy:
A rectangular picture frame is 14 inches long and 4 inches wide. Which dimensions could a similar picture frame have.
8 L X 21 W
35 L X 15 W
49 L X 14 W
7 L X 3 W

Answered by Penny Nom.
Positive and negative values of a function 2018-01-30
From Grayson:
f(x)=x^6-x^4
Interval: ( negative infinity, negative one )
Test Value: negative two
Function Value f(x): positive forty eight
Interval: ( negative one, zero )
Test Value: negative one
Function Value f(x): zero
Interval: ( zero, positive one )
Test Value: positive one
Function Value f(x): zero
Interval: ( positive one, positive infinity )
Test Value: positive two
Function Value f(x): positive forty eight

What is the sign of f(x) for each Interval?

Answered by Penny Nom.
Simplify 1/2^i 2018-01-28
From Deepak:
How do I simplify this complex equation:

z=1/(2^i)

Answered by Penny Nom.
y as an expression in terms of x 2018-01-20
From Mark:
What is y as an expression in terms of x ?
Answered by pennt.
The mass of a length of tree trunk 2017-11-13
From SM:
What is the mass of a length of 1m of trunk a tree trunk of uniform cross section of area 0.5 sq.m is made of wood of relative density 0.8?
Answered by Penny Nom.
f(x)=-x^2-6x find f(x-2) 2017-10-27
From Kenneth:
f(x)=-x^2-6x find f(x-2)
Answered by Penny Nom.
Salary plus commission 2017-10-16
From Natasha:
Veronika works at Future Shop and earns ₱10.50/h plus 6% commission on sales. Last week Veronika worked 40 hours. What was Veronika’s weekly gross salary if her total sales were ₱2050?
Answered by Penny Nom.
27000001 2017-10-09
From Tulashiram:
If a× b =27000001, then what is the value of a & b ?
Answered by Penny Nom.
The square root of 6 is irrational 2017-10-02
From John:
http://mathcentral.uregina.ca/QQ/database/QQ.09.06/sylvia1.html
In the initial assumption of that proof, root 6 is assumed to be a/b where a and b have no common factors, but why does having a common factor make it irrational?

Answered by Penny Nom.
How many guests were present? 2017-09-17
From SM:
How many guests were present at an Italian dinner if every 2 guests shared a bowl of salad, every 3 guests shared a bowl of pasta, and every four guests shared a bowl of meatballs, and there were 65 bowls used altogether?
Answered by Penny Nom.
An impossible problem 2017-09-15
From Fay:
Given math homework problem of: Gary and Larry given 2 numbers and told to add together. Gary subtracted and got 14. Larry multiplied and got 799. I tried substitution:
X-Y=14 and X x Y=799
X= Y+14
(Y+14) Y=799 and here I'm stuck at Y squared+14 Y= 799
using substitution I got 47x17=799 but not 14b when subtracted. HELP?????

Answered by Penny Nom.
Simultaneous equations with fractions 2017-09-09
From Farah:
Hi, my name is Farah. I hope you can help me with this question . X/2 + g/5= 3 and 2g - f = 10
Answered by Penny Nom.
Simultaneous equations 2017-09-02
From keto:
x-y=2,x^2+xy=12
Answered by Penny Nom.
A 4 sided lot 2017-08-15
From Micheal:
If one side of .4 acre is 25 feet what would the other three sides be
Answered by Penny Nom.
Significant digits 2017-08-06
From Pat:
The directions for a problem are: "Solve the problem, expressing your answer using the correct number of significant figures."

Problem 0.0043 divided by 2.04
Answer before using the correct number of significant figures is 0.002107843

Is the correct number of significant figures the number below?
2107843

Pat

Answered by Penny Nom.
Long division with decimals 2017-06-16
From Kenneth:
Hello:

Why is it recommended to move the decimal from a divisor when dividing so that the divisor is a whole number? I know how to divide by moving the decimal from the divisor, but I cannot explain the reason.

I thank you for your reply.

Answered by Penny Nom.
Solve the equation completely cos 2x = 1 2017-06-08
From Lava:
Solve the equation completely cos 2x = 1
Answered by Penny Nom.
Simultaneous equations with fractions 2017-06-02
From Jamal:
1/x + 1/y =5 and 1/y - 1/x =-1
Answered by Penny Nom.
Yards per second to miles per hour 2017-05-11
From Scott:
If a soccer ball travels 30 yards or roughly .017 miles in 2 seconds how fast was the ball going in mph?
Answered by Penny Nom.
Successive Differences 2017-04-06
From Twaha:
Please find the equation of the sequence 1 2 4 7 11 16 22
Answered by Penny Nom.
Is a large integer divisible by 2^n? 2017-03-28
From Sahand:
You are given number x that is very large (that large that it can't be divided by hand) can we find out that x is divisible by 2^n or not?
Answered by Penny Nom.
More on calculating the area (acreage) of a four-sided lot 2017-03-06
From Kerri:

Can you please show the formula used to calculate areas as provided on this page:

Calculating the area (acreage) of a four-sided lot


Answered by Penny Nom.
The inside diameter of a silo 2017-02-28
From JOHN:
I need to find the inside diameter of a cement silo with an outside circumference 118 ft and a wall thickness of 7 inches. thank you John Kerr.
Answered by Penny Nom.
Five equations 2017-01-16
From Muhammed:
A x 4 = E
B ÷ 4 = E
C + 4 = E
D - 4 = E
___________
& A + B + C + D = 100
what is the value of E

Answered by Penny Nom.
Cutting a wire into 18 cm lengths 2016-12-29
From Shambhavi:
A long wire of some length is cut into small pieces of 216 cm leaves a small piece of length of 89 cm of this part were cut into equal pieces each of 18 cm the length of the left over wire would be
Answered by Penny Nom.
Simultaneous equations with fractions 2016-12-21
From Kimi:
I am stuck on this,can someone please help me????

1/2x+1/3y=11
8x+2/5y=102
Can someone please explain how to solve questions like these?? Or else I will never learn...
Thank you

Answered by Penny Nom.
Divisibility of 3n^5+7n 2016-12-14
From Parag:

Question from Parag, a student:

if n is a natural number,then 3n^5+7n is divisible by
a)2
b)3
c)5
d)7

i got the answer but still i need a valid alternate approach.


Answered by Penny Nom.
How far apart are the boats? 2016-12-13
From Halley:
Two boats leave port at the same time. Boat A travels east at a speed of 12 km/hr. Boat B travels southwest at a speed of 14 km/hr. After two hours, how far apart are the boats? North is 0 degrees. How do I figure this out. Thanks
Answered by Penny Nom.
The sum of three integers is 30 2016-12-06
From Vaishnavi:
_+_+_:30. Blanks are to be filled by(1,3,5,7,9,11,13,15) and numbers can be repeated.
Answered by Penny Nom.
Three friends bought 13 books 2016-12-06
From alexis:
Marina bought 4 books José bought as many books as Ben bought together the 3 friends bought 13 books how many books did Ben buy
Answered by Penny Nom.
A pie chart 2016-12-05
From vickie:
Determine the central angle needed to form a pie chart for the following housing characteristic data: 63% owner occupied, 27% renter occupied, and 10% vacant
Answered by Penny Nom.
Simultaneous equations 2016-11-25
From Rohan:
x-2/3 + y - 1/4 = 13/12
2-x/2 + 3 + y/3 = 11/6

Answered by Penny Nom.
A square inside a circle inside a square 2016-11-13
From Jeff:
Greetings,

I came across a question from a textbook from years ago. I've been trying to solve it, but am not sure if my approach is correct.

There are 2 squares (1 inner, 1 outer) & 1 circle.

The inner square is the largest square that will fit inside the circle. It has an area of 1 unit.

The circle is the biggest that will fit in the outer square.

What is the area of the outer square?

Answered by Penny Nom.
The amount of material remaining on a roll 2016-11-11
From Yoh:

Question from Yoh:

Hello,
I am trying to find impressions on a roll (either full roll or partial).
Let's say I have the following information.
- Outer Diameter of roll
- Inner Diameter of roll (cardboard core)
- thickness per layer
- Each cut off length (impression length)

Now let's say a roll has a 40in outer diameter, the cardboard core has a 3.75in diameter and a thickness of .002. Each impression has a cut off of 14inches.
With this the roll will have approximately 2,700 impressions. How would I find the remaining impressions if the outer diameter of the roll is only 6.5inches?

Thank you.


Answered by Penny Nom.
A pattern rule for a sequence 2016-10-28
From Grace:
Here's a Grade 7 question, we need to find the pattern rule:

1 - 1
2 - 3
3 - 6
4 - 10
......
It may involve exponents, but we can't seem to figure it out.
thank you.
Grace.

Answered by Penny Nom.
A word problem at the fair 2016-10-25
From Rick:
Have 48 tickets to spend, have to spend them all in exactly 10 events. Rides cost 6 tickets games cost 4 tickets & treats cost 3 tickets
Answered by Penny Nom.
20/2 + 2 * 2^2 + 6 = 8 2016-09-15
From Simone:
20/2 + 2 * 2^2 + 6 = 8

insert brackets to make this true

Answered by Penny Nom.
The radius of a coffee cup 2016-09-12
From Brett:
What is inside radius, in centimeters of a coffee cup if it holds 350g of coffee when filled to a depth of 9.5 cm? Assume coffee has the same density of water, 1.00g/cm3. A numeric value is expected and not an expression.
Answered by Penny Nom.
The dimensions of a rectangle 2016-08-31
From Hazym:
How do I find the length(which is 15m longer than the breadth) and the breadth of a rectangle just by its perimeter which is 70 m?
Answered by Penny Nom.
A suspension bridge 2016-08-18
From sai:
The main cable of a suspension bridge has the shape of a parabola. The cables are strung from the top of two towers, 61 metres apart, each 15.25 metres high above the roadway. The cable is 1.5 metres above the roadway at the point that is directly between the towers.

h(x)=a(x-30.5)^2+1.5
where a is the constant and its value to be determined.

a) determine the value of the constant a by using that the fact that height of the suspension cable, h(x) is 15.25 metres at each tower. Hence, write the updated model equation.

Answered by Penny Nom.
Two lots 2016-08-08
From Deniece:
I have 2 lots that are joined together. One lot is 38' X 105' and the other is 45' X 105'. What is the total in acre size. Or, how close are these lots to make one full acre? I want to sell both lots and want to advertise using the total lot size. Thank you.
Answered by Penny Nom.
The dimensions of a rectangle 2016-07-25
From Krunox:
A rectangle is twice as long as it is wide. Its perimeter (the sum of the lengths of its sides) is 60 cm. Find its length and width.
Answered by Penny Nom.
The number of sides of a polygon 2016-07-23
From Shriya:
All the angles of a polygon are either 155° or 140°.
There are twice as many angles of 155 °or 140°.
Find the number of sides of the polygon

Answered by Penny Nom.
Two equations with fractions 2016-07-22
From kanesh:
p/2+q/3=3

p/4+2q/3=3

Answered by Penny Nom.
The composition of a function with itself 2016-07-17
From Mel:
If f(1 + 3x) = 1 * x, solve f(f(x))
Answered by Penny Nom.
A 1.4 acre square 2016-06-28
From Jamie:
Dear Faithfullness,

I was previously to be purchasing a square 1 acre plot, which I believe was to have 209feet, 4 sides perimeter.

How big would a 1.4 acre site be?
In sq feet length ie original 209ft + (the adeed 0.4)

Answered by Penny Nom.
Simultaneous equations 2016-06-25
From sena:
2x/3+3y/2=-1
4x-5y=22

simultaneous equation

Answered by Penny Nom.
The lateral side length of a cone 2016-06-05
From Diane:
Question is find the lateral side length of a right cone with area of 372 sq. cm and base circle radius of 9 cm.
Answered by Penny Nom.
The area of a 4-sided lot 2016-05-25
From prasad:
How to find area of a land whose sides are 41ft,33ft,32.3ft and 33.2 ft. Pl give me the formula and proof.
Answered by Penny Nom.
A trough with a triangular cross section 2016-05-21
From Clarice:
A trough having an equilateral triangle end sections has sides equal to 0.4 m and 7m long.what is the volume of the liquid in the container if the depth of the water is one half the depth of the trough?
Answered by Penny Nom.
The volume of a truncated rectangular pyramidal pond 2016-05-13
From Paul:
How do you calculate a partially filled truncated rectangular pyramid if you always know the bottom rectangle, the maximum height top rectangle perimeter, but have a varying height. Similar to filling up a pond you know the current height and dimensions at the max rectangle how do you calculate it half full i.e. 10x30 outside perimeter with a 2x8 base and a max height of 6ft how do you calculate it at 3ft without re-measuring the top perimeter. Thanks, Paul
Answered by Harley Weston.
A pair of equations with fractions 2016-05-11
From tiya:
hello, i want to know how to solve this question.

m/6+2n/3=6
-m/10=2n/5=2

Answered by Penny Nom.
At what time will Sara catch up with Jake? 2016-05-07
From Laura:
At 11:00 a.m., Jake started driving along a highway at constant speed of 50 miles per hour. A quarter of an hour later, Sara started driving along the same highway in the same direction as Jake at the constant speed of 65 miles per hour. At what time will Sara catch up with Jake?
Answered by Penny Nom.
A six digit code 2016-05-06
From Ralph:
How many combinations are possible if a "safe" has four buttons and you can use a six digit code? (these are just single press buttons..)For example, this could be one possible combination: Button 4 Button 1 Button 1 Button 3 Button 2 Button 4. Thanks!
Answered by Penny Nom.
Covering a site with 6 inches of rock 2016-04-26
From Carla:
if a pump house that measures 20'x16' sits in the middle of a site that is 50'x60' and you apply crushed rock to the site at depth of 6", how many yards will you need to haul?
Answered by Penny Nom.
External and interior angles of a regular polygon 2016-04-19
From pearl:
a polygon has n sides.The exterior angle is 8 times the interior angle
find the value of the interior angle
find the value of n

Answered by Penny Nom.
Conversion of an equation 2016-04-11
From Marie:
How to you convert the equation of a line from x-intercept form (y=(x-x1)(x-x2)) to general form (y=ax^2+bx+c) if it involves square roots?

For example:
Convert this equation to general form: y=(x+the square root of 5)^2

Thanks for the help!

Answered by Penny Nom.
Filling a hole with 14 tons of rock 2016-04-08
From Barry:
If I had 14 tons of inch and one half sized aggregate rock, how large of a square or rectangular shaped hole would I need to hold that amount?
Answered by Penny Nom.
The remainder theorem 2016-03-27
From Pratyasha:
A quadratic polynomial when divide by x+2 leaves a remainder of 1 and when divided by x-1 leaves a remainder of 4. What will be the remainder if it is divided by (x+2)(x-1)?
Answered by Penny Nom.
Polynomial division 2016-03-25
From Ashley:
(4x-4+6x^3-x^2)/(3x-2)

The instructions say to "perform the division".

Answered by Penny Nom.
A rotating schedule for a softball team 2016-03-25
From Caroline:
I couldn't find a similar question, hopefully there isn't one!

I have a softball team of 18 people (7 girls and 11 boys), but am limiting each game to 13 players (6 girls and 7 boys). The season is 10 games long. How do I create a rotating schedule which allows members to play evenly? I tried separating the boys and girls to create rotation for each but I got confused. Some are unable to attend all games but for the simplicity of a rotation schedule can you please help?

Answered by Robert Dawson and Victoria West.
The sides of a triangle 2016-03-15
From Hitarth:
Why we cannot construct a triangle ofngiven sides 5cm,5cm and 10cm?
Answered by Penny Nom.
Can you construct a regular 21-gon? 2016-03-08
From Peter:
hi, i know that if i use a compass to draw a circle, i can use the same setting to mark 6 intervals along the circle circumference, how would i go about marking out 21 equally spaced points around the circumference?

if there an online calculator i can us, or some other interesting trick?

tia

peter

Answered by Robert Dawson.
A geometric progression 2016-03-03
From Pauline:
A woman measures the height of her child at birth and at monthly intervals afterwards.The child's height increases by 5% per month. Find the number of measurements she has made before the child's height is twice what it was at birth
Answered by Penny Nom.
Solve for x and y 2016-02-27
From ntshidi:
Y=1/2x+4and1/4x-6
Answered by Penny Nom.
How can I express Sin 4A in term of Sin A? 2016-02-22
From Odukoya:
How can I express Sin 4A in term of Sin A?
Answered by Penny Nom.
A bonus of 8% of sales 2016-02-22
From Barghavi:
A man earns $325 per week plus an additional 8% on any sales over $500. If he makes $6,250 in sales, how much money in total does he earn that week? thank you!
Answered by Penny Nom.
0/0 2016-02-14
From Noni:
Q. 0÷0=1 Why it is wrong when 4÷4=1; 3÷3=1; 2÷2=1: 1÷1=1 ?
Answered by Penny Nom.
Two equations in two unknowns 2016-02-13
From Anumba:
4x + 2y = 4
7x - y = -11

Answered by Penny Nom.
The midpoints of the sides of a quadrilateral 2016-02-05
From Andrea:
The segments, joining, in order the midpoints of consecutive sides of a quadrilateral form a parallelogram.
Answered by Penny Nom.
The height of a triangle 2016-02-03
From Brooklyn:
How do I figure out the height of a triangle when it only tells me the base of he triangle? And vice versa.
Answered by Penny Nom.
Division base 5 2016-02-01
From Jenalee:
I am having problems with a dividing bases question. I reviewed two that I could find in the database and tried the question as shown but I am still getting the wrong answer.

My question is 32\24043 to the base of 5 for both.

Answered by Penny Nom.
Simplify 2016-02-01
From Sabira:
Simplify step by step
5x/8 + 7y/18 = 6

Answered by Penny Nom.
A system of linear equations 2016-01-24
From kareem:
my name is kareem
and i am a student i have a math puzzle and i tried to solved it but it always have same mistake
x-y=9
x+z=12
z-n=14
y+n=2

Answered by Penny Nom.
The volume of a silo 2016-01-18
From Clinton:
Can you show me how to calculate the ground corn capacity in a steel silos of 10' x 10' high and 10' x 20' high and 10' x 24' high and 10' x 28' high
Answered by Penny Nom.
The perimeter of a one quarter acre square 2016-01-15
From Kerry:
If you have a perfect square that is one quarter of an acre what are the length of the sides and how do you arrive at the solution? thank you, Kerry.
Answered by Penny Nom.
A sequence 2016-01-05
From Mia:
the next three terms in each sequence. 0.4, 0.54, 0.68, 0.82,
Answered by Penny Nom.
Constructing simultaneous linear equations 2015-12-28
From Deborah:
Linda thinks of a two-digit number. The sum of the digits is 8. If she reverses the digits, the new number is 36 greater than her original number. What was Linda's original number?

Thank you!

Answered by Penny Nom.
Converting a fraction to a percent 2015-12-22
From Sharon:
How do you turn a fraction into a percentage? I'm beyond confused with this. Even though I think of myself as a good math student doing 8th grade math at a young age, this just makes me want to scream! I get the way you turn 1/4 into 25%, but when it comes to something like 7/9 = ? I just can't figure out the answer.
Answered by Penny Nom.
The angles of a triangle 2015-12-17
From Faith:
Does the measure of angle determine the length of its side? For example two angles are congruent then the sides are also congruent because from my understanding the angle determine the shape of triangle.
Answered by Penny Nom.
Sierpinski Pyramid 2015-12-14
From victor:
Is a tetrix (a Sirpienski Pyramid/Tetrahedron) its own hyperplane?...alternatively, is it a hyperplane?
Answered by Robert Dawson.
Linear equations in two variables 2015-12-13
From priya:
I have problem in solving these equations please help me today itself very urgent:
I)2x+y=y
II)pie*x+y=9

Answered by Penny Nom.
Travelling in opposite directions 2015-12-06
From Roselyn:
Jenny and Mark commute to work, travelling in opposite directions. Jenny leaves the house at 8 AM, and averages 35 kph. Mark leaves at 8:15 AM, and averages 40 kph.At what time will they be 40 kilometers apart?
Answered by Penny Nom.
Dividing by a fraction 2015-11-21
From Jennifer:

Question from Jennifer, a student:

I want to know how to get the right answer for this math problem, dividing a fraction in to a whole number , example 3/8 using the whole number 6? Thanks


Answered by Penny Nom.
A volume in cubic yards 2015-11-12
From James:
The first formula for figuring cubic yards of concrete need is length in feet, times width in feet, times depth in actual inches, divided by 320. Such as, 150x 50 feet x 4inches divided by 320 = cubic yards. How and why does this formula work?
Answered by Penny Nom.
Simplify -3(x-7) 2015-10-21
From geraldine:
simplify: -3(x-7)
Answered by Penny Nom.
A six team league to play a 20 game schedule 2015-10-19
From Kelly:
Hi there. Some of your posts are close, but my problem is evening out the home and away games. We have 6 teams in our league and we are going to play a 20 game schedule, playing everyone 4 times. We will play Fridays and Saturdays, each weekend. Of course the ideal would be to have each team have a home and an away game each weekend, but I realize that is impossible, but I would like it as close as possible. Any help would be much appreciated!! We also were considering opening with a home and away - Friday then Saturday with the same teams. Then carrying on with playing everyone else and then possibly finishing up with the same home and home
Answered by Victoria West.
Six teams playing five games 2015-10-18
From Aidan:
I have six teams that will be playing five games.
I want each team to play each game once and each team to play each other team once.
Is this possible or should I add an extra round and let some teams play each other twice?

Answered by Victoria West.
The dimensions of a rectangle 2015-09-25
From Erick:
A rectangular strip of land has a perimeter of 124meters and an area of 385 square meters. Find its possible dimensions
Answered by Penny Nom.
A 6-person team that has 9 players 2015-09-21
From Sheri:
I am organizing a 6-person team that has 9 players. How do I schedule them fairly over a 9 week league?
Answered by Victoria West.
Seven switches with 3 stages each 2015-09-15
From jeff:
We are trying to solve a combination problem. If I have 7 things that can be in 3 states (say on/partially on/off), and all 7 have to a state. How many different combinations can there be? It doesn't seem like we should treat it as 21 independent things, but not sure how to break the problem apart.
Answered by Penny Nom.
Division and units 2015-09-08
From Kenneth:
Hello:

Would there be a reason to divide a number without units by another number with units? Here is an example:

35 divided by $5.00 or 35/$5.00

The answer cannot have units ($) because $5 X 7 does not equal 35 but instead $35.00.

$35.00 divided by 5 or ($35.00/5) and $35 divided by $5.00 or ($35.00/$5.00) make more sense.

I thank you for your reply.

Answered by Robert Dawson.
Factor x^2 + y^2 2015-08-26
From Jerome:
Factor: x^2+y^2
Answered by Penny Nom.
Four equations in 3 unknowns 2015-08-23
From damien:
solve:
S x U - T = 44
T - U x S = 36
U + T - S = 7
U x U / S = 4

Answered by Penny Nom.
The length of a shadow 2015-08-01
From maaz:
Hello
I am having trouble with this question:

Lizzie, who is 6 feet tall, stands in her driveway at night, exactly 24 feet from the base of a spotlight, and casts a shadow that is 12 feet long if her friend Hannah who is 5 feet tall decides to stand next to lizzie how long will her shadow be?

Answered by Penny Nom.
1 + 1 + 1 + 1 + 1 + 1 + 1 + 1 + 1 + 1 + 1 + 1 x 0 + 1 = ? 2015-06-18
From Sharon:
1 + 1 + 1 + 1 + 1 + 1 + 1 + 1 + 1 + 1 + 1 + 1 x 0 + 1 = ?

I got 1 as my answer despite BODMAS making it 12 because logic tells me I ought to place brackets around the first set of repeated addition. Could you please clarify this for me? Thank you 😊

Answered by Harley Weston.
An octagonal frame around a pool 2015-05-17
From James:
I have a 20' pool and need to put a frame around it using 2by 4 what r the lengths and angle cuts
Answered by Penny Nom.
An angle of depression problem 2015-05-14
From haxir:
find the height of the balloon directly above a town if the measure of angle of depression of another town 5km from the first town is 20°!6`
Answered by Penny Nom.
Repeated decimals 2015-05-09
From Vir:
Years ago I (re?)discovered 'cyclic division'. For example: if you arrange the number along a circle and put the number 142857 at the centre all the numbers taken cyclically, starting with 1, are fully divisible by 37. Whatever the starting point of this number, it remains fully divisible by 37. what is more, the number can be formed by taking the digits clockwise as well as anti-clockwise..This I call "full cyclic divisibility". In many cases, only clockwise cyclic divisibility is possible. But I have not come across a case where ONLY anti-clockwise divisibility occurs. Thus clockwise cyclic divisibility seems to be favoured. Could this be construed as a sign of chirality in mathematics?..
Answered by Chris Fisher.
The number of possible musical notes using an n-key instrument 2015-05-04
From Farihin:
Lets say that i have keys, and each key is for notes of a musical instrument, So i wanted to find out the number of notes i can get for a certain number keys, of course in the form of an equation. Notes can use as many keys, it can use 1, or 2, or 3, or even 100.
Notes in real life is not as such, but ignore reality. I tried doing this but i can't seem to find a formula for it. For example, i have 4 keys, say A, B, C, and D. so, for notes that uses one key are 4, which is A, B, C, and D themselves. for notes that uses two keys are 6,
AB, AC, AD, BC, BD and CD.
for notes that uses three keys are 4,
ABC, ABD, ACD and BCD.
lastly for notes that uses all four keys is 1, ABCD.
So, the total will be 4+6+4+1=15#

The nth term for the first equation is n, the second is [(n^2)-n]/2 the third and the fourth, i don't know but the final answer should be like,
n + [(n^2)-n]/2 + [3rd] + [4th]

Sorry for the long question though...

Answered by Penny Nom.
13 games, 3 possible outcomes 2015-05-01
From Kennedy:
Hello, hope it finds you well.
Am not that good with numbers, can you please help me get all the possible outcomes of 13 games, with a home win, draw or an away win as three possible results for each of the 13 games.

Thanks in advance

Answered by Penny Nom.
The method of elimination 2015-05-01
From oreanna:

Question from oreanna, a student:

How do u solve 2x+9y=3

7x-4y=-25 in elimination


Answered by Penny Nom.
Expand and Simplify (3x+5)(5x-9) 2015-04-23
From chloe:
Expand and Simplify (3x+5)(5x-9)
Answered by Penny Nom.
How many boats total are in the marina? 2015-04-15
From Carla:
In a marina, 3/4 of the boats are white, of the remainder 4/7 are blue, the rest are red. There are 9 red boats. How many boats total are in the marina? My answer is 84 because if 1/4 of the boats are divided into sevenths, that makes the whole marina a 28 part item. 3/28 are red so 84 is the total number of boats. My child thinks I am wrong.
Answered by Penny Nom.
Two airplanes 2015-04-14
From john:
two planes leave an airport at the same time, one going northwest (N35*W)at 400 mph and the other going east at 332 mph. How far apart are the planes after 4 hours to the nearest mile?
Answered by Penny Nom.
Some non-decreasing sequences 2015-04-13
From Sky:
Hi I'm a Student and i'll try my best to state the problem perfectly.

The number of non-decreasing sequences of size at least 1 and at most N, such that each element of the sequence lies between L and R, both inclusive.

Eg:- N=1 L=4 R=5
ans= 2. [{4},{5}]

N=3 L=4 R=6
ans= 19. [{4},{5},{6},{4,4},{4,5},{4,6},{5,5},{5,6},{6,6} {4,4,4},{4,4,5},{4,4,6},{4,5,5},{4,5,6},{4,6,6},{5,5,5},{5,5,6},{5,6,6} ,{6,6,6}]

Answered by Claude Tardif.
A word problem with fractions 2015-04-09
From Lorraine:
If the numerator of a certain fraction is doubled and the denominator is increased by 1, the fraction becomes 1/2.

If the numerator of the original faction is squared and the denominator is decreased by 2, the fraction becomes equal to 1.

Let x be the numerator and let y be the denominator of the original fraction. Write down two simultaneous equation in x and y.

Solve these equations to find two possible values for the given fraction.

Answered by Penny Nom.
The weight of a steel block 2015-04-04
From Mark:
the weight of a steel cube 37 inches X 28 inches X 7 inches
Answered by Penny Nom.
The side length of a gazebo 2015-03-14
From Dennie:
From middle of side to middle of opposite is 14'6". What is the length of the side?
Answered by Penny Nom.
Commission 2015-03-13
From Rose:
A shopkeeper receives 12.5% (12 1/2 %) commission on the sale of books from the publisher. In a certain month,he receives a commission of rs.250 from the publisher. How much was his sale for that particular month??
Answered by Penny Nom.
Angles of elevation and depression 2015-03-08
From Timmy:
Joel is walking down a street and sees a 115 ft tall building in front of him. He stops 190 feet from the base of the building at the tip of the building's shadow. Round answers to three decimal places.

A. If there was a piece of rope from the top of the building to Joel, how long would it be?
B. What is the angle of elevation from Joel to the top of the building?
C. Margaret says that she could find the angle of depression from the top of the building to Joel by subtracting the angle of elevation from 90°. Is she correct? Explain.

Answered by Penny Nom.
The definition of percent ( % ) 2015-03-07
From Kenneth:
Hello:

I saw this definition online from the Free Dictionary (The Free Dictionary.com) regarding the definition of percent (%):

"Noun 1. a percent sign - a sign (`%') used to indicate that the number preceding it should be understood as a proportion multiplied by 100."

What is this "proportion" multiplied by 100?

When I think of a proportion, I think of a/b = c/d. How can a number preceding the percent sign be understood as a proportion X 100? I thank you for your reply!

Answered by Chris Fisher and Claude Tardif.
A man is standing on top of a building 2015-03-06
From kiki:
a man of height 1.5m standing on top of a building of height 48.5m views another building across the square. he observe that the angle of depression of the bottom of the building is 40 degrees and the angle of depression of the top of the building is 5 degrees. Both buildings stand on the same level
a) calculate the distance of the man from the base of the building across the square measured along the line of sight

Answered by Penny Nom.
Two consecutive positive integers 2015-03-01
From meha:
Find two consecutive positive integers , sum of whose squares is 365.
Answered by Penny Nom.
Two equations with fractions 2015-02-26
From Pulane:
Hi math centre I've been trying to solve these equations for days now please help (6/x)-(1/y)=4
(9/x)+1=(-2/y)

Please help me solve them simultaneously
Thank you

Answered by Penny Nom.
3 over "x" plus 5 2015-02-23
From Kenny:
3 over "x" plus 5. I don't know this is either a polynomial and if so how would you define it.
Answered by Harley Weston.
Two equations 2015-02-16
From nigel:
2x+1/2y=1
6x-3/2y=21

Answered by Penny Nom.
A stone is dropped in a lake 2015-02-14
From Wendy:

Hi, I have problems solving this problem. We didn't exactly go over these kind of problems and the book doesn't have an example either. Thank you for your help!

A stone is dropped in a lake, creating a circular ripple that travels outward at a speed of 80 cm/s.
(a) Find a function g that models the radius as a function of time t.

g(t) =

(b) Find a function f that models the area of the circle as a function of the radius r.

f(r) =

(c) Find f compose g.
f compose g =


Answered by Penny Nom.
The volume of Lake Utah 2015-01-29
From Hannan:
Lake Utah has a surface area of 3,846 square miles and an average depth of 10.5 feet. In cubic miles, how much water does it hold? How would you approach this question? Where would you start?
Answered by Robert Dawson and Penny Nom.
128/(-16)/(-2) 2015-01-28
From jackie:
128/(-16)/(-2) I was wondering if you can show me how to work this question out
Answered by Harley Weston.
Is a square a trapezoid? 2015-01-26
From Katie:
Can a trapezoid sometimes be a square?
Answered by Penny Nom.
sin x = cos x 2015-01-23
From Mubashir:
there are two curves y=sinx and y=cosx intersecting at point where x is less than 90 and greater than 0.to find the point I made a equation sinx=cosx but donot know what to do further .please help me by solving this equation sinx=cosx also tell me that have you solved this equation by reference to the graph or by other means please do elaborate your solution in your answer.
Answered by Penny Nom.
The height of a building 2015-01-20
From Emily:
A man 2m high observes the angle of elevation to the top of the building to be 70 degrees. And the angle of depression to the bottom of the building to be 19 degrees. How tall is the building?
Answered by Robert Dawson.
Pumping water from a borehole 2015-01-18
From patience:
An empty tank for storing water from a borehole has a volume of 480 cubic metres. If it is filled by a pump that pumps water at a rate of 16 litres per second, how many hours will it take the pump to fill this tank please show all calculations
Answered by Penny Nom.
Mutually prime composite numbers 2015-01-09
From nivedita:
Two composite number that is mutually prime?
Answered by Penny Nom.
Missing digits in a phone number 2015-01-07
From Janelle:
I had a friend try and play a game on me, regarding phone numbers, so I only know the area code , and the last four digits of the phone number. Which means, there are 3 digits that I need to figure out all the possibilities to.
Answered by Penny Nom.
A silo is composed of a hemisphere and a cylinder 2015-01-03
From Ami:
A silo is composed of a hemisphere and a cylinder. The total height for the silo is 20m and the diameter of the silo is 8m. What is the volume of the silo to the nearest cubic meter?
Answered by Penny Nom.
1 + 2 + 3 + ... + (2n - 1) 2015-01-01
From Brian:
Hi Maths Central
My wife presented me with a query which may have a simple answer, but one that I can’t deduce or explain.

Take the string of numbers, n=1,2,3,4,5…
It seems that n(2n-1) = Sum((1…..(2n-1))
e.g. for n=5, both 5 x 9 and Sum(1….9) equal 45, and so on for other values of n.

Could you please provide an explanation? Does it have an underlying reason and a name?

Look forward to your response.
Brian

Answered by Penny Nom.
Rates, percentages and units 2014-12-30
From Kenneth:
Hello:

If percentages have no units, why are some percentages called rates, as in interest rate, or perhaps a tax rate of 7% as an example? A rate has units of different quantities.

I thank you for your reply.

Answered by Robert Dawson.
Remainder 2014-12-08
From Muqadasah:
A number, when divided by136, leaves remainder 36. If the same number is divided by 17, what will be the remainder?
Answered by Penny Nom.
How do you change a decimal to a ratio? 2014-11-29
From Esmeralda:
How do you change a decimal to a ratio? Express the decimal 0.2 as a ratio.
Answered by Penny Nom.
An arithmetic progression 2014-11-19
From Gbenga:
In an A.P the difference between 8th and 4th term is 20. The 8th term is 1\2 times the 4th term . Find arithmetic progression..
Answered by Penny Nom.
How many square feet in 10 gallons of liquid at 1/4 in thick? 2014-11-14
From Rich:
How many square feet in 10 gallons of liquid at 1/4 in thick
Answered by Penny Nom.
1/6k=-11 2014-11-13
From Marty:
Using the balance method, what is the answer to 1/6k=-11, trying to find what k equals. Thanks
Answered by Penny Nom.
The density of sea sand 2014-11-02
From Lenny:
Hi There,

I need to find out the the density of sea sand to convert one metric ton to cubic meter as I am submitting for a tender which is in cubic meter but the requirement is in metric ton. I searched through the web site but no density of sea sand is given though there is specific Gravity of dry sand is available. I was given a density of 1.6 to work with. If that is true am I right to say that 1 cubic meter of sea sand is equivalent to 1.6 metric ton? Please help as I am getting confused with what I am able to find out on the web.
Thanks much in advance
Rgds
Lenny

Answered by Penny Nom.
A linear system 2014-10-14
From pheter:
4/x - 1/y = 3 .... equation (1)
6/x - 2/y = 5 .... equation (2)

Answered by Penny Nom.
A fact family containing 4, 4 and 16 2014-10-14
From Leslie:
I am helping my son and the question is 4,4,16 in fact family what does that mean
Answered by Penny Nom.
Building an 8 sided box 2014-10-14
From Michael:
I want to build an 8 sided box. The North, East, South, and West sides I want them to be 4 feet in length.
The NE, SE, SW, and NW sides I want to be 2 feet in length. What angles do I need to cut my angles.
Thanks Michael

Answered by Penny Nom.
Sieve of Eratosthenes 2014-10-03
From Hope:
using Sieve of Eratosthenes to determine for in instance prime numbers between 1 to 200, to what nth number should i stop?, 4th, which is 11 or what?
Answered by Robert Dawson.
Trig functions and the unit circle 2014-10-02
From Jake:
I was wondering what conclusions can be drawn about the trigonometric functions and how they work about the circle. Can you also please give me an explanation for it? Thank you.
Answered by Penny Nom.
Division in base 3 2014-09-30
From Nnubia:
1021 in base 3 divided by 2 in base 3
Answered by Penny Nom.
A rectangle and a triangle 2014-09-27
From rajesh:
The perimeter of a rectangle is equal to the perimeter of a right-angled triangle of height 12 cm if the base of the triangle is equal to the breadth of the rectangle, what is the length of the rectangle?
Answered by Penny Nom.
Two equations in x and y 2014-09-25
From seyilogo:
solve y=2x - 3 and (4x - 2y) / x + y = 1 simultaneously
Answered by Harley Weston.
Two equations in two variables 2014-09-18
From Susan:
(28x + 36y) - [20000 - .75(28x + 36y) + 60000] = 5000
x + y = 10000
solving two equations involving variables

Answered by Penny Nom.
A six team track schedule 2014-09-15
From Paul:
I need to make a six (6) team track schedule. It needs to be five (5) triangular (3 team) meets with each team playing each team twice
Answered by Victoria West.
Cutting a round cake so that it doesn't dry out 2014-08-26
From James:
I'm wondering if there's a simple way to calculate the area between two parallel chords of a circle equidistant from its diameter, or if I have the area, to find the distance between the two chords.
Here's my "problem". You may have heard of the way of cutting a round cake so that it doesn't dry out - make two parallel cuts (chords) the length of the cake, take the middle piece, then push the two pieces together.
So I know the area of a 12" cake, and I want say, exactly an eighth of the cake. How wide do I cut the centre piece? Now to get even more difficult, the next day I want another eighth from the centre. How wide do I cut the next pieces, and so on...? Thanks, James

Answered by Harley Weston.
Revisiting 8 golfers in two foursomes 2014-08-19
From steve:

I noticed you had an answer on how can you put 8 golfers in even foursomes over 7 games so that everyone plays with everyone equally. However the formula showed the following sequence that was used but did not go past the second day. Can you change the 0s and 1s into the numbers 1 to 8 instead. Here is what you have written:
Day 0 : (0, 0, 0, 0, 1, 1, 1, 1)
Day 1 : (0, 0, 1, 1, 0, 0, 1, 1)
Day 2 : (0, 1, 0, 1, 0, 1, 0, 1)
Day 3 : (0, 1, 1, 0, 1, 0, 0, 1)
Day 4 : (0, 0, 1, 1, 1, 1, 0, 0)
Day 5 : (0, 1, 0, 1, 1, 0, 1, 0)
Day 6 : (0, 1, 1, 0, 0, 1, 1, 0)

Thank you so much for your anticipated help.

Steve


Answered by Harley Weston.
A barge of triangular cross section 2014-08-18
From tushar:
a barge of triangular cross section is 20m long 12 m wide and 6m deep.its floats in SW at a draft if 4m find its displacement
Answered by Penny Nom.
How do you convert square mm to square cm? 2014-08-13
From David:
How do you convert square mm to square cm?
Answered by Tyler Wood.
4821x14y is an 8-digit number divisible by 72 2014-08-06
From RAYA:
if 4821x14y is an 8-digit number divisible by 72. How many values can x and y take?
Answered by Penny Nom.
The dimensions of a rectangle 2014-08-03
From Jamaica:
if the perimeter of a rectangle is 30m and its area is 56 m2.find its length and width?
Answered by Penny Nom.
The sum of the first 50 terms of an arithmetic progression 2014-07-26
From Joshua:
Hello ...my is Joshua...I'm a grade 11 student...I got a question

Calculate the sum of the first 50 terms of an arithmetic progression: 112:98:84

Answered by Penny Nom.
An octagon around a tree 2014-07-16
From floyd:
I want to build an octagon 4 feet from base of tree from 4x4's. What length do I cut my pieces
Answered by Penny Nom.
Solve sin3x = cos2x 2014-07-14
From Emanuel:
If sin3x=cos2x, find the value of x when x<0<90'
Answered by Penny Nom.
A 5 sided lot 2014-07-10
From mia:
have the feet numbers of three sides, but the forth side is in a "V" so it has two figures. Question: can i add the two figures of the "v" together to get one figure, so then I would have the figures for all four sides.
Answered by Harley Weston.
67 cubic yards in cubic metres 2014-07-09
From gavin:
Convert 67 cu yards into cu metres
Answered by Penny Nom.
The method of elimination 2014-07-05
From leo:
please explain how can i solve this problem

3x-6y=-38
6x-9y=44

using elimination and simultaneous method thank you :)

Answered by Penny Nom.
A six digit number 2014-06-30
From anupam:
hi,
my daughter asking a solution for the below question, kindly help.

Write the smallest 6 digit number wit digits ( 2,4,3,0,5,7).

plz this is an urgent requirement.

regards,
anupam

Answered by Penny Nom.
Simultaneous equations 2014-06-20
From rana:
solve the simultaneous equations
a)3x=7y
12y=5x-1

Answered by Penny Nom.
Two regression lines 2014-06-14
From catherine:
Given a set of data values ,we can get two regression lines .Explain.
Answered by Robert Dawson and Penny Nom.
A small solid silver pyramid 2014-06-13
From Jenesis:
For Awards Night at Baddeck High School, the math club is designing small solid silver pyramids.The base of the pyramids will be a 2 in by 2 in square. The pyramids should not weigh more than 2.5 pounds. One cubic foot of silver weighs 655 pounds. What is the maximum height of the pyramids?
Answered by Penny Nom.
The weight of salt in a basket 2014-05-17
From Sumit:
I want to determine how many kg of salt or XYZ thing can be filled in a basket
Length = 4.5"
Breadth = 5"
Height = 6"

Even its formulae

is kg quantity can be different for semi solid thing

Answered by Penny Nom.
The length of a shadow 2014-05-09
From vijay:
A girl of height 80 cm is running away from the base of a lamp-post at a speed of 2 m/s. If the lamp is 4 m above the ground, find the length of her shadow after 5 seconds.
Answered by Penny Nom.
A cone of vision 2014-04-29
From David:
It is known that a fish in water looking up has a 97 degree "cone" of vision that sees "through" the surface of the water. If a fish lies 4 inches below the surface, the cone forms a window (circle) smaller than if a fish lies 8 inches below the surface. What is the ratio of inches of depth to the radius of the circle on the surface that is its visual window?
Answered by Penny Nom.
A triangular chicken pen 2014-04-27
From Cierra:
Margaret has two lengths of fence, 20 meters and 24 meters, for two sides of a triangular chicken pen. The third side will be on the north side of the barn. One fence length makes a 75° angle with the barn. How many different pens can she build if one fence is attached at the corner of the barn? What are all the possible lengths for the barn side of the pen?

Not sure what they are asking here... please show step by step what to do! Thank you so much!

Answered by Penny Nom.
The derivative of sin(x) 2014-04-26
From Lucky:
f(x)=Sin(x), by first principle its f'(x)...show me how to solve such problem.
Answered by Penny Nom.
Simultaneous equations with fractions 2014-04-19
From Maryam:
I looked at your example of simultaneous equations with fractions and applied it to my question from an educate exam papers but I couldn't get it to work. The question is:

x/8 - y = -5/2
3x + y/3 = 13

Answered by Penny Nom.
A slab of wet clay 2014-04-12
From Karen:
How many square feet ( at 1/4 inch thickness) are in 100 lbs. of wet clay?
Answered by Penny Nom.
Two cones 2014-04-09
From c.j:
what is the length of the radius of the LARGER cone(the LARGER cone has a slant height of 15) when the SMALLER cone has a radius of 8 and a slant height of 12ft ,please help.
Answered by Penny Nom.
The sides of a triangle 2014-04-06
From Michael:
I am supposed to sove for the length of side "b" of an irregular triangle. I am given the following:
Side a: 65'
Side b: Find this length
Side c: 50'
Angle A: unknown
Angle B: unknown
Angle C: 52 degrees
I am supposed to use the law of cosines to solve for side "b" and my teacher says there is no mistake in the "givens" for the problem. I do not see how this can be done using the law of cosines and i have not figured out how to sove for angle B to use the law of cosines.

Answered by Penny Nom.
A 4 digit number 2014-04-04
From LIM:
"A" is a 4 digit number formed by all the numbers from 1 to 4. When "A" is divided by 9, the remainder is the biggest possible value. What is the biggest value of A?
Answered by Chris Fisher.
Shadows of a father and son 2014-04-02
From esada:
the father and son cast a shadow of 11 feet and 8 feet,respectively if the son is 4'8'' in height,how tall is the father?
Answered by Penny Nom.
Reducing the size of a poster by a scale factor of 1/3 2014-04-01
From deeshon:
eric is making a smaller copy of a poster that measures 20.4 cm wide by 34.2 cm long.he reduces the original by a scale factor of 1/3.what are the dimensions of the copy
Answered by Penny Nom.
An arithmetic progression 2014-03-31
From Japheth:
The 3rd term of an A.P is 10 more than the first term while the fifth term is 15 more than the second term. Find the first term?
Answered by Penny Nom.
A formula for determining concrete volume in cu. yd. 2014-03-19
From Steven:
I know a couple of formulas for determining concrete volume in cu.yd. the one I use most frequently is length in feet x width in feet x thickness in inch. Divided by 324. It works every time. Where does the number 324 come from.
Answered by Penny Nom.
The area of a 5 sided lot 2014-03-15
From Michael:
Question from michael:

This lot is in feet. 59x154x109x188x137 per the plot plan

Answered by Harley Weston.
A parabolic suspension bridge 2014-03-11
From jeffrey:
the towers of a parabolic suspension bridges 200 meter long are 40 meter high and the lowest point of the cable is 10 meter above the roadway.Find the vertical distance from the roadway to the cable at 50 meter from the center.
Answered by Penny Nom.
Simple interest 2014-03-08
From lynn:
I need to find what the interest will be for: 5000.00 @ 16.5% annual rate, and the same for 10.8 % for 5000.00 annual rate all one question

thank you for your aid

Answered by Penny Nom.
The angle of depression 2014-03-08
From Ranger_minor:
A woman of height 1.4 metres standing on the top of a building 34.6 metres high views a tree some distance away. she observes that the angle of depression of the bottom of the tree is 35 degrees and the angle of depression of the top of the tree is 29 degrees.

assume that the building and the tree are on level ground :
1). calculate the distance of the woman from the top of the tree measured along her line of sight.

Answered by Penny Nom.
Simultaneous fractional equations 2014-02-15
From benjamin:
hi math central. benjamin here. during class, i had problem with this topic. normally i wont have problems with math but this topic i just too hard for me. please help i am having exam and test next week on this topic

here is the question:
using substitution method, solve the simultaneous equation.
(x+1)/(y+2)=0.5

(x-2)/(y-1)=1/3

Answered by Penny Nom.
Converting cost per cubic yard to cost per ton 2014-02-05
From Mayra:
If 18 cubic yards= 25 tons and it's $28.00 per yard how much is it a ton. I need help please a refresher on my algebra. Thank you
Answered by Penny Nom.
Two nonlinear equations 2014-01-26
From Naryn:
(1÷x) + (1÷y) = (7÷12)
xy = 12

Answered by Penny Nom.
When would one flip the inequality sign? 2014-01-09
From Natasha:
Would one flip the inequality symbol in this equation: (explain why) (-9a)/(-9) > 81
And please explain in what circumstances one would flip the inequality sign
THANKS!

Answered by Penny Nom.
An arithmetic sequence 2014-01-06
From paris:
find the twenty-fifth term of an arithmetic sequence whose first term is 12 and whose common difference is -6
Answered by Penny Nom.
25% profit 2014-01-02
From Finn:
Hello,
The question is all about buy-and-sell business.
Problem:
Pencil - $6 for whole sale price
     $8 if I sell the item How do I get the 25% profit? (you can change the whole sale price and the retail price[if i sell the item])
if I buy the pencil at 24 pieces and sell it at 24 pieces.

Answered by Penny Nom.
0.35 as a fraction 2013-11-28
From Laura:
How do you write 0.35 as a fraction or mixed number?
Laura

Answered by Penny Nom.
Substitution type simultaneous equations 2013-11-03
From Kayla:
I am having problems with substitution type simultaneous equations, when the variable you are substituting is a algebraic one:
y=x^2-3x+4 and 3x-2y=1
I have rearranged 3x-2y=1 to get x=(1+2y)/3 but when I substitute this x value into the other equation, I get the wrong answers!
Would appreciate any help! Thank you.

Answered by Penny Nom.
Proportional rates 2013-10-10
From Varsha:
A province's Ministry of Social services has found that both the number of people needing social assistance and the province's total expenditures on social assistance are proportional to the rate of unemployment. Last August when the provincial unemployment rate was 8.4 %, the province provided assistance to 89,300 individuals at a total cost of 4107.4 million. The forecast unemployment rate for next August is 7.9%. How many people can the province expect to need social assistance next August? What amount should the province budget for social assistance in August?
Answered by Penny Nom.
How many cubic centimetres are there is a cubic meter? 2013-10-10
From Larry:
Change 1meter(to 3rd power) to centimetre(to 3rd power). This is a metric conversion problem
Answered by Penny Nom.
An Arithmetic Progression 2013-10-08
From collins:
In an A.P. the difference between the 8th and 4th term is 20 and the 8th is one and half times the 4th term... what is the common difference and the first term
Answered by Penny Nom.
A triangle problem 2013-10-02
From raneem:
ABC is a triangle in which : BC=20cm. M(<B) =29 and m(<C)=73 . D is the midpoint of BC Find the length Of AB and AD approximated to 2 decimal places
Answered by Penny Nom.
A schedule for 2, 7 team divisions 2013-09-24
From Sampson:
I'm looking to set up a weekly schedule within a 22 week segment of 2 7 team divisions where every team plays every other team in their divison 3 times and have 4 cross over games with 4 of the other 7 teams in the other division. I've yet to figure this out. Thanks for your time.
Answered by Victoria West.
Miles per minute to miles per hour 2013-09-08
From Adam:
Convert 250 miles per min to miles per hour
Answered by Penny Nom.
How we convert cm to mm? 2013-09-08
From Génesis:
How we convert cm to mm?
Answered by Penny Nom.
The sum of all whole numbers from 1 to X 2013-09-06
From Tim:
How do I develop a rule for the sum of all whole numbers from 1 to X when I have no idea how to do this
Answered by Penny Nom.
A sequence or a series? 2013-09-05
From Rahul:
Whether the following sequence is increasing or decreasing? I even do not kow whether to call it a sequence or not. an = (1/(1+n))+(1/(2+n))+....+(1/(n+n)). I am confused. It looks like a series to me. Please help.
Regards,
Rahul

Answered by Robert Dawson.
Tesseract 2013-08-22
From Dan:
I am not a mathematician. This seems to me an intuitively simple enough problem that I very much need an answer to from someone who's mathematics are better than mine. Please help.

The question is: for a tesseract of side length = 1 what is the distance of the center of each cube from the center of the tesseract ?

I think I have calculated the distance of each vertex from the center, and of the center of each edge from the center, but the question above baffles me.

(anyone not having a clue what I am talking about can brush up here http://en.wikipedia.org/wiki/Tesseract )

Thanks in advance - Dan V

Answered by Robert Dawson.
Similar rectangles 2013-08-17
From Mattie:
The dimensions of a rectangular-shaped picture frame are 14 inches long and four inches wide. Which dimensions below represent another frame that i geometrically similar>

a. l=49 in. and w=14in.
b. l=7in and w=3in
c. l=21in and w=8in
d. l=24in and w=14in

thanks

Answered by Penny Nom.
Using trig to find the height of a hill 2013-08-14
From Anna:
From the top of a hill, the angles of depression of two successive milestones on a level road, which leads straight away from the hill, are 5degrees and 15degrees respectively. Fine the height of the hill.
Suggestion: BE is drawn perpendicular to AD. Find BE, then BD, finally CD.

Thanks :)

Answered by Penny Nom.
A triangle construction 2013-07-27
From Nazrul:
Two angles and the difference of the lengths of their opposite sides of a triangle are given. How can I draw the triangle. Please help me.
Answered by Chris Fisher.
What is the smallest number? (i.e. the closest number to zero) 2013-07-22
From Charlie:
What is the smallest number? (i.e. the closest number to zero)
Answered by Harley Weston.
Simultaneous equations 2013-07-10
From Warren:
solve this simultaneous equation:
xy=4
2x+3y=14

Answered by Penny Nom.
An arithmetic progression 2013-07-06
From ashok:
the 4th and 10th term a.p respectively 7 and 19 find its 15th term.....
Answered by Penny Nom.
A 3 acre plot 2013-06-17
From Chad:
I have 3 acres 100ft at the top and bottom what is the length of each side to make up the 3 acres
Answered by Penny Nom.
{(1+x)^1/3-1/3X(1+x)^-2/3}/(1+x)^2/3 2013-06-17
From STEPHEN:
{(1+x)^1/3-1/3X(1+x)^-2/3}/(1+x)^2/3
Answered by Penny Nom.
An isosceles tiangle 2013-06-16
From Izzy:
what's the height of an isosceles triangle which has a base of 50 m, and both of the other sides are 25 m?
Answered by Penny Nom.
Simson line 2013-05-23
From Nazrul:
From any point p lying on the circumcircle of the triangle perpendiculars PD , PE are drawn on BC,CA respectively. The line segment ED intersects at the point 0 ; prove that PO ⊥ AB ( PO is perpendicular to AB ).
Answered by Chris Fisher.
A 2-digit number 2013-05-13
From A teacher:
Find a “2-digit number” where the sum of the 1st digit (on the Left) and the square of the 2nd digit equal the same number.
Answered by Lorraine Dame, Harley Weston.
siin (A) and sin (A/2) 2013-05-09
From shanaia:
given that sin A=4/5 and A is obtuse.find sin (A/2)
Answered by Penny Nom.
A silver figurine 2013-05-03
From Renee:
I have a silver figurine, the density of silver is 10.5g/cm3. if the figurine weighed 149 g. to determine the volume, he dropped it into a cylindrical glas of water. the diameter of the glass is 6 cm and the figurine was pure silver, by how much should the water level in the glass rise?
Answered by Penny Nom.
How can we draw the equation "x+y=1" in 3d? 2013-05-03
From shimaa:
how can we draw the equation "x+y=1" in 3d
Answered by Robert Dawson.
How do i reverse this formula? 2013-04-20
From MK:
I have 22,000,000 worth of gold and wants to want to resell the gold back with a profit. Each transaction of gold will have a transaction fee of 15%, so if i resell the gold back at 22,000,000, I will only get back 18,700,000 after 15% transaction cut. The current formula is X-15%X = Y where X = 22,000,000 and Y = 18,700,000 Will u be able to come up with a new formula if we don't know the value of X but we know the value of Y
Something like this X-15%X = 22,000,000. How do i reverse this formula to find the value of X
Thanks

Answered by Penny Nom.
4 linear equations with 3 unknowns 2013-04-12
From Marian:
how to solve for 3 unknowns in 4 simultaneous equations
Answered by Penny Nom.
The distance between 2 lines in 4D 2013-04-01
From samane:
how can i measure the distance between 2 lines in 4D??
Answered by Chris Fisher.
We can't write sinx and cosx as a finite polynomial. 2013-03-31
From rimoshika:
prove that we can't write sinx and cosx as a finite polynomial.
Answered by Walter Whiteley.
Simultaneous equations with fractions 2013-03-31
From Terence:
5/x-6/y=1 17/x+30/y=16 I been spending whole day to solve this question. Would be very grateful if you can help I try The denominator value is a equations term which make is simultaneous equations so hard.
Answered by Penny Nom.
The dimensions of a rectangle 2013-03-19
From stephanie:
the length of a rectangle is 5m more than twice its width, and the area of the rectangle is 88m^2 find the demensions of the rectangle
Answered by Penny Nom.
Six-digit passwords 2013-03-19
From Judy:
Hello:
Can you please explain why the answer to the following question is 10^6? What is the number of possible six-digit passwords when using the digits 0 through 9, with repetition allowed.
Thank you.

Answered by Penny Nom.
An algebraic exercise 2013-03-18
From autumn:
given: f(x)=x/(x^2+2) determine: [f(x-h)-f(x)]/h
Answered by Penny Nom.
A messy arithmetic expression 2013-02-24
From niiaryee:
(((((5^2*4^3)1/5+9^3/3)+(25+(4*5)/15)^2-10)/10/2)3)))))
Answered by Penny Nom.
Fractions over fractions 2013-02-22
From Kenssa:
Question from Kenssa, a parent:

Simplify: 3-(7/x)-(6/x^2) / (3/x^2)+(5/x)-2

Answered by Harley Weston.
The continuity of f(x,y)=ln(x^2+y^2) 2013-02-17
From anu:
the question says we have to find the points in the plane where the function is continuous: f(x,y)=ln(x^2+y^2) . here we aren't given a particular point (x,y) where we have to check a function's continuity. what is to be done if we have to check continuity over the whole domain of the function? please help .
Answered by Harley Weston.
A word problem involving toys 2013-02-14
From sandy:
Each boy gets 5 toys.Each girl gets 3 toys.There are 150 pupils.
The boys had 74 more toys than girls.
How many boys?
How many girls?

Answered by Penny Nom.
Simultaneous equations 2013-02-10
From Michael:
2P + 1/3V =8
3P - 2/V=5

Answered by Penny Nom.
An arithmetic progression 2013-02-06
From loberto:
the 3rd term of an a.p is 10more than the 1st term,while the 5th term is 15more than the 2nd term,find the sum of the 8th and 15th terms of the a.p if the 7th term is 7times the 1st term
Answered by Penny Nom.
A relation on a collection of sets 2013-02-04
From raji:
suppose R be a non empty relation on a collection of sets defined by ARB if and only if A intersection B is nullset.then.....the correct answer is R is symmetric and not transitive...my doubt is how it is symmetric
Answered by Penny Nom.
The fourth side of an irregular polygon 2013-02-01
From Emran:
I have a irregular polygon. I know 3 of the 4 sides, and 2 of the angles. A-B is 285, B-C is 149, and C-D is 310. Angle B is 135 degrees. and Angle C is 45 degrees. Is there a formula to solve for the final side? Thanks.
Answered by Penny Nom.
The weight of a pipe full of water 2013-01-16
From joe:
how do you figure out the weight of a pipe full of water
Answered by Harley Weston.
A balance puzzle 2013-01-14
From Emily:
I'm stuck on this problem and feel that it can't work! The problem is: there is a door with a balance scale on it. In order to open the door, you have to use the 5 weights to balance the scale. On the left side you have 2 boxes to place the weights and on the right side you have 3. You're 5 weights are as follows: 1lbs, 2lbs, 3lbs, 4lbs, 5lbs. How do you place the weights so that the scale is balanced?
Answered by Penny Nom.
The side lengths of a triangle 2013-01-07
From Katie:
How do I find the side lengths of a triangle knowing the base and its height?

Thank you,

Answered by Penny Nom.
A triangular island 2012-12-29
From Udit:
A long time ago Mr Gibson found an island shaped as a triangle with three straight shores of length 3 km,4 km and 5 km. He declared an 'exclusion zone' around his island and forbade anyone to come within 1 km of his shore. What was the area of his exclusion zone?
Answered by Penny Nom.
A 12 acre parcel of land 2012-12-28
From John:
What are the dimensions of a 12-acre parcel if it is 3 square acres deep and 4 square acres wide?
Answered by Penny Nom.
Two polynomials are divisible by x - 2 2012-12-26
From Tehmas:
Determine the values of m and n so that the polynomials 2x^3+mx^2+nx-3 and x^3-3mx^2+2nx+4 are both divisible by x-2.
Answered by Penny Nom.
The last 4 digits of a cell phone number 2012-12-22
From Jacob:
If the last 4 digits of a cell phone number, XXX-XXX-ABCD, are restricted where: A cannot be 0 or 1 and B cannot be 0 / how many possible 4 digit combinations are left?
Answered by Penny Nom.
A project on reclaiming water 2012-12-11
From shannon:
I'm doing a report to reclaim water off of our campus facility to store in a cistern to use to flush toilets.

In Southeast Wisconsin and average of 82 inches of rain and snow fall annually. I want to collect that off of the roof of our school building. The roof is 37128 square feet. how many gallons annually could I collect?

Answered by Penny Nom.
The angles of elevation and depression 2012-12-03
From Chelsey:
a person on a balcony of one building looks towards a second building. if the angle of elevation to the top of the second building is 25 degrees, the angle of depression to the bottom of the second building is 17 degrees, and the balcony of the first building is 22 feet above the ground, what is the height of the second building?
Answered by Penny Nom.
Transposition error 2012-12-03
From Carol:
What is the correct term for when a child writes 61 instead of 16, or 83 for 38--when they switch the place values of a number?
Answered by Chris Fisher.
A function problem 2012-11-18
From nahla:
f: IN --> IN
n --> f(n)
for every n that belongs the IN : fof(x) = 4n - 3
and for every n that belongs to IN f(2^n) = 2^(n+1) -1

Calculate f(993)

Answered by Penny Nom.
The square of any odd number, decreased by 1, is divisible by 8 2012-11-16
From bailey:
Prove that the square of any odd number, decreased by 1, is divisible by 8
Answered by Penny Nom.
A mathematical expression with the answer 7 2012-11-10
From emily:
hey um i need to find and problem that fallows bedmas that has one division one multiplication and one sub and one add and one brackets and one exponents that has the answer of number 7
Answered by Penny Nom.
Forming an algebraic expression 2012-10-27
From ashley:
A sidewalk has a constant width W and comprises several short sections having lengths 12, 14,and 10 feet. How do I find the simplified expression that gives the number of square feet of sidewalk?
Answered by Penny Nom.
Some 6 digit numbers 2012-10-23
From Mason:
How many different 6 digit numbers can you make using the digits 1 ,2 5, 6, 7, and 9? How many of these six-digit numbers are divisible by 6?
Answered by Penny Nom.
A 2 acre plot 2012-10-22
From bob:
If a plot of land on the south side is 290.16 feet wide, how many feet on the west and east side to the north to make a 2 acre plot?
Thank You Bob

Answered by Penny Nom.
Converting a fraction to a decimal 2012-10-20
From A Teacher:
My students want an easy way to convert a fraction to a decimal, can I get one from you?
Answered by Penny Nom.
A proof by contraposition 2012-10-19
From Rahul:
I am not able to understand the following,
To prove that if for all e>0, |x|0, then |x|>=e. I understand the approach very well but I do not understand why if |x|=e then |x|>=e. If it is so then why not |x|= Thanks in advance!'
Rahul.

Answered by Penny Nom.
Composition of functions and one to one 2012-10-17
From Ariana:
If f o g are one to one function,does it follow that g is one to one? Give reasons for your answers
Answered by Penny Nom.
A word problem involving a fraction 2012-10-12
From Derrick:
If the numerator and denominator of a fraction are both decreased by 1 the fraction becomes 2/3. If the numerator and denominator are both increased by 1 the fraction will be 3/4. Find the original fraction. How to do?
Answered by Penny Nom.
Find an algebraic expression 2012-10-03
From Karen:

Question from Karen, a parent:

Find an algebraic expression relating the input and output.

input output
   
3 1
9 3
15 5
21 7
27 9

Answered by Penny Nom.
A problem involving simple interest 2012-09-27
From Tash:
The amount of money in a single year of an investment after P dollars were initially invested is A = P + Prt, where r is the rate of simple interest. What expression describes P? How much money was initially invested if the account has $1000 one year after the initial investment and the interest rate was 5%?
Answered by Penny Nom.
Changing the sign of an inequality 2012-09-19
From Bryauna:
Why do you change the signs in inequalities?!
Answered by Penny Nom.
The dimensions of a rectangle 2012-09-12
From Becky:
A rectangle has an area of 48 square inches and perimeter of 32 inches. Find the dimensions of the rectangle.
Answered by Penny Nom.
Six digit numbers 2012-09-11
From Fiona:
using the numerals 0, 2, 3, 5, 7, 9, make as many six-digit numbers as you can. Rearrange them into ascending order
Answered by Robert Dawson.
Minutes and seconds 2012-08-29
From Casey:
I have to write a variable equation. The questions says there are 60 seconds. but we need to write and equation to solve for minutes. Is it 1/60 or 1/s
Answered by Robert Dawson.
Vector division 2012-08-28
From Nazrul:
If A and B be two vectors where B = 3 A The What is B/A ?
Please help me.

Answered by Robert Dawson.
An arithmetic progression 2012-08-22
From A student:
the 3rd term of an A.PPP is 10 more than the first term while the 5th term is 15 more than the second.find the sum of the 8th and 15th terms if the 7th term is 7 times the first term.
Answered by Penny Nom.
0.687 acres in square feet. 2012-07-18
From Roy:
The land parcel I am looking at says Acreage .687 how do I determine square feet?
Answered by Robert Dawson.
A sand box with 14 sides 2012-07-10
From Tom:
Want to build a sand box for a pool that is fourteen foot round wanted to make it in fourteen pieces of wood what angle would each piece be and how long would each piece be
Answered by Harley Weston.
Six people divided into three groups of two 2012-07-09
From Fatima:
Six people call them A,B,C,D,E,F are randomly divided into three groups of two,find the probability of the below event(do not impose unwanted ordering among groups) E andF are in the same group I solved it but I have a doubt that it is wrong . My answer is 576 Please help to solve this problem.
Answered by Lorraine Dame and Penny Nom.
An octagonal sandbox around a pool 2012-06-12
From Linda:
My pool is 15 feet across and it is round.

How do I measure to cut wood to build a sandbox around it? was thinking it will look like an octagon.

Thanks

Answered by Harley Weston.
The position of an aircraft relative to the airport 2012-06-08
From Dennis:
"Air traffic controllers usually describe the position of an aircraft relative to the airport by altitude, horizontal distance, and bearing.  Suppose an aircraft is at altitude 500m, distance 15km, and bearing 35 degrees east of north.  What are the x,y, and z components (in meters) of the position vector. The x-axis is east, the y-axis is north and the z-axis is vertical.
Answered by Penny Nom.
Long division 2012-06-07
From Emily:
Simplify the expression using long division (9x^2-41x-6)/(x-4)
Answered by Penny Nom.
Division in base eleven 2012-05-27
From Zoe:
How do you divide numbers that are in base 11?

For example;

9A7A6A divided by A

Answered by Penny Nom.
cos(theta/30) = 1 2012-05-14
From Hope:
cos (theta / 30 = 1
I am very confused as to how to solve it. Can you help?

Answered by Penny Nom.
The dimensions of a rectangle 2012-04-30
From Nikki:
Name the dimensions of a rectangle that has the same area as this triangle? I found the area of the triangle, which is 30. What do I have to do in order to complete this math problem?
Answered by Penny Nom.
Third side of a irregular triangle. 2012-04-27
From Darya.:
How to find third side of irregular triangle if c = 17m, b = 28m and area S = 210m^2.
Answered by Penny Nom.
How to find the base length of a isosceles triangle if only the sides are given? 2012-04-25
From aqilah:
how to find the base length of a isosceles triangle if only the sides are given?
Answered by Chris Fisher.
A rectangular prism 2012-04-05
From Tori:
How do you find the base height if the base width is 10ft. The height is 7ft. And the volume is105ft.
Answered by Penny Nom.
A uniform probability density function 2012-03-26
From Noeline:
The label on a bottle of liquid detergent shows contents to be 12 ounces per bottle. The production operation fills the bottle uniformly according to the following probability density function:
f(x) = 8 for 11.975 ≤ x ≤12.10
and
f(x) = 0 elsewhere

a. What is the probability that a bottle will be filled with 12.02 or more ounces?
b. What is the probability that a bottle will be filled between 12 and 12.05 ounces?
c. Quality control accepts production that is within .002 ounces of number of ounces shown on the container label. What is the probability that a bottle of this liquid detergent will fail to meet the quality control standard?

Answered by Penny Nom.
Forming a triangle from 3 line segments 2012-03-15
From rustom:
A point X is selected at random from a line segment AB with midpoint 0. Find the probability that the line segments AX, XB, and A0 can form a triangle.
Answered by Penny Nom.
The dimensions of a rectangle 2012-03-13
From irvyn:
if you are only given the area and the perimeter how do you find the dimension?
Answered by Penny Nom.
The dimensions of a conical tent 2012-03-04
From yash:
a conical tent is to accommodate 11 people.Each person must have 4m square of space on the ground and 20m cube at air to breathe.Find the height and radius of the conical tent.26202
Answered by Penny Nom.
Integral 1/(25-x^2)^3/2 2012-02-22
From John:
Integral 1/(25-x^2)^3/2
Answered by Harley Weston.
The 10000th day of her life 2012-02-21
From derrick:
Ella was born early in the morning on a Friday.She got married on the 10000th day of her life. What day of the week was it?
Answered by Penny Nom.
Simplify the expression 2012-02-12
From Christy:
Hi guys, ok so I'm having problems with solving this equation since it's been so long I've done precal.
I have to find critical points so I have to find the derivative first. This is what I've done so far s(t)= (t-6)^4 * (t+1)^2
t'= 4(t-6)^3*(1)*(t+1)^2+ 2(t+1)(1)*(t-6)^4
so this is where I got stuck, which is the algebra part. How can I simplify this to get the answer to figure out the critical points.

Answered by Penny Nom.
Litres in a box 2012-02-09
From vicky:
how much litre water can be filled in a box of 12 inches *24 inches *12inches
Answered by Penny Nom.
A tree growth modelled by a geometric series 2012-02-08
From Steph:
Hi I am really struggling with this question please help !!!!
a pohutukawa tree is 86 centimetres when it is planted. in the first year after it is planted , the tree grows 42 centimetres in height.Each year the tree grows in height by 95% of the growth of the previous year. assume that the growth in height of the pohutukawa tree can be modelled by a geometric sequence.
A)find the height of the tree 5 years after it is planted and figure out the maximum height the pohutukawa tree is expected to reach in centimetres

Answered by Penny Nom.
The curvature of the earth 2012-02-08
From sean:
Question from sean, a student:

Two people 1.8 metres tall walk directly away from each other until they can no longer see each other (due to the curvature of the earth, which has a radius of about 6378 km).
A) Find a function relating the height of two identical objects with the distance between them using the scenario above as an example.
B) Sketch this function (you may use Graphmatica if you wish). Over what domain and range does the function exist?
C) Describe this relation in practical terms.

Answered by Harley Weston.
The dimensions of a rectangle 2012-01-31
From Dazz:
Hi I was wondering how to find the length of a rectangle if you know the area is 484 square inches And the length is 10 square inches or 22 square inches?
Thanks~Dazz

Answered by Penny Nom.
1 + 3 + 3^2 ...+3^(n-1) = 3^n - 1/2 2012-01-27
From Vicki:
I am trying to find out how to do show how this proof was worked.
Here is the end result 1 + 3 + 3^2 ...+3^(n-1) = 3^n - 1/2

This equation was used to find the number of white triangles in the Sierpinski Triangle

Answered by Walter Whiteley.
Tangent of theta 2012-01-17
From stahl:
explain what the 'tangent of theta' means. Draw and label a diagram to help with your explanation.
Answered by Harley Weston.
One acre of dirt 6 inches deep 2012-01-11
From Cameron:
Ok what im wanting to know is how many pounds of dirt are in 80 acres of land that is 6 inches deep? My friend and I are stumped on this question
Answered by Penny Nom.
The dimensions of a rectangle 2012-01-06
From joseph:
The perimeter of a rectangle is 72 inches. The length of the rectangle is three more than 2 times the width of the rectangle. What is the length of the rectangle? Write and solve an equation to determine the dimension of the rectangle.
Answered by Penny Nom.
Limits and composition 2011-12-30
From Rahul:
I want to know about limit proofs of composite functions. Like limit of log of a function equals log of limit of the function
Answered by Penny Nom.
Three sides of a triangle 2011-12-24
From saba:
the three sides of a triangular lot have lengths 10,11and 13cm,respectively. find the measure of its largest angle and the area of the lot?
Answered by Penny Nom.
The weight of 36 inch steel pipe 2011-12-22
From Keith:
What is the weight of 36 inch steel pipe 3/4 wall per foot in lbs.
Answered by Penny Nom.
i+i^2+i^3+i^4...+i^102 2011-12-13
From megan:
how do you solve i+i^2+i^3+i^4...+i^102
Answered by Robert Dawson.
Simple interest 2011-12-11
From sandeep:
Use the formula I = Prt to solve.

Damon deposits $500 into a savings account that pays simple interest at a rate of 0.65% per year. How long will it take Damon to earn $130 in interest?

Answered by Penny Nom.
Squares and triangles 2011-12-06
From Liaqath:
You have squares and triangles.
Altogether there are 33 sides.
How many squares do you have?
How many triangles do you have?

Answered by Penny Nom.
A suspension bridge 2011-11-30
From jennifer:
suspension bridges like the golden gate bridge, are used to span large distances. when the main curved cables are attached to the deck by vertical cables they will end up in the shape of parabola. assume that we need to build a bridge that spans 2,400 feet. the two towers 165feet tall each where placed at 400feet from either side. the lowest point of the center of the bridge at 10feet. vertical suspension cables where placed at 25foot interval. how many feet of cable are needed to connect the deck to the main cables between the two towers? show all working.
Answered by Penny Nom.
The 3rd term of an A.P is 10 more than the first term 2011-11-29
From Olaniyan:
the 3rd term of an A.P is 10 more than the first term while the fifth term is 15 more than the second term. Find the sum of the 8th and 15th terms of the A.P if the if the 7th term is 7 times the first term.
Answered by Penny Nom.
Converting from base 4 to base 10 2011-11-29
From anjg:
How to convert base 4 number 2^11 to base 10 number?
Answered by Penny Nom.
An arithmetic expression 2011-11-28
From Muhmmad:
2/5*-3/7-1/6*3/2+1/14*2/5
Answered by Penny Nom.
The LCM of 6,15 and x is 90 2011-10-30
From Richie:
Hi there,
There are 2 parts to the question.
#1. Factorise 90. This is easy, 90=2x3x3x5

#2
LCM of 6,15 and x is 90. What are possible values of x if x is odd?

From #1, since 90=6x15, how can this be used to work out possible values of x?

Thanks in advance, Richie

Answered by Penny Nom.
A mythical soccer ball 2011-10-27
From Joel:
We've been working on this problem diligently and can't seem to come up with the answer book's answer. We think it may be wrong, yet want to check it with an expert. Here goes. The school's new soccer balls are covered with 64 regular hexagonal panels. Each hexagon measures 2 inches between opposite corners and 1.5 inches between opposite sides. What is the total surface area of the soccer ball?
Answered by Robert Dawson and Lorraine Dame.
1 divided by 0 and infinity 2011-10-24
From ritika:
we say that one divided by zero gives us infinity, then why zero multiplied by infinity does not gives us one?????????????
Answered by Robert Dawson.
A 6% commission 2011-10-15
From Tamara:
if you earned a gross wage of $810, which includes a salary of $90 and a 6% commission how much is the net sales?
Answered by Penny Nom.
The sides of a regular octagon of area 800 sq. ft. 2011-09-21
From Susan:
Help please. I am building an octagon tree house to live in. I would like the total square footage to equal 800 to 1,000 sq ft. How long should each side of the octagon be.
tx, susan

Answered by Penny Nom.
A four digit number 2011-09-16
From gina:
i am having a hard time figuring this out.. i believe the answer is 3105 but the 0 is throwing me off... i am a 4 digit number between 3100 and 3200, none of my digits are even, the sum of my digits is 9, the largest digit is in the ones place, what number am i ??
Answered by Penny Nom.
Composition of functions 2011-09-05
From Jenna:
Let f(x)=x^2-1 and g(x)=1-2x. Find the indicated values.
1. f(g(1)) and g(f(1))

Thanks,
Jenna

Answered by Penny Nom.
The dimension of the box in centimetres 2011-08-27
From Eliza:
Find the dimension of the box in centimetres. Then find its volume in cubic centimeters. The box is 0.7m (l) 0.4 (h) 0.2 (w)
Answered by Penny Nom.
58 choose 4 2011-08-26
From Jean:
Using only the numbers 1-59, how many times would the number 23 appear in combinations of 5 numbers?
... Can a list be provided as well?
Thank you very much,
Jean

Answered by Penny Nom.
A trapezoid 2011-08-02
From Nazrul:
How can I draw a trapezoid whose parallel sides and other two sides are given.
Answered by Chris Fisher.
The dimensions of a rectangle 2011-05-08
From yolanda:
If the length of a rectangle is 3ft less than twice its width and the area of the rectangle is 54ft^2,what are the dimensions of the rectangle?
Answered by Penny Nom.
The linear thermal expansion of steel 2011-05-03
From Scot:
Several questions on your site deal with the linear thermal expansion of steel. Such as how much will a piece of steel grow in length if it is heated. My question is similar but I would like to know if there is a different calculation to determine how much the diameter of a round bar will grow when heated. Can you tell me how I can calculate how the diameter of .500" round steel will increase for every degree of temperature change? If a bar is raised from 60 degrees F to 120 degrees F how much will the diameter change?
Answered by Robert Dawson.
An exclusion zone around a triangle II 2011-05-03
From Aishwarya:
A long time ago Mr gibson found an island shaped as a triangle with three straight shores of length 3km, 4km and 5km. He declared an exclusion zone around his island and forbade anyone to come within 1km of his shore. What was the area of his exclusion zone? This question was answered but did not understand the explanation.
Answered by Robert Dawson.
(27^r^-1)^-2/3 2011-04-24
From Yung-Ju:
(27^r^-1)^-2/3
Answered by Penny Nom.
Decreasing numbers 2011-04-13
From Hadi:
A number is called a "decreasing number" if each digit in the number is less than the digit to its left. For example, 87420 is a deceasing number. How many five-digit decreasing numbers are there?
Answered by Claude Tardif.
Eliminate y 2011-04-07
From Lynn:
2x + y = 8
y + 3z =5
z + 2w =1
5w + 3x = 9

Form three equations with y eliminated

Answered by Penny Nom.
The rate of change of the area of a parallelogram 2011-04-05
From Gevork:
Let a parallelogram have sides of 8 and 12 and let vertex angle A be decreasing at a rate of 2degrees per minute. Find the rate of change of the area of the parallelogram when angle A equals 30 degrees.
Answered by Penny Nom.
Converting millimeters to centimeters 2011-04-04
From Pat:
I need to convert 67 mm to centimeters how do I do this?
Answered by Penny Nom.
3% commission 2011-04-01
From bertha:
A real estate broker’s base annual salary is $18,000. She earns 3% commission on total sales. How much must she sell in real estate value during the year to earn $65,000? Set up an equation and solve. Show all work to receive full credit. Round final answer to the nearest dollar.
Answered by Penny Nom.
The rectangle is reduced to 90% of its original size 2011-03-31
From stephon:
A rectangle has a length of 4cm and a width of 8cm.

If this rectangle is reduced to 90% of its original size what will be the new length

Answered by Penny Nom.
[(90+36-4) ÷ 2] x 15 = 2011-03-30
From ken:
[(90+36-4) ÷ 2] x 15 =
Answered by Penny Nom.
sin x = -0.25 2011-03-29
From Wayne:
How do you solve for x in the equation sin x = -0.25

the answer is 3.394 and 6.030 but I don't know the steps they used to calculate this

Answered by Penny Nom.
Write 3 1/5 in decimal form 2011-03-24
From Stephenie:
converting fractions????

3 1/5 into decimal???

Answered by Penny Nom.
6(M-1/9)=55/12 2011-03-15
From Colleen:
6(M-1/9)=55/12
Answered by Penny Nom.
The mass of a metal box 2011-03-11
From Pamela:
A metal block has the dimensions of 3.16 inches wide, 2.15 inches high, 4.98 inches long. What is the volume of the block in cm^3? What is the block's mass if the metal has a density of 5.89g/cm^3.
Answered by Robert Dawson.
Vertices and sides of a polygon 2011-03-04
From RINA:
My questions is : would a vertices and sides of a polygon be the same number.  For example, a octagon it has 8 points and 8 sides. Am i correct?
Answered by Robert Dawson.
Find the dimension of the original rectangle 2011-02-28
From sandra:
the perimeter of a rectangle is 40 cm. if the length were doubled and the width halved, the perimeter would be increased by 16cm. Find the dimension of the original rectangle
Answered by Robert Dawson.
A camera's line of sight 2011-02-26
From MJ:
A rocket that is rising vertically is being tracked by a ground level camera located 3 mi from the point of blast off when the rocket is 2 mi high its speed is 400mph At what rate is the (acute) angle between the horizontal and the camera's line of sight changing
Answered by Penny Nom.
Slicing a rectangular parallelepiped 2011-02-23
From tom:
a) Pass a plane containing a vertex of a rectangular parallelepiped and a diagonal of a face not containing that vertex to cut a pyramid from the parallelepiped. What fraction of the volume of the parallelepiped is the volume of the pyramid thus cut off?

b)Pass a plane through a cube of edge 8 in so that the section formed will be a regular hexagon. Through each side of the hexagon pass two planes, one plane containing one of the two vertices of the cube which are farthest away from the plane of the hexagon, the other plane containing the diagonally opposite vertex of the cube. Find the volume of the solid bounded by these planes.

c)three of the edges of a rectangular parallelepiped that meet in a point are also the lateral edges of a pyramid. What fraction of the parallelepiped is this pyramid...?

you know the my only problem is how to illustrate this one...if only i could ...can you help me....all I need is the drawing and I can do the rest ..it's fine if you'll only answer any of these but I'll be thankful if you will do the three :)....

Answered by Penny Nom.
Triangles with perimeter 16 cm 2011-02-22
From Chong:
How many triangles (up yo congruence) with perimeter 16 cm and whose lengths of its side are integers?
Answered by Chris Fisher.
Reversing the digits 2011-02-08
From rejie:
if the digits are reverse , the number is 9 less than the original number.what is the equation?
Answered by Penny Nom.
Calibrating a conical tank 2011-02-05
From Bill:
Hi, I have a round tank with tapered sides where I know the diameter at the top and bottom. Is there a formula I can use to calculate the volume by measuring from the bottom up the side (at the angle of the side) to any given point? Thanks, Bill
Answered by Stephen La Rocque and Penny Nom.
An even multiple of 27 2011-02-01
From parth:
the 6 digit # 63x904 is an even multiple of 27 what is X
Answered by Penny Nom.
The sides of an equilateral triangle 2011-01-27
From Cristal:
Hello :) How can i find the sides of an equilateral triangle when only given the height 10m?
Answered by Penny Nom.
Mathematics and a musical dilemma 2011-01-19
From rahul:
how is mathematics applied in entertainment?
Answered by Harley Weston.
2x = 2+2x 2011-01-15
From james:
2x=2+2x can you help me find what x epauls?
Answered by Penny Nom.
The height of a lamp 2011-01-13
From Dorothy:
I need to find the lamp height with casting shadow (base line of triangle where a boy 1.6m tall stands 3m from base of street lamp and has a 2m shadow. In other words, think of a right angle triangle with zero height starting at left, then 2m to right stands boy (1.6m high). Angle (hypotenuse) increases up to top of street lamp with 'x' height and 3m base.
Answered by Penny Nom.
The weight of a tank 2011-01-11
From tom:
A tank, open at the top is made up of thin sheet iron 1in. thick. The internal dimensions of the tank are 4 ft. 8in. in long ;3 ft. 6 in.in wide ;4ft 4 in. in deep Find the weight of the tank when empty, and find the weight when full of salt water. (Salt water weighs 64 lbs. per cu. ft., and iron is 7.2 times as heavy as salt water)
Answered by Harley Weston.
Set notation 2011-01-10
From Ron:
The question asked in a handout:

8k - 5 < 35, keI

I believe the answer to k is 5. But what does the k e(E) I mean in this?

Answered by Harley Weston.
The third side of a triangle 2010-12-28
From andrea:
in triangle abc, a-b=5 and the area is 50cm2.If angle c- 90degrees, find the length of c.
Answered by Penny Nom.
The thermal expansion of steel 2010-12-20
From roger:
Knowing that the coefficient of thermal expansion of steel is 6.5E-06 in/in/deg F. How do you calculate the loads applied as a result of the expansion?
Answered by Robert Dawson.
A geometric progression 2010-12-15
From Abeth:
find the value of x so that 2(x-1), x+3, x will be a geometric progression.
Answered by Penny Nom.
The value after 30 years 2010-12-15
From Abeth:
At the end of every year, the owners of a building which costs 2, 500, 000 pesos deducts 15% from its carrying value as estimated at the beginning of the year. find the estimated value at the end of 30 years.
Answered by Penny Nom.
A rectangular solid 2010-12-08
From ryan:
Good day! Have a question on rectangular solids. We were asked to find the dimensions of a rectangular solid, we were only given the volume of the rectangular solid, the total area, and the altitude. I tried using the volume of the rectangular solid and was able to get the area of the base, but I don't know how to get its length and its width. Hope you could help me again. Thanks!
Answered by Penny Nom.
Sales as a function of advertising 2010-12-08
From Adori:
The sales S(in thousands if units) of a product after x hundred dollars is spent on advertising is given by S=10(1-e^kx). Find S as a function of x if 2500 units are sold when $500 is spent on advertising.
Answered by Penny Nom.
Simultaneous equations 2010-12-05
From ryan:

Question from ryan, a student:

3         4
--   -    --      =     1      (1)
x         y

7       2             11
--   -  --        =   --       (2)
x       y              12


Answered by Chris Fisher and Stephen La Rocque.
Simplify the logarithmic expression 2010-12-02
From Adori:
Use the properties of logarithms to simplify the logarithmic expression. Log(1/250) base 5
Answered by Penny Nom.
How many gallons of water will it take to fill a classroom? 2010-12-01
From deena:
How many gallons of water will it take to fill a classroom that is 26 feet long by 19 feet wide and 9 feet high? How many tons of water will that be?
Answered by Stephen La Rocque.
Sides and diagonals of a polygon 2010-11-25
From nicky:
The sum of sides of two polygons is 12 and the sum of their diagonals is 19. What kind of polygon are they?
Answered by Penny Nom.
A remainder of 3 2010-11-24
From khodr:
find the smallest number (>3) that , when divided by 6 , 8 , 15 , and 18 , gives a remainder of 3 .
Answered by Robert Dawson.
Determine if a point is inside a cube 2010-11-24
From Ali:
hi every one i have a problem with cube they told me to write a program that determine if a point is inside a cube so i need the equation of the cube to do the comparison. in sphere it is easy sqrt((x-xp)+(y-yp)+(z-zp))<=radios if u please could help me
Answered by Robert Dawson.
A 20" octagon 2010-11-19
From CHARLES:
I have a 20" octagon and it has 8 parts. In inches how big is each part. I was told each part was 5.86".The top of the octagon breaks into 3 equal parts don't equal 20".When i did my math i thought it should be 6 5/8" which is closer to 20"
I am trying to cut a octagon out of a 20"circle

Answered by Stephen La Rocque.
Simple interest 2010-11-17
From Abeth:
At what simple interest rate will a sum of money triple itself in 5 years time?
Answered by Penny Nom.
A six digit number containing no zeros 2010-11-15
From John:
Find a Six digit number containing no zeros in which the first digit is three less than the third, the third digit is three less than the fifth,and the second digit equals the sum of the last three digits.
Answered by Penny Nom.
How far must the pitcher travel to get to the ball? 2010-11-04
From ken:
A baseball player bunts a ball down the first base line. It rolls 35ft at an angle of 26 degrees with the first base path. The pitchers mound is 60.5 ft from the plate. How far must he travel to get to the ball.
Answered by Penny Nom.
The sides of a trapezoid 2010-11-03
From Bobbi:
Isosceles Trapezoid base=60”; two 45⁰angles at base, two 135⁰ angles at the top, vertical distance between base & top=16”. What is the length of the top line and the two equal side lines?
Answered by Penny Nom.
The weight of some dirt 2010-11-03
From Maggie:
I am wanting to know how many tons of dirt we r going to need to fill a basement. The measurements we r going with r 15ftx15ftx8ft.
Answered by Penny Nom.
I have a sign size regulation I have to meet. 2010-10-26
From Scott:
I have a sign size regulation I have to meet. The size is .6 meters squared. at first look this would be 2 ft x 2 ft. I believe it would be larger then that. Can you confirm what the size would be in inches please.
Answered by Penny Nom.
a^(2^n) 2010-10-22
From Tim:
I am trying to understand a^(2^n). The hint they give is a^(2^(n+1)) = (a^(2^n))^2 I am writing a program that will solve a^(2^n) recursively but need to understand the power before I begin. I am currently pursuing writing (a) x (a^(2^(n-1))) where the (a^(2^(n-1))) would be the recursive function call a n approaches 0. Once n is 0, the result would be multiplied by a two more times. Anyway, explaining these powers would be appreciated. I will most likely complete the program before the answer but I want to understand the logic of these powers. Thank you, Tim
Answered by Stephen La Rocque.
Powers 2010-10-20
From dylan:
how do you write 20736 in exponential form .same for 1728 and 50625.

is there a formula to figure out how to express large know numbers in exponential form.

Answered by Penny Nom.
An exponential equation 2010-10-19
From Sandi:

Question from sandi, a student:

I have several questions similar to this one and was wondering if you could walk me through this one. I'm totally lost on how to do it.
Paramecia reproduce by splitting in two. In a laboratory flask, a colony of paramecia had an initial population of 500. Each day, the population of the paramecia was counted. The results are as listed.
Time (in days)------Population
0-----------------------500
1-----------------------600
2-----------------------720
3-----------------------864
4----------------------1037
5---------------------1244
6---------------------1493
7----------------------1792
8---------------------2150
1.)Using graphing calculator make a scatter plot of the data in table.
I think I did this part right I set my window at Xmin=0 Xmax=10 Xscl=1 Ymin=0 Ymax=2500 Yscl=100 Xres=1
2.) Determine an exponential equation to represent the population as a function of time without using a graphing calculator.I have no clue how to do this.
3.)Suppose the flask and food supply is large enough to support the trend of the population growth. Estimate the population of the colony when the time is 10 days.


Answered by Penny Nom.
Simplifying fractions 2010-10-13
From Alice:
14times (9divied3) + 12
______________________
18 divied (26-24)

don't get this.......

Answered by Penny Nom.
Tiles on a bedroom floor 2010-10-07
From Rochelle:
John's bedroom floor is square in shape. He used 625 tiles, with a side length of 200 mm, to tile the whole floor. Calculate the area and dimensions of the bedroom
Answered by Penny Nom.
A division sentence for an array 2010-10-05
From Laura:
On a math test Grace needs to look at an array and write a division sentence for the array. Grace writes 18/3=6 on her answer sheet. Describe what the array on Graces test looks like.
Answered by Penny Nom.
4√3 + 2√1/3 2010-09-25
From Michelle:
How do you solve 4√3 + 2√1/3? Please show step by step.
Answered by Penny Nom.
Six nines 2010-09-16
From Steph:
It's sort of one question. We have to use six nines to get the numbers 1-30. I got all but 22, 23, 24, and 30. Like 15 is 9+9-(9+9+9)/9.
Answered by Penny Nom.
A building and a flag pole 2010-09-09
From paul:
A flag pole and a building stand on the same horizontal level. From the point p at the bottom of the building,the angle of elevation of the top t of the flag pole is 65 degrees. From the top q of the building the angle of elevation of the point t is 25 degrees.If the building is20 meters high. Calculate the distance pt
Answered by Penny Nom.
Disposing of some concrete 2010-08-12
From Kathy:
We are needing to dispose of concrete that we are tearing out of our backyard and I need to figure the dimensions and weight. I figured we have approx. 419 sq ft of concrete at 4" thick. Could you tell me the how much that comes to in cubic ft, cubic yards and weight. So I can get some estimates on pricing to haul off and dispose of. thank you
Answered by Harley Weston and Tyler Wood.
A shrunken car 2010-07-31
From Diana:
Car manufacturers have outdone themselves. They have discovered how to shrink a car to make parking easier. At the touch of a button, a car shrinks by 2 feet in the width and 4 feet in the length. The original car base is 24 square feet longer than that of the shrunken car. If the original cars length is twice the width, find the dimensions of both cars. Identify the unknowns carefully.
Answered by Penny Nom.
70 cubic meters of mulch 2010-07-30
From zsuzsi:
i have 70 cubic meters of mulch to move and an 8 tonne truck to use how much tonnage is to 1 square meter so that i can work out how many trips i need to make
Answered by Penny Nom.
The suspension cables of a bridge 2010-07-29
From Mike:
what is the formula for the suspension cables of a bridge. The towers are 200 ft above the roadway The towers are 3400 ft apart The cable if at 8ft in the middle of the span
Answered by Robert Dawson.
The tonnage of some gravel 2010-07-22
From Gregory:
i have a road that is 8 miles long 20 feet wide and 4 inches deep what is the tonnage for gravel i would need
Answered by Penny Nom.
Simplify (2x)3 2010-07-16
From emily:
how to Simplify (2x)3
Answered by Melanie Tyrer and Penny Nom.
Converting kilograms to cubic metres 2010-07-16
From Ilai:
I want to convert kg to cubic metre
Answered by Penny Nom.
The area of a 4 sided region 2010-07-14
From Cliff:
If I wanted to know the square footage of an area that has four different sides and only had the length of one side and the angles at the ends of that length; could the other lengths be found if I wanted to have a total of 130,680 square feet?
Answered by Robert Dawson and Tyler Wood.
Thermal expansion of a steel beam 2010-06-25
From chris:
If a 50 ft steel beam can expand up to 4inches when heated to 1000f How much will a 162 ft and six inches steel beam expand under the same conditions?
Answered by Robert Dawson.
(x^3 + 11x) is divisible by 6 2010-06-24
From PT:
Given that x is a non-zero integer, how do you show that for all values of x, (x3 + 11x) is divisible by 6?

I know it works but how do I answer the "all values of x" part?

Thanks in advance!

Answered by Robert Dawson.
Expand and simplify 2010-06-15
From Alaa:
Expand and simplify
(x-9)(x+2)

Answered by Penny Nom.
11 trillion divided by 309,418,000 2010-06-05
From Dion:
I am struggling to calculate what 11 trillion divided by 309,418,000 is.
Answered by Penny Nom.
The capilano suspension bridge 2010-06-03
From nida:
the capilano suspension bridge in north vancouver is the world's highest footbridge of its kind. the bridge is 140m long . from the ends of the bridge the angles of depression of a point on the river under the bridge are 41 degrees and 48 degrees. how high is the bridge above the river to the nearest metre
Answered by Penny Nom.
Math in the workplace: nursing 2010-05-24
From Taylor:
How is math used in the workplace - Nurse Anesthetist?
Answered by Penny Nom.
Divisibility by 3 2010-05-23
From Cathleen:
To math central. I have to do a maths extension question that I don't understand. At first I thought I did. It is about the dividing by three. In one part of the question, it asks me to show that the rule of division by three does not work for 23142 with a little 5 down the bottom. What doe base 5 mean? We first thought that the little 5 down the bottom meant multiplying y the power of five. Can you please tell me what it means so I can finish this question?
Answered by Penny Nom.
converting y=mx+b into ax+by+c=0 and vice versa 2010-05-22
From Paulo:
converting y=mx+b into ax+by+c=0 and vice versa
Answered by Tyler Wood.
(x+1)(x+2)(x+3)/(x+1)(x+2) 2010-05-19
From Nazrul:
Simplify : (x+1)(x+2)(x+3)/(x+1)(x+2)
Which answer is correct:
(i) x+3
(ii) (x+2)^2(x+3)
Please help me.

Answered by Harley Weston.
A 22 litre bag of topsoil 2010-05-06
From Blanche:
I do a fair amount of gardening when the weather permits. Just wondered if anyone call tell me what a 22 litre bag of topsoil would weigh and how to convert that. My husband says 20 lbs. I say noway I think it's more. Thanks to whomever can help me. :)
Answered by Penny Nom.
Line of sight 2010-05-06
From David:
I live in St. Joseph, Michigan and there is an ongoing argument regarding line-of-sight over the horizon.

Standing on a 200 foot high bluff here, people swear they can see the top of the Willis (nee Sears) Tower in Chicago, which is about 1653 feet high.

It is my contention that this is actually a "refracted reflection" and not direct-line-of-sight.

So, to settle the argument, I'd sure like some simple explanation for this, even if-and I hope not-I am incorrect.

David

Answered by Harley Weston.
Interest 2010-05-04
From RW:
If you earned 55.00 for a year in your savings acct. The account pays 0.5 % interest. How much money did you initially have in the savings account.
Answered by Penny Nom.
The rejection region 2010-05-01
From Mong:
In Tests of Hypothesis, what is the decision when the test statistic we compute from the sample information is exactly equal to the critical value? that is, we reject null hypothesis or do not reject it? Thanks!
Answered by Chris Fisher.
A geometric progression 2010-04-30
From Kalyani:
sum of infinite geometric progression is 9 and common ratio is 1/10 then sum up to 8 terms is?
Answered by Chris Fisher.
Commission 2010-04-24
From TAHIR:
I want to pre-determine the amount of commission on sale of my product which will be the amount that I shall have to pay .
For example, the price is $ 600and commission 5% .
1) I calculate commission on 600 @ 5% = $ 30
When added $ 600 the retail price = $ 630
2) But on $ 630, 5% commission = $ 31.50
So we make a loss of $ 1.50 .
3) Again if we add this $ 1.50 to $ 630 the price = 631.50
But the commission when calculated does not tally
and we make a loss of $ 0.075.
Please let me know how to determine the commission beforehand so that it matches our accounts.

Answered by Penny Nom.
4 + 10 + · · · + (6n − 2) 2010-04-21
From Lan:
Find the sum 4 + 10 + · · · + (6n − 2). The answer is 3(n^2) + n. How?
Answered by Penny Nom.
Four less than three times a number is 14 2010-04-16
From David:
My daughter has two questions that we could use some help with.

First ?
Four less than three times a number is 14

Secound?
The quotient of -15 and w

Answered by Penny Nom.
Polar coordinates 2010-04-14
From Lan:
Given the rectangular equation (x^2)+2x+(y^2)+y=0, find the polar equation.
Answered by Harley Weston.
Counting by sevens 2010-04-13
From Larry:
Lee counted by 7's beginning with one of the whole numbers from 1 through 7, until Lee passed 1000. If Lee counted three of the following numbers, which number did Lee NOT count?

a. 107
b. 184
c. 534
d. 641

Answered by Robert Dawson and Penny Nom.
6 items are taken 3 at a time 2010-04-12
From Kristen:
Explain the three different scenarios that could occur and would result in the following number of outcomes when 6 items are taken 3 at a time. Give real-life examples.

a. 63 = 6 x 6 x 6 = 216 possibilities

b. 6! / (6 - 3)! = 6 x 5 x 4 = 120 possibilities

c. 6! / (6 - 3)! 3! = 20 possibilities

Answered by Tyler Wood.
A cube with twice the volume 2010-04-11
From Woojin:
From a given cube create another cube with twice the volume. the cube: 5cm each.
Answered by Tyler Wood.
Sample size 2010-03-29
From Rae:
What sample size was needed to obtain an error range of 2% if the following statement was made? "75% of the workers support the proposed benefit package. These results are considered accurate to within + or - 2%, 18 out of 20 times. This seems like a straight forward question but I'm getting it wrong. Could you please help me out even just the set up would be appreciated so I can see if that's where I'm going wrong. Thanks
Answered by Harley Weston.
A radio tower 2010-03-26
From Alex:
The height of a radio tower is 450 feet, and the ground on one side of the tower slopes upward at an angle of 10 degrees. How long should a guy wire be if it is to connect to the top of the tower and be secured at a point on the sloped side 110 feet from the base of the tower?
Answered by Harley Weston.
What is 8.6597 divided by two thirds? 2010-03-22
From patsy:
what is 8.6597 divided by two thirds
Answered by Penny Nom.
A quadrilateral with 4 known sides and 1 known angle 2010-03-19
From samuel:
Name: Samuel
Status: Student

I have a quadrilateral with 4 known sides and 1 known angle, and I'm trying to evaluate the other angles of my quadrilateral.

By the law of cosines, I can easily find my opposite angle (using the diagonal as a basis for the equation).

However, to find the two remaining angles, I have found no other way so far than to use the other diagonal, which can be found with the equation attached (from geometry atlas).

Is there any simpler way?

Answered by Robert Dawson and Harley Weston.
How many zeros? 2010-03-18
From jeff:
1 light yr is 6,500 billion miles, the galaxy "Andromeda" is 2,500,000 million light yrs, how many zeros are in this equation ?
Answered by Robert Dawson.
Repeating Decimal 2010-03-14
From Gerald:
Find the 1987th digit in the decimal equivalent to 1785/9999 starting from decimal point. Can you give us a short but powerful technique in solving this problem? thanks so much..
Answered by Chris Fisher.
A 14 side well house cover 2010-03-12
From Kenneth:
I am 35 yr I am wanting to build a well house cover. I'm trying to figure out how long the pieces need to be and what angle they need to be for a 4 ft dia with 14 side well house. I would love an answer but would also like to know how to figure it in the future. Thanks Kd
Answered by Harley Weston.
The nth derivative of x^(n-1) log x 2010-03-10
From shambodeb:
This is a successive differentiation problem by Leibnitz theorem

If y = xn-1 log x ; Proof nth derivative y(n) = (n-1)!/x

Answered by Harley Weston.
The side length of a square 2010-03-08
From tara:
am trying to help my brother's daughter in maths, tomorrow she has maths exam 1. Area of sq. picture is 441 sq.cm what is the length of its side.
Answered by Penny Nom.
0.999 ^ (500) 2010-03-07
From debra:
I just need to know how to solve the following problem without using a calculator: .999 ^ (500). I know the answer is .606, I just want to do it by hand since I can't use a calculator on my test.
Answered by Penny Nom and Claude Tardif.
The dimensions of a rectangle 2010-03-03
From Kirsten:
A rectangle is twice as long as it is wide. Its area is 128 square meters. What are the dimensions of the rectangle?
Answered by Tyler Wood.
The test for some disease is 99% accurate 2010-02-24
From baaba:
Assume that the test for some disease is 99% accurate. If somebody tests positive for that disease, is there a 99% chance that they have the disease?
Answered by Chris Fisher.
Fahrenheit and celcius 2010-02-19
From Carol:
Zack is packing for a trip to Scotland. He read that the average temperature in the Summer is 18degrees C during the day, and 7 degrees during the night. What is the difference between the high and low temperatures in degrees Fahrenheit? (F=9/5C=32)
Answered by Lorraine Dame.
Square poles 2010-02-16
From Steve:
If 1 pole=16.5', and you have an area that is 146 sq. poles, how many sqaure feet would you have? I calculate that to be 146(16.5)X146(16.5)=5,803,281 sq. feet Is that correct? I am told I need to divide by 146, which makes the answer 39,748. I do not understand why I need to divide by 146....
Answered by Penny Nom.
4-digit positive numbers 2010-02-15
From marjon:
Find the sum of all the 4-digit positive numbers with no zero digit.
Answered by Penny Nom.
The dimensions of a flag 2010-02-15
From dawn:
a state flag was twice as long as it was wide and had an area of 288 sq inches, what were its, dimensions
Thanks

Answered by Penny Nom.
The difference of the two numbers 2010-02-15
From Steve:
The difference of the two numbers 'abcdef ' and ' fdebca ' is divisible by 271. prove that b = d and c = e.
Answered by Claude Tardif.
Can a line segment curve over two planes? 2010-02-14
From Graham:
I am working on a math fair project. Can a line segment curve over two planes? Such as, if I had a three dimensional L bracket and I drew a line segment on it with a marker starting on the bottom of the L and had it curve around the corner and up the top, would it still be considered one line segment? Or is that two line segments? Is there a rule that a line segment can only occupy one plane? Thank you. Graham
Answered by Chris Fisher.
A sequence 2010-02-07
From Shalev:
hi,

i had a job interview yesterday. it had mathematical questions. couldn't solve 1. i m obsessed with it. please find out what the answer is..

it goes like this: 2,4,5,5,8,6,_,_,14,8

thank u very much

Answered by Claude Tardif.
I want to convert my glaze percents into grams 2010-02-05
From linda:
I am a potter. I want to convert my glaze percents into grams. ex. 20% of 20 lbs would equal how many grams? is there a formula to figure this?
Answered by Penny Nom.
An impossible isosceles triangle 2010-01-31
From Hailey:
An isosceles triangle has one angle that measures 50 degrees and another that measures 70 degrees. Why can't this triangle be drawn?
Answered by Penny Nom.
Solving a triangle 2010-01-25
From Paige:
how do i solve a triangle with one angle of 73 degrees, one angle of 32 degrees, and one side of 23cm?
Answered by Harley Weston.
A sequence 2010-01-22
From brahmaji:
0 7 26 63 _ 215 342 fill the blank? find the missing number
Answered by Robert Dawson.
The volume of a silo 2010-01-21
From heather:
The height of the silo is 30ft and the face that rests against a barn is 10 ft wide. If the barn if 5 ft from the center of the silo what is the capacity of the silo?
Answered by Penny Nom.
Area in square meters 2010-01-20
From Carla:
A football field is 120 yd long and 50 yd wide. What is the area of the football field, in square meters?
Answered by Penny Nom.
How fast am I walking? 2010-01-10
From KEVIN:
IF I WALK 3 MILES IN 28 MINUTES, HOW FAST AM I WALKING?
Answered by Penny Nom.
The intervals where the function is positive and negative 2010-01-10
From Ron:
Hello

I'm trying to find out the intervals where the function is positive and negative. It's for a polynomial function y= -(x+2)^2 (x-2) and y= (x+1)(x+4)(x-3)

I have tried the right and left side of each x-intercepts, but I still don't understand the results

thank you for your help

Answered by Penny Nom.
A pair of simultaneous equations 2010-01-09
From Yumiko:
Solve the following pair of simultaneous equations.

x^2 -4x = y^2-4
3y=2x - 3

Answered by Penny Nom.
Linear programming using the Simplex Method 2009-12-28
From William:
A gold processor has two sources of gold ore, source A and source B. In order to keep his plant running, at least three tons of ore must be processed each day. Ore from source A costs $20 per ton to process, and ore from source B costs $10 per ton to process. Costs must be kept to less than $80 per day. Moreover, Federal Regulations require that the amount of ore from source B cannot exceed twice the amount of ore from source A. If ore from source A yields 2 oz. of gold per ton, and ore from source B yields 3 oz. of gold per ton, how many tons of ore from both sources must be processed each day to maximize the amount of gold extracted subject to the above constraints? I need a linear programming solution or algorithm of the simplex method solution. Not a graphical solution. Thanks.
Answered by Janice Cotcher.
Stefan-Boltzmann Law 2009-12-28
From florence:
Hi- I have a college tutor and we are stumped with this question. Assuming a filament in a 100 W light bulb acts like a perfect blackbody, what is the temperature of the hottest portion of the filament if it has a surface area of 6.3 X 10^-5 m^2? The Stefan-Boltzmann constant is 5.67 X 10-4 W/(m^2 X K^2.\ Thank you for your help. If you can only answer one question per day please answer this one and guide me through a solution.
Answered by Janice Cotcher.
Finding Density Given Volumetric Thermal Expansion Coefficient 2009-12-26
From florence:
Hi- Please help me to apply the formula for this problem. The coefficient of volumetric for gold is 4.20 X 10^-5 C degrees. The density of gold is 19,300 kg/m^3 at 0.9 C degrees. What is the density of gold at 1050 degrees C. Could you please explain how to get the solution of 18,500 kg/m^3 Thank you for your help Florence
Answered by Janice Cotcher.
Triangles on a base of 2.4 meters 2009-12-26
From Allan:
Please,How do I calculate the height of a triangle when I only know the width of the base line,It is 2.4 mtrs.
Thankyou very much.

Answered by Penny Nom.
Finding Specific Heat of a Substance 2009-12-26
From erin:
Hi- 2 kg metal requires 1.00 X 10^4 J of heat to raise its temperature from 20 degrees C to 40 degrees C. What is the specific heat capacity of the metal?
Answered by Janice Cotcher.
A trig question 2009-12-15
From A trig question:
Hey, my name is Candle
I'm in academic math10 and am stuck on my trig... one question I thought I had right because i used the cosine law I got wrong and can't figure out why... here's a copy of the question. (i guessed it was D... but my teacher said it's B)
Thanks
Candle

Answered by Robert Dawson.
Energy in calories 2009-12-15
From Josephine:
A soft drink manufacturer claims that a new diet soft drink is now "low Joule". The label indicates that the available energy per serving is 6300 J. What is the equivalent of this energy in calories? (1 Calorie=1000 cal)
Answered by Robert Dawson.
A telephone pole on a slope 2009-12-14
From Marissa:
A 10 meter telephone pole casts a 17 meter shadow directly down a slope when the angle of elevation of the sun is 42 degrees. Find the angle of elevation of the ground. Its a law of sines problem.
Answered by Penny Nom.
A number base that's not a positive integer 2009-12-11
From Nick:
Is it possible for a number to have a base that's not a positive integer? Base 1 gives a result that's meaningless but possible. I have no concept of a base 0, a negative base, or a fractional base, never mind any other base. I think there isn't any base that's not a positive integer, but, knowing that math keeps jumping ahead and sometimes has inventions before anyone knows how to exploit them, I think I'd better ask.

Thank you.

Nick

Answered by Robert Dawson and Claude Tardif.
How fast is the distance between the two cars decreasing? 2009-12-08
From Jenny:
Two cares are on a collision course toward point P. The paths of the two cars make a 30 degree angle with each other. The first car is 40 km from P, and traveling toward P at 16 km/hour. The second car is 50 km from P, traveling at 20 km/hour. How fast is the (straight line) distance between the two cars decreasing. (Hint: Law of Cosines)
Answered by Harley Weston.
Simplifying a quadratic expression 2009-12-07
From Sabbie:
Hi, I need help solving this equation for my physics homework,so I can solve the quadratic equations for x and y. I've derived the following equations from the information provided in the question.

2a+3b = 4 -- 1
2a^2 + 3b^2 = 62 -- 2

a=2-(3/2)b is substituted into equation 2.

so, i get
2 (2-(3/2)b)^2 + 3b^2 = 62

this is where the problem begins. i cannot work through this to get to the correct quadratic equation

5b^2-8b-36=0

I can't seem to be able to figure out where I keep going wrong. I would really appreciate it if you could give me a step-by-step breakdown of the workout.

thanks!

Answered by Penny Nom.
(9 - x^2)/(x - 3) 2009-12-04
From Sandy:
9-x^2/x-3
I need to know how to solve this.
Thanks

Answered by Penny Nom.
Milliliter conversion 2009-11-29
From Jan:
I am trying out a new bath salt recipe for one of my clients that has little children. Here then is the question, the recipe for color changing bubble bath calls for 0.15ml dry FD&C color. Since the recipe came from Australia, I have not a clue of how to change this into USA measurments
Answered by Penny Nom.
Six-letter words 2009-11-24
From christine:
How many six-letter words (not necessarily an English words) are there in which exactly three of the letters are z's?
Answered by Claude Tardif.
Solving two equations, one with a square root 2009-11-23
From kacie:
y = square root of x+3
x-4y = -7

im having trouble with this problem...i have to find where they intersect.

Answered by Harley Weston.
A simplification 2009-11-21
From Lou:
In the following problem, [(7)/(x+2)]-[(x+8)/(4-x^2)]+[(3x-2)/4-4x+x^2)] what steps do you use to convert the middle portion to become +[(x+8)/(-4+x^2)]? The answer to the problem is supposed to be (11x^2-18x+8)/(x+2)(x-2)^2
Answered by Penny Nom.
Three angles and one side of a triangle 2009-11-16
From Esther:
How do i find the sides of an acute triangle if i know the angels are 60,45,75 and i only know one side which is 10? Thanks!
Answered by Penny Nom.
Two similar pentagons 2009-11-16
From louis:
The lengths of five sides of a pentagon are 1,3,5,7,and 12 units. If the length of the longest side of a similar polygon is 18 units, find the perimeter of the larger pentagon. I think I figured it out Not sure though.
Answered by Penny Nom.
The line through D(-4, 0) and E(2, 6) 2009-11-16
From Rogerson:
The point F is on the line through D(-4, 0) and E(2, 6) so that DF=4DE. Find the coordinates of F.
Answered by Penny Nom.
A drawing of a baseball diamond 2009-11-09
From Beth:
My husband has a drawn diagram on a piece of paper of a baseball diamond. He knows that on the paper 1 1/16" (1.6875") = 90 actual feet. How do we calculate how many feet 5 7/8" on the paper would be?
Answered by Penny Nom.
3x^3-3x-3 and 3x+6 2009-11-04
From spencer:
My question is 3x^3-3x-3 devided by 3x+6
Answered by Penny Nom.
Density of dirt and gravel 2009-10-26
From ken:
if digging a load of dirt and gravel in mining how many yards of material to the ton thanks
Answered by Penny Nom.
Grams per square meter 2009-09-30
From MANISH:
how to calculate gsm given kg of 144 sheets and size in inches example 28" x 22" & 16.3 kgs = ?gsm (of 144 sheets)
Answered by Penny Nom.
Sawdust 2009-09-29
From joel:
What is the density of saw dust
Answered by Harley Weston.
Grams to percent 2009-09-20
From brandy:
how do you convert 1.5 grams to a percent
Answered by Stephen La Rocque.
A sequence 2009-09-19
From Oliver:
I am struggling to work out the missing terms in the following sequence:

1, 3, _, 7, 11, 18, _

Answered by Chris Fisher.
Repeating decimal to fraction 2009-09-18
From joan:
how can i answer it easily to convert terminating decimal to fraction? example of this is that, convert ...143143143... to fractional form
Answered by Robert Dawson.
Quotient 2009-09-14
From Jala:
What is a quotient and what are the instructions for one?
Answered by Penny Nom.
Simplify 2009-09-04
From Sarah:
How do you simplify: (x-4)^3?
Answered by Robert Dawson.
Simultaneous equations 2009-08-28
From onias:
solve 3/a - 2/b = 1/2 , 5/a + 3/b = 29/12
Answered by Robert Dawson.
How do you convert cents/mile to dollars / hour? 2009-08-26
From seanna:
how do you convert cents/mile to dollars / hour
Answered by Penny Nom.
A paper towel roll 2009-08-19
From Jeff:
I am making a spiral tube with paper that is 2" in dia. and 102" long I will be using paper that is slit 3" wide how many lineal feet of paper will I need to to cover the 102" I will be using 3 rolls of paper that will over lap the other by half to make a hard tube (paper core) in a roll of paper towels Thanks Jeff
Answered by Penny Nom.
How do i convert 6 feet into yards? 2009-08-14
From taylor:
how do i convert 6 feet into yards?????????????????
Answered by Robert Dawson.
Fitting the curve y=a*exp(b*x)+c 2009-08-12
From aika:
Could one show me the complete algorithm and formula for finding the coefficients (a,b, and c) in exponential regression model y=a*exp(b*x)+c
Answered by Robert Dawson.
Gravel in an aquarium 2009-08-09
From Bev:
Hi...I worked this part out, but want to know how many kilograms of gravel I will need...thanks Bev

Cylinder shape.

A cylindrical shape that has a radius of 82 centimeters and a depth of 7 centimeters has a volume of: Volume

= 0.14787 cubic meters
= 147870 cubic centimeters
= 0.1934 cubic yards
= 5.2219 cubic feet
= 9023.5 cubic inches
= 39.063 U.S. gallons
= 147.87 liters
* calculations accurate to 5 decimal places of precision

Answered by Harley Weston.
Eye Height Level Found from Visual Angle 2009-08-07
From Jolie:
Hello, I am trying to figure out what height from the ground a person's eyes are if they have a 12 degree visual angle to a screen that is 58cm away from their eyes? Thank you.
Answered by Janice Cotcher.
Dimensions 2009-07-29
From Deborah:
The blueprint for landscaping a yard has a scale of 1/2" to 1 foot. If the blueprint is a rectangle 18 inches by 22 inches, what are the dimensions of the yard?
Answered by Robert Dawson.
x^3+1/x^3=18*sqrt(3) 2009-07-08
From Nazrul:
If x^3+1/x^3=18*sqrt(3) then how can I evaluate the value of x+1/x? Thank you for your help.
Answered by Robert Dawson.
The sides of a triangle 2009-07-08
From Susan:
The second side of a triangle is 5ft more than the length of the first side. The third side of the triangle is 10ft less than the length of the first side. The perimeter of the triangle is 61ft. Find the lengths of each side of the triangle.
Thank you so much!

Answered by Penny Nom.
Simultaneous Equations 2009-07-06
From Mukulu:
Solve the equation simultaneously X/5=(Y+2)/2= (Z-1)/4 ……………….eqt 1 3X+4Y+2Z-25=0 ………………eqt 2
Answered by Janice Cotcher.
Coefficient thermal expansion of steel 2009-06-29
From roshni:
Coefficient thermal expansion of steel is 0.00000645/in/in/deg F if F was C(celcius) then what is the answer
Answered by Robert Dawson.
The dimensions of a cuboid 2009-06-24
From laura:
a cuboid has the surface area of 842 cm2, can you work out the dimensions?
Answered by Robert Dawson and Harley Weston.
Cubic meters of gravel in a 7 ton lorry 2009-06-21
From Jackie:
I have to transport gravel in a 7 ton lorry and I know how many cubic meters of gravel I need, but I need to know how many cubic meters of gravel a 7 ton lorry can hold.
Thanks, Jackie

Answered by Penny Nom.
Two questions from math class 2009-06-18
From Con:
Hello,

My name is Con and my son is required to answer the following questions for his maths class.

He has attempted Q1 through trial and error and has found the answer to 72453. Is this correct?

He has attempted to draw the triangles described in Q2 in a number of ways and has found that BE can not equal ED and is dependent of angle BAC. Therefore, he claims that the triangle can not be drawn/practical. Is this correct or is there a slolution?

Q1.
Digits 2, 3, 4, 5 and 7 are each used once to compose a 5-digit number abcde such that 4 divides a 3-digit number abc, 5 divides a 3-digit number bcd and 3 divides a 3-digit number cde. Find the 5-digit number abcde.

Q2.
Let ABC be a triangle with AB=AC. D is a point on AC such that BC=BD. E is a point on AB such that BE = ED = AD. Find the size of the angle EAD. Con

Answered by Chris Fisher.
Successive differences 2009-06-18
From Jonathan:
I'm trying to find the next number sequence for this equation: 1 11 35 79 149 251, my problem is that I worked it out and ended up with a single number 17. What am I doing wrong. Thank you for any help.
Answered by Robert Dawson and Penny Nom.
Thermal Expansion of Steel 2009-06-17
From Ken:
Hi there, We are rollforming steel roofsheeting in 65M lengths and the = question of linear expansion has cropped up.I would like to know what = the expansion rate of this sheet would be over a temperature rise of say = 40degree F.in mm per Meter or whatever the norm is. The sheet is 0.53mm = thick and is 700mm in width,I hope this is sufficient info to enable you = to do your calculation.Many thanks, in anticipation. Ken
Answered by Janice Cotcher.
Divisibility 2009-06-17
From Sophia:
Hello
Please help my son with the solutions to the following:

a) Determine the remainder when 2^2009 + 1 is divided by 17;
b) Prove that 30^99 + 61^100 is divisible by 31;
c) It is known that numbers p and 8p^2+1 are primes. Find p.

Again, your assistance is greatly appreciated.
Thanks
Sophia

Answered by Robert Dawson.
Vectors and the Law of Cosine 2009-06-08
From lauren:
once force of 20 pounds and one force of 15 pounds act on a body at the same point so that the resultant force is 19 pounds. Find, to the nearest degree, the angle between the two original forces
Answered by Janice Cotcher.
Initial Velocity of a Car Plunging Off a Cliff 2009-06-07
From Mimi:
A car fails to take a sharp turn on a flat road and plunges off a cliff. The car lands 65m horizontally and 43m vertically from where it left the edge of the cliff. How fast was the car travelling?
Answered by Stephen La Rocque.
The dimensions of a rectangle 2009-05-30
From Linda:
A rectangle has a perimeter of 38 feet & an area of 48 square feet. What are the dimensions of the rectangle?
Answered by Stephen La Rocque.
A sequence 2009-05-26
From Jay:
what is the equation that would give me the next 3 numbers in this progession: 0,1,5,14,20
Answered by Stephen La Rocque.
The quotient of 5/8 divided by7/4 2009-05-25
From tammy:
find the quotient of 5/8 divided by7/4
Answered by Stephen La Rocque.
The Smythe family lot 2009-05-24
From Shrima:
We must determine the size of the lot. The Smythe family lot is 235ft across the front and 212ft down the side and 254ft down the other side. The sides of the lot are parallel to each other and perpendicular to the front. The size of the lot is two dimensional measure therefore you need to find the areaenclosed by the lot line.
Question:

What is the size of the Smythe lot including the unit?

Answered by Penny Nom.
The position of the fulcrum 2009-05-23
From jim:
I think I need a formula. I need to know how far an object will be lifted. A beam is 246 inches long on one side of the fulcrum, and 41 inches on the other side, if I push down 36 inches on the long side of the beam, how much will the short side move up?
Answered by Stephen La Rocque.
A geometric progression 2009-05-16
From sweta:
find the ratio of GP if the first term is 1 and the sum of third and fifth term is 90.
Answered by Penny Nom.
Unit Conversion for Force and Mass 2009-05-08
From Ray:
Good Day! I was confused about lbm and lbf and slug. I knew that lbm is a unit for mass and lbf is a unit for weight. my problem is some books follow this relationship where lbf=(lbm times the gravitational pull) which is 32.2 ft/(s^2) while other books treat lbm=lbf. other books also treat lbm=(lbf times 32.2)... which is correct? Is 1kg=2.205 lbm or 1kg=2.205lbf? Is 1BTU=778.16 lbf-ft or 1BTU=778.16 lbm-ft? what is the relationship of slug on these two, lbm and lbf?
Answered by Janice Cotcher.
Cubic feet and cubic yards 2009-05-08
From TJ:
21 bags at 2 cubic feet per bag. How many yards would that be?
Answered by Penny Nom.
The dimensions of a toy chest 2009-05-07
From charlotta:
My son needs to find the 3 dimensions of a toy chest that has to have a volume of 24 cubic feet can you show how to get the answer
Answered by Penny Nom.
8.55 acre of land 2009-05-06
From Mary:
I have a deed with 8.55 acre of land, how much land is that .55 ?
Answered by Robert Dawson.
-5^2= -25 2009-05-06
From tracy:
show how to perform this calculation and why -5^2= -25
Answered by Robert Dawson.
How do you convert 1248 to base 6? 2009-05-02
From Tamara:
How do you convert 1248 to base 6?
Answered by Penny Nom.
Decimeters and centimeters 2009-04-30
From Jeannette:
I cannot convert because of a question that throws me off. For example, A mother African elephant avg's 250 dm in height. A newborn baby African elephant avg's 89cm in height. How many cm shorter is the baby African elephant than its mother? Which one do I convert first? in order to subtract the number from the baby?
Answered by Penny Nom.
The volume of 3-D figures 2009-04-29
From Serena:
What is the formula for finding the volume of 3-D figures
Answered by Robert Dawson.
The integral of a to power x squared 2009-04-28
From JIM:
WHEN I ATTENDED U.OF T. (TORONTO ) MANY YEARS AGO WE WERE TOLD THE FOLLOWING INTEGRAL COULD NOT BE SOLVED : a to power x squared . is this still true ?

CURIOUS , JIM

Answered by Robert Dawson.
An odd shaped lot with a curve to it 2009-04-25
From Chris:
Hi - Im trying out to figure out how to calculate the size of an odd shaped lot with a curve to it.

Chris

Answered by Harley Weston.
2+6+12+20+30+42 2009-04-24
From fredy:
what is the sigma notation for the series 2+6+12+20+30+42?
Answered by Stephen La Rocque.
Miles per hour and feet per second 2009-04-22
From mary:
A car traveling at 60 miles per hour, how many feet per second has it traveled?
Answered by Robert Dawson.
Percent increase from a negative number to a positive number 2009-04-21
From Brian:
I have no problem calculating year over year net profits but am having trouble when it comes to a negative.
2007 Net Profit = -110,333.00
2008 Net Profit = 66,054.00

Brian

Answered by Harley Weston.
Two regular hexagons 2009-04-21
From niko:
Two regular hexagons are ____________ similar.
Answered by Stephen La Rocque.
Two similar rectangles 2009-04-19
From Alyssa:
The ratio of the lengths of corresponding sides of two similar rectangles is 3:5. the small rectangle has an area of 36 square centimeters. What is the are of the large rectangle?
Answered by Penny Nom.
A four-sided lot 2009-04-12
From Robert:
back of property is 137' across,with an angle of 122deg.on the left side and an angle of 140 deg. on the right side,the left side is 123' and the right side is 93' long,the left front is 130 deg. and the right front side is 150 deg. and the front of the property measures 78'
Answered by Harley Weston.
A fraction in its simplest form 2009-04-02
From Michael:
I'm in 4th grade and need to express decimals as a fraction in its simplest form. Is there a step by step method to figure out?

ex 0.64 = 64/100 = ?

Answered by Robert Dawson.
Thirty-two divided by 3 2009-04-02
From Shelly:
x = 32/3

x = 10 2/3 Divide 32 by 3: 32 / 3 = 10 R2.

What I don't get is why is it that 10 with a remainder of 2 ( 10 R2 ) equals 10 2/3? shouldn't it be 10 2/10 ?????

Answered by Robert Dawson.
A spherical Tootsie Roll Pop 2009-04-01
From Tracy:
A spherical Tootsie Roll Pop you are sucking on is giving up volume at a steady rate of .8 ml/min. How fast will the radius be decreasing when the Tootsie Roll Pop is 20 mm across?
Answered by Harley Weston.
Four identical lots 2009-03-30
From Marina:
I really want to know the answer to this problem for my 6th grade son. I've already sent this question with the drawing but I couldn't send it correctly. I hope this one will pass through. Q: Divide evenly and identical the figure representing a lot, into four for the 4 siblings. I've sent a figure drawing as attachment. I will describe this in case it will not reach you: its a square divide into 4 triangle and 1 triangle is taken out living a letter M figure and this letter M figure is the one that will be divided.into 4 even and identical parts.

Marina

Answered by Claude Tardif.
Adding in base six 2009-03-29
From Lucy:
If you are working in base 6, what do you have to add to 4(base 6) to get 10(base 6)????
Answered by Penny Nom.
Subdividing a four-sided lot 2009-03-27
From kannan:
i have a 1.5 acre land having four sides
A-B=64.8 meters
B-C=94.2 meters
C-D=54.4 meters
D-A=127.2 meters
angle at D is 90
the side AB (frontage)is road facing i want to split the plot in to two halves having same almost area and almost equal frontage, kindly help me, if possible explain with drawing.

Regards
Kannan

Answered by Harley Weston.
Dividing by fractions 2009-03-25
From Mitch:
-2 divided by -2/3 = ??
Answered by Penny Nom.
Comparing positive and negative fractions 2009-03-24
From Christyrose:
Hello. I am a sixth grade math student and i need help on how to compare negative and positive fractions.

Here is is:
3/4 -2/3 7/10 -5/6

I'm not sure how to do the negative parts because i was absent for a couple days. Thank you!

Christyrose

Answered by Penny Nom.
Conversion factor 2009-03-23
From kab:
Would like to photocopy a Landscape Design. What is the Conversion Ratio or magnification required to convert 1" = 10 feet to 1/4" = 1 foot?
Answered by Harley Weston.
A rectangular open box 2009-03-21
From tina:
An open box is to be made from a rectangular piece of tin by cutting two inches squares out of the corners and folding up the sides. The volume of the box will be 100 cubic inches. Find the dimensions of the rectangular piece of tin.
Answered by Stephen La Rocque.
A 20 foot piece of pipe 2009-03-19
From chris:
How many 8 inch pieces can you get from a 20 ft. piece of pipe?
Answered by Penny Nom.
0.0007 as a percent 2009-03-18
From Shelly:
I need to know how to divide 0007 by 100 and soon! and I have no idea how I can get 100 to go into a number like that! here's the question this problem originated from:

change the decimal into a percent:
0.0007 _________

How do I change THAT number into a percent?

How do I change 12.455 and 92.348 and 29.005 into percents as well? I know how to change numbers like 0.6 and 0.70 into percents but not the numbers I told you I'm having problems with. Please someone explain this to me and also please focus mainly on the first number I asked about but don't forget the others please. Please reply ASAP I need to know this in about a week or less!

Answered by Penny Nom.
The sides of a parallelogram 2009-03-17
From Sami:
If ABCD is a parallelogram, prove that line AB is congruent to line CD. Clearly state your reasons and conjectures.
Answered by Penny Nom.
An impossible trig problem 2009-03-14
From Alisha:
If csc theta = 3 and sec theta = square root of 3, what are the values of tan theta and cos^2 theta?
Answered by Harley Weston.
Related rates 2009-03-14
From Jeevitha:
The side of an equilateral triangle decreases at the rate of 2 cm/s. At what rate is the area decreasing when the area is 100cm^2?
Answered by Stephen La Rocque.
A polyhedron 2009-03-10
From Mollie:
Hi, my name is Mollie and I'm in 5th grade. Here's the math problem I have for homework tonight:

"Larissa made a model of a polyhedron using 8 pieces of clay for the vertices and 18 straws for the edges. What type of polyhedron did Larissa make?" Thanks.

Answered by Penny Nom.
Related rates 2009-03-09
From Megan:
A plane flying with a constant speed of 330 km/h passes over a ground radar station at an altitude of 3 km and climbs at an angle of 30°. At what rate is the distance from the plane to the radar station increasing a minute later?
Answered by Harley Weston.
What percent commission does Aaron earn? 2009-03-09
From Jerry:
Aaron works part time as a salesperson for an electronics store. He earns $4.75 per hour plus a percent commission on all of his sales. Last week Aaron worked 20 hours and earned a gross income of $143.75. If his total sales for the week were $1,500, what percent commission does Aaron earn?
Answered by Penny Nom.
The sides of a triangle 2009-03-07
From felicia:
ona map, tannersville, chadwick, and barkersvile for a triangle. Chadwick is 70 miles from Tannersville and Barkersville is 90 miles from Tannersville. Which is a possible distance between Chadwick and Barkersville?
a. 5miles
b. 10 miles
c. 150miles
d. 200 miles

Answered by Harley Weston.
The amount of material remaining on a reel 2009-03-04
From James:
Question from james:

I am after a standard calculation so that after each usage of the Reel I can get the Remaining Quantity in kilos left on the roll

below is an example of roll

THICKNESS: 30 MICRON
REEL WIDTH: 7.5 CM
REEL DIAMETER: 76 CM
REEL WEIGHT: 7.13 KILOS

The core centre has a 9cm diameter, the weight of the roll is excluding the core , and after each use the diameter of the role is measured.

I am a factory supervisor working on project and need this calculation info for template.. Thanks James

Answered by Harley Weston.
A shape with 10 sides 2009-03-03
From Sandra:
One of my son's math questions is as follows: Suppose I have a shape with 10 sides. I choose a vertex then draw lines to the other vertices that dont share sides with the first vertex. How many vertices will that be and why?
Answered by Penny Nom.
The area of a 5-sided lot 2009-03-02
From Linda:
I have a problem I'm hoping you can provide some help with. When I check the property information for our home on our District of North Vancouver website, it shows that our property is supposed to be 1146.99 Square Metres. Yet another property which has identical measurements is listed as having 1173.7 Square Metres. I would like to figure out who is correct.
Answered by Harley Weston.
An array with 2 more rows than columns 2009-02-28
From Mario:
Write a division sentence modeled by an array that has 2 more rows than the number in each row.
Answered by Penny Nom.
Simplify the square root of 125 x^2 2009-02-27
From Ashley:
√125x²

How do you simplify this equation assuming that variables can represent any real number?

Answered by Robert Dawson.
Triangles within triangles 2009-02-24
From Mari:
a large shaded triangle changes each day with a white triangle appearing in the center of each shaded one. If this pattern continues, how many white triangles will be there on the 6th day? On the 6th day, what fraction of the large outer triangle will be white?THANKS!
Answered by Robert Dawson.
Augmented matrix: independent, dependent, or inconsistent? 2009-02-23
From Anna:
Perform row operations on the augmented matrix as far as necessary to determine whether the system is independent, dependent, or inconsistent.

x + y + z = 11
x - y + 3z = 5
2x + 2y + 2z = 15

Answered by Harley Weston.
Pressure at Given Depth 2009-02-22
From Tamara:
A sea is 3km deep. The average density of sea water there is 1020 kg/m^3. What is the water pressure at the bottom?
Answered by Janice Cotcher.
Find the resultant of this displacement pair 2009-02-22
From katydidit:
Find the resultant of this displacement pair:
500 miles at 75 degrees east of north and 1500 miles at 20 degrees west of south.
How do I graph this and how do I solve this problem?

Answered by Penny Nom.
What deminsions should my box be? 2009-02-19
From Mary:
My item is 57cm x 36cm x 6cm and I want to put 20 items is 1 box. What deminsions should my box be?
Answered by Stephen La Rocque.
Angle of depression 2009-02-18
From Meeka:
An aircraft flying at an altitude of 2000m is approaching an airport. If the angle of depression of the airport is 5 degrees, what is the distance from the plane to the airport measured along the ground? Round your answer to the nearest tenth of a kilometer.
Answered by Robert Dawson.
Miles per hour to kilometers per second 2009-02-17
From Tamara:
If a car goes 60 miles in an hour how many kilometres will it go in a second?
Answered by Penny Nom.
Rent and a commission 2009-02-17
From cliff:
a landlord receives $95.00 rent 5% commission was paid to get to $95.00 what was the original number. The answer is obvious but what is the formula. Thanks. Cliff
Answered by Penny Nom.
The sides of a polygonal building 2009-02-13
From mark:
length of sides of an 18 ft.diameter bldg

i need to find out if i should build an octagon or decagon to keep the sides of the bldg at 8ft or less. how do i figure the length of the sides of each?

Answered by Penny Nom.
Can you convert numbers with units to percents? 2009-02-12
From Kenneth:
To change a number to a percent, move the decimal two places to the right and add a percent sign. For example, 0.35875 = 35.875%.
If the number has a dollar sign or some other kind of unit, for example, pounds, feet, etc. how is the number converted to a percentage.

For example, change $0.35875 and 0.75 lbs. to a percentage. How is this done. I have never seen $35.875% or 75 lbs.% before.

Answered by Robert Dawson and Harley Weston.
Cubic feet gas to grams of gas 2009-02-11
From tim:
I was trying to convert cubic feet gas to grams of gas. Is there a formula to complete this task?
Answered by Robert Dawson.
Cubic centimeters and ounces 2009-02-06
From Dawn:
1-- syringe contains 5.5cc of medicine & cost $13.50
1 --2 oz bottle of medicine cost $17.50

How many cc's in 2 oz, & which is the better deal?

Answered by Stephen La Rocque.
A 30-60-90 triangle 2009-02-03
From Inez:
If you have a 30-60-90 triangle and the only side you get is 73 and a 90 degree box, how do you find the area?
Answered by Penny Nom.
Calculating markup 2009-02-03
From Tonya:
I'm trying to figure out the formula for the following, I have a product that I want to receive a certain profit, but with this product I also have to pay a percentage of commission to someone. I need to know how to calculate my new selling price taking into account the markup percentage, but I don't want my cut to be lowered. I have calculated the markup percentage to get my new selling price with the commission, but it lowers my cut once I pay the commission and I don't want it to do that. Thanks
Answered by Penny Nom.
Simultaneous equations with an xy term 2009-02-01
From angelee:
xy+5x-2y-10=0 2x+y=1
Answered by Penny Nom.
limit sinx/x 2009-01-30
From Jackie:
how to evaluate limit sinx/x as x tends to zero if x is in degrees
Answered by Stephen La Rocque and Harley Weston.
25,000 pounds of sheet metal 2009-01-27
From JESSICA:
I need to convert 25,000 lbs. of 32x48 sheet metal into total sq. feet. What would be my formula?
Answered by Robert Dawson.
The dimensions of a pool 2009-01-18
From denise:
a rectangular swimming pool is three times as long as it is wide. if the perimeter of the pool is 320 feet, what are its dimensions
Answered by Penny Nom.
The whole number that precedes a number w+3 2009-01-17
From Morgan:
write a variable expression for "The whole number that precedes a number w+3"
Answered by Penny Nom.
Fuel in a tank 2009-01-16
From Steve:
I am not a math expert and want help seeing how much fuel is in a tank at one given time the tank that i have is 27 inches round. It is 69 inches long. laying length ways. it holds 140 gallons of fuel at fill up. I do know that it would hold less at the bottom of the tank an more in the center. Is there anybody that could just give me the figures by the inch without a formula. thanks for your time
Answered by Stephen La Rocque.
0.0708 lbs per linear yard 2009-01-16
From Kevin:
If I know that my product is .0708lbs per linear yard how do I calculate how many Linear yards are in a lb?
Answered by Penny Nom.
An expression for q quarters and n nickles 2009-01-16
From Morgan:
a variable expression for:

"The value in cents of q quarters and n nickels"

Answered by Penny Nom.
Does probability apply here? 2009-01-15
From Paul:
Suppose I have a dice in my hand that I am about to roll. The probability that I roll a six is, all things being equal, 1/6. I accept that.

Suppose now the I roll the dice and immediately cup my hand over the result. What is the probability that I have rolled a six? People seem to want to say it is still 1/6. But it can't be can it!? It is surely either 1 or 0, depending on whether I have in fact rolled a six?

Answered by Robert Dawson.
A one cubic mile lake 2009-01-09
From dye:
how many liters are in a one cubic mile lake
Answered by Robert Dawson.
660 yards = ?? miles 2009-01-08
From Ashley:
How do I convert yards to miles, specifically, 660 yards = x miles. Please show work so I can figure other problems out.
Answered by Penny Nom.
(a x b) intersect (b x a) 2009-01-08
From sean:
is it possible to have two sets such that n((a X b) intersect (b X a) =3
Answered by Harley Weston.
Numbers or Percentage or probability or what? 2009-01-07
From Chew:
Teacher A teaches 50 students and 45 of them obtained straight As. Teacher B teaches 5 students and 5 of them obtained straight As. Which teacher would you send you child to and why?
Answered by Robert Dawson and Harley Weston.
Converting grams to milliliters 2009-01-04
From Faiza:
I was looking for how to convert grams into mL and I read on this site to convert one to the other you need to know what substance you are measuring. so if I am measuring sugar or any of the baking ingredient like flour how do I convert them from grams to mL is there a simple method I can use to do that?
Answered by Penny Nom.
The square root of 164 2008-12-31
From Melissa:
How do you simplify the square root of 164?
Answered by Stephen La Rocque.
The volume of a feed hopper 2008-12-18
From John:
I need to calculate the volume of a feed hopper, and I'm not sure how to break it down. The top of the hopper is 36" x 36", it is 30" deep, and ends at a 6" x 6" plate. One side of the hopper is straight top to bottom, of course tapering on two sides to meet at the plate. The other three sides angle down at about 75 degrees. I need to determine the cubic foot volume of this hopper (it is used for ground coffee) so I can configure a vibrator to knock down residual grounds. Thanks.
Answered by Robert Dawson.
The angle between two lines 2008-12-17
From abhi:
how to calculate the angle between two lines, given the length of the lines.. angle should vary from 0 - 360 in the counterclockwise direction
Answered by Robert Dawson and Harley Weston.
650g+2kg+195g=how many kg 2008-12-11
From christian:
650g+2kg+195g=how many kg
Answered by Penny.
Wall covering 2008-12-10
From Jeanie:
I need to convert 14 LY of wallcovering (54" wide) to square feet. Can you please help me?
Answered by Penny Nom.
The volume of a pipe 2008-12-10
From Walter:
What is the volume of a pipe 7.92 inches inside diameter and 22 km. long in cubic feet of water
Answered by Robert Dawson and Harley Weston.
A wine decanter 2008-12-10
From Suzanne:
I am trying to buy a wine decanter for my dad for Christmas. One big problem I am having shopping online is finding the right size. Wine is sold by metric ml, with a standard bottle holding 750ml of wine. Decanters are measured by oz, and the sizes are all over the place, with one site alone having sizes 25oz, 26 1/2oz, 50oz, 59 1/8oz, and 68oz.

Can you help me with metric conversions so I can figure out how much wine these decanters hold? Like everybody else, I need this as soon as possible so I can get it shipped in time for Christmas. (I'm disabled & don't drive, so online shopping is my only way to get gifts.)

Thanks for your help!

Suzanne

Answered by Chris Fisher and Robert Dawson.
What is one trillion divided by 10? 2008-12-09
From mark:
can you please tell me what one trillion divided by 10 is
Answered by Robert Dawson and Penny Nom.
The volume of a yogurt pot 2008-12-04
From Robyn:
Okay, I know you have some answers on converting this already, but I can not convert the yoghurt pot I need yet into ml. Is there some way to explain this using the yoghurt as an example? The pot's 150g of lowfat strawberry yoghurt... would the 'low-fat' have an effect on this? I'm sorry if this is a bother for you to answer, I just don't seem to be able to get it... Have a nice day. - Robyn
Answered by Janice Cotcher and Penny Nom.
Division in different bases 2008-12-02
From MICHELLE:
DIVIDE 538 BY 14 IN BASE 2, 3, 4 & 5
Answered by Penny.
3 equidistant points 2008-12-01
From Damien:
How do you find 3 equidistant points (C,D,E) on a line between point A(Xa, Ya) and point B(Xb, Yb) so that AC, CD, DE and EB are all equal?
Answered by Chris Fisher.
What is so important about quadratics? 2008-11-29
From zoe:
what is so important about quadratics?
Answered by Harley Weston.
Dividing Large Numbers 2008-11-26
From Mark:
what is 7.9 trillion divided by 301 million?
Answered by Janice Cotcher.
The path of a small sailboat 2008-11-19
From jane:
a sailor in a small sailboat encounters shifting winds. she sails 2.00 km East then 3.40 km North East, then an additional distance in an unknown direction. Her final position is 6.68 km directly east of the starting point. find the magnitude & direction of the third leg of the voyage.
Answered by Harley Weston.
An algebraic exercise with fractions and radicals 2008-11-18
From Rubén:
Question from Rubén, a teacher:

I have the ecuation:

U = [yb / (ab+a^2)]^1/2 + [ya / (ab+b^2)]^1/2

I know for sure this reduces to

U = [y (1/a + 1/b)]^1/2

but I cannot find a way to get into that result!

Thanks

Answered by Rubén Osuna.
The midpoint o a line segment 2008-11-15
From Jane:
The vertices of a triangle are at (1,7), (6,-1) and (0,3). Find the coordinates of the midpoints of the sides.
Answered by Penny Nom.
A ratio to a decimal 2008-11-14
From manny:
Express 22 : 5 as a decimal. Round to the nearest hundredth
Answered by Penny Nom.
How far are the boats apart? 2008-11-14
From dom:
Two boats leave port at the same time. They leave at 150 degree angle. One boat travels at 10mph and the other at 20mph. After two hours how far are the boats apart?
Answered by Penny Nom.
The angles and sides of a triangle 2008-11-13
From JAMIE:
a triangle with a side(b)37m an angle(C)70degrees and (a)79m find values of angles A and B and length of side c
Answered by Stephen La Rocque.
An odd shaped lot 2008-11-13
From Janice:
Hi I am attaching a survey of an odd shaped real estate lot. I am hoping you can help me determine the acreage.

Janice

Answered by Harley Weston.
How do you work out 2/17 as a decimal? 2008-11-13
From p:
how do you work out 2/17 as a decimal?
Answered by Penny Nom.
A barrel on its side 2008-11-13
From Dave:
Question from Dave:

How many gallons are left in a 36x60 in. barrel (laying on its side) and has 16 in. of gasoline left. I have attached a diagram.

Answered by Harley Weston.
Math and nursing 2008-11-10
From alina:
how many math do you net for nurse
Answered by Stephen La Rocque.
A flagpole and a yardstick 2008-11-07
From Wanda:
One boy holds a yardstick vertically at a point 40 feet from the base of the flagpole. The other boy backs away from the pole to a point where he sights the top of the pole over the top of the yardstick. If his position is 1 ft 9 in from the yardstick and his eye level is 2 ft above the ground, find the height of the flagpole.
Answered by Penny Nom.
A bay window 2008-10-30
From Scott:
Given the length of an arc, the rise of an arc, and the number of segments that I need to divide the arc into - how do I determine the length of each segment? Imagine a bay window. It could have 5 side or 7 sides, just as an example. How do I determine the width of each window given that the unit will be mounted into a frame with a 96 inch opening. The rise of the unit will extend out 18 inches. Lets say that the number of single windows unit within the unit is 5. How wide should each window be? Do you understand?
Answered by Harley Weston.
The length of a rectangle is 6 inches less than 3 times its width. 2008-10-26
From Angela:
The length of a rectangle is 6 inches less than 3 times its width. Find the dimensions of the rectangle if its area is 45 square inches.
Answered by Penny Nom.
Put in lowest terms 6a^2c/8ab 2008-10-24
From hana:
put in lowest terms 6a^2c/8ab
Answered by Stephen La Rocque.
A falling block 2008-10-18
From KOBINA:
A 50 kg block being held at rest 20m above the ground is released. the block falls (no friction). how fast is the block traveling (in m/sec) when it has lost 40% of its original potential energy.
Answered by Stephen La Rocque.
Two equations in two unknowns 2008-10-17
From Dushayne:
Please help me in solving this problem:
a. 3x-4y=32
5x+2y=10

b. 2x+3y=11
4x+3y=10

Answered by Penny Nom.
The dimensions of a rectangle 2008-10-16
From cristine:
the area of a rectangle is 675sq.cm. if the width is 1/3 of the length. find the dimensions of the rectangle....
Answered by Penny Nom.
Millimeters to inches 2008-10-15
From diane:
can you convert milliliters to inches?
Answered by Penny Nom.
The sides of an octagon 2008-10-12
From side lengthsGarry:
I need the length of the sides and the degrees of the angles for a 30 inch diameter octagon
Answered by Janice Cotcher.
The dimensions of a pool 2008-10-08
From Justine:
This may be a really silly question but I am trying to work out what the possible dimension a pool may be if we have 20m length of wall of an above ground pool (laid out flat on ground)

We have been given a second hand above ground pool and we know it was in an oval configuration and we are just trying to get a guesstimate of the potential size it may be as the people couldn't tell us the size

Answered by Penny Nom.
Algebraic fractions 2008-10-06
From Kayla:
(x^2-9)/x times (x^3-4x)/(x^2+5x+6)
Answered by Harley Weston.
A product of positive and negative numbers 2008-10-03
From kaylou:
Suppose you were given 13 numbers and asked to find their product. Seven of the numbers were positive, and the rest were negative. Would your product be positive or negative? Why?
Answered by Penny Nom.
The unusable space in a container 2008-10-02
From Billy:
an object is 42 inches long, 29 inches wide, and 69 inches high. The usable capacity of this container is 36.4 cubic feet. How much space in percentage is left over from the container?
Answered by Penny Nom.
Does an oval have sides? 2008-10-02
From reid:
My 6 yo neice came home with her math homework and she was supposed to identify which objects had sides.One of the objects was an oval.I don't believe it has sides because it is curved and I don't think that would make it an object with sides.What would be the correct answer?Thanks,Reid
Answered by Janice Cotcher.
The length of a tank 2008-10-01
From alex:
I have a tank which I can only partly see. It is 70cms wide, 45 cm deep and hold 500litres. How long is it?
Answered by Harley Weston.
Conversion to cubic feet 2008-10-01
From Dorothy:
What is the cubic feet of an item that is 7"x16 7/8"x19"
Answered by Harley Weston.
z(z+1)-x(x+1) / z-x 2008-09-30
From sylvia:
z(z+1)-x(x+1) / z-x

HOW DO I SIMPLIFY THIS

Answered by Penny Nom.
3,6,10,15,and 21 2008-09-28
From jarred:
i am currently stumped on a math project that requires me to find out the recursive formula for a sequence of numbers. the numbers in the sequence are 3,6,10,15,and 21. Thank you for your time.
Answered by Walter Whiteley.
144 square inches 2008-09-27
From mike:
how much cm is in 144 square inches?
Answered by Penny Nom.
Simplify 6(a+2b)+8a-16b 2008-09-27
From madison:
6(a+2b)+8a-16b
Answered by Penny Nom.
Three equal ratios 2008-09-26
From Jodi:
For the ratio given, find three equal ratios: 2 miles/8 minutes
Answered by Penny Nom.
Sample variance: inches to centimeters 2008-09-22
From Anita:
The average height of a sample of basketball players is 6 feet, 2 inches or 74 inches. The standard deviation of this sample of players is 4 inches. If each person's height were to be multiplied by 2.54, what would be the value of the resulting sample variance?
Answered by Harley Weston.
The sides of an octagon 2008-09-21
From mark:
I need to know the measurements of an octagon with a inside diameter of 34.5 inches from side to side
Answered by Penny Nom.
Simplify [[1/(x+3)]-1/3]/x 2008-09-19
From diana:
How does [[1/(x+3)]-1/3]/x simplify
Answered by Harley Weston.
Oils and fats 2008-09-18
From Barry:
I'm doing a nutrition assignment and I'm adding up my Oils and Fats category. I have 10ml of butter to add to 31g of fat so I'm trying to figure out what to do or I should say how to figure out the answer. Barry
Answered by Penny Nom.
The weight of a concrete cylinder 2008-09-18
From Curtis:
I am trying to calculate the weight of a concrete cylinder. The diameter is 56" and the thickness or depth is 24". Can anyone help me? If possible I would like to see the formula as to how the answer was derived. Thank you
Answered by Harley Weston.
Simplifying equations 2008-09-18
From diana:
I do not know how to simplify equations. How does 5x^2-7X+2 become (x-1) (5x-2)?
Answered by Penny Nom.
Cubic yards and quarts 2008-09-16
From Mary:
I need 2 yards of soil, how many quarts is this?
Answered by Penny Nom.
Three addends that sum to ten 2008-09-13
From Alfred:
How many probabilities are there to have 3 addends to get a sum of 10? What is the formula?
Answered by Harley Weston.
A number that divides by six but not by three 2008-09-12
From Ken:
A number that divides by six but not by three
Answered by Stephen La Rocque and Victoria West.
How would put .12 with 2 repeating into a fraction? 2008-09-09
From Savanna:
How would put .12 with 2 repeating in fraction?

Savanna!

Answered by Penny Nom.
How many ml are in 40 grams? 2008-09-09
From Nancy:
I need to know how many ml are in 40 grams. So what is the base 1ml= how may grams
Answered by Harley Weston.
An exclusion zone around a triangle 2008-09-07
From Awrongo:
A long time ago Mr Gibson found an island shaped as a triangle with three straight shores of length 3 km,4 km and 5 km. He declared an 'exclusion zone' around his island and forbade anyone to come within 1 km of his shore. What was the area of his exclusion zone?
Answered by Stephen La Rocque and Penny Nom.
A security camera 2008-09-07
From Rita:
A security camera in a neighborhhod carnival is mounted on a wall 9 feet above the floor inside a video gallery. What angle of depression should be used if the camera is to be directed to a spot 6 feet above the floor and 12 feet from the wall?
Answered by Stephen La Rocque and Harley Weston.
Simplify ((5/7)/(5/14)) + (3/4) 2008-09-06
From Emal:
How do you simplify completely: ((5/7)/(5/14)) + (3/4)?
Answered by Penny Nom.
3 miles in 6 seconds 2008-09-05
From Grant:
if a rocket sled is traveling 3 miles in 6 seconds, how fast is it going in miles per hour?
Answered by Harley Weston.
The selling price 2008-09-04
From Robert:
Client wants to sell his property providing he is left with 100000.00$ net. How do I calculate selling price if commission is 6% and taxes on the commission 6% Fed and 7.5%prov. I'm taking the real estate class and this one really got me.
Answered by Penny Nom.
I need an equation that best fits these numbers. 2008-09-03
From Vallatini,:
Attached, please find a plotted curve (pdf file). I have pulled values from this curve (see below). I need an equation that best fits these numbers. Can you help?
Answered by Harley Weston.
An algebraic expression 2008-08-29
From Antonio:
write a variable expression for 9 less than k
Answered by Penny Nom.
Long Division with Decimals 2008-08-28
From Ghita:
I need help with these 2 questions: Find the quotients: 52.576 divided by 21 71.331 divided by 93 Ralph spent 5/9 of an hour mixing paints and 7/9 of an hour painting.How much time in all did Ralph spend on his project? Thank you,Please answer soon
Answered by Janice Cotcher.
557 mL of benzaldehyde 2008-08-28
From Kara:
Calculate the mass of 557 mL of benzaldehyde. The density of benzaldehyde is 1.04 g/mL.
Answered by Penny Nom.
The square root of a fraction 2008-08-24
From Lauren:
How do you solve square roots of fractions? Does the format change if it is either a proper or an improper fraction?

Ex. the square root of 1/4 or the square roots of 80/25

Answered by Penny Nom.
Radii and Chords Create a Non-Right Triangle 2008-08-22
From Beary:
AOC is a diameter of circle O. Line AB is 12, and lines OA and OC (the radii) are 10. Find the length of line BO and chord BC.
Answered by Janice Cotcher.
Simplifying Algebraic Expressions 2008-08-22
From Jacky:
x^2-y^2+4x+4y
Answered by Penny Nom.
Converting centimetres to metres 2008-08-21
From walter:
how do you convert the smaller unit to a decimal of the larger unit and then solve 17 m - 130 cm
Answered by Penny Nom.
Angular & Linear Speed from a Sine Graph 2008-08-19
From Kim:
Kim, a student: I am given a graph with a wave. The amplitude is 5cm and the period is 4cm. I am suppose to find the angular speed. What I need to know is the formula for angular speed and how do I use these numbers to get the correct answer.
Answered by Janice Cotcher.
Metres and miles 2008-08-18
From Bill:
I enter a race that is 10,000 meters long,how many miles do i have to run?
Answered by Victoria West.
Solving for Shared Height of Two Right Triangles 2008-08-17
From Heidi:
find the height of a triangle, which can be split into two right triangles, but the base (50m) is not split equally in half. one end of the base is 40 degrees, while the other is 30 degrees.
Answered by Janice Cotcher.
Similar Triangles Given a Median 2008-08-17
From RAM:
Sides AB , AC and median AD of a triangle ABC are respectively proportional to PQ PR and median PM of another triangle PQR. Show that ABC is similar to PQR
Answered by Janice Cotcher.
Converting to cubic yards 2008-08-15
From Lisa:
I need to convert 32,000 square feet, 5 inches deep to cubic yards. Can you help me? Thank you
Answered by Harley Weston.
Similar triangles 2008-08-12
From ramarao:
D is a point on side BC of a triangle ABC such that BD/CD=AB/AC. prove that AD is the bisector AD is the bisector of BAC
Answered by Chris Fisher.
Arc-length and sector-angle 2008-08-06
From Benson:
If chord length, radius are given, How to find the sector angle and arc-length
Answered by Janice Cotcher.
24 in by 14 in by 14 in in cubic feet 2008-08-05
From Tom:
I have a box that has internal dimensions of: 24in. H x 14in. W x 14in. D. How many cubic feet is this internal space?
Answered by Penny Nom and Victoria West.
Why can't you divide by zero? 2008-08-04
From Donna:
Why can't you divide by zero?
Answered by Chris Fisher.
Lot size of a four sided lot 2008-07-30
From JAMES:
I'm trying to find out this lot size in acres. I looked at a bunch of your suggestions, but I guess I'm not smart enough to figure it out.. I have attached an image of the lot, hopefully with enough info. for you..

Thanks,
Jim

Answered by Penny Nom.
Right Sided Limit of an Exponential Function 2008-07-29
From joseph:
I am trying to find the limit as X approaches 0 from the positive side of x to the sin of x which look like this lim x^sinx x->0+
Answered by Harley Weston.
Trigonometric 2008-07-28
From kiran:
In triangle , Sin@ = ?
Answered by Penny Nom.
What is the domain of (f/g)x? 2008-07-28
From Warren:
If f(x)=3x-5 and g(x) = square root of x-5, what is the domain of (f/g)x?
Answered by Penny Nom.
Alpha level 2008-07-23
From anonymous:
What is the advantage of using an alpha level of .01 versus a level of .05, What is the disadvantage of using a smaller alpha level?
Answered by Janice Cotcher.
Simultaneous equations 2008-07-23
From Franco:
Solve

3 D + E - F = -10
-2 D - F = -4
-3 D - 4 E - F = -25

Franco

Answered by Penny Nom.
Choosing 4 numbers from 70 numbers 2008-07-21
From Jeannine:
When using numbers from 1 to 70 in sets of 4, how many combinations are there without having the same combination twice? Thanks
Answered by Penny Nom.
Three consecutive positive integers 2008-07-20
From AC:
find three consecutive positive integers such that the product of the first and third, minus the second, is 1 more than 4 times the third
Answered by Penny Nom.
A bijection from (0,1)x(0,1) to (0,1) 2008-07-20
From Adam:
I'm trying to prove that the function that takes the open square (0,1)x(0,1) to (0,1) is a bijection (and hence a continuum).

If we take an element (x,y) of (0,1)x(0,1) and represent (x,y) as (0.x1 x2 x3 x4..., 0.y1 y2 y3 y4...) aka x1 represents the tenths digit of x, x2 represents the hundredths, etc. Then we can define a function f((x,y)) = 0.x1 y1 x2 y2 x3 y3... However, this is not a bijection. I hypothesize this is because you'd be unable to create the number 0.1 as x=0.1 and would have to be y=0, which contradicts the open interval (0,1) defined for y. We have been told though, if we create the same function, except that we "group" 9's with their next digit into a "block" we can create a bijection. For example, if x=0.786923 and y=0.699213, then we define x1 to x3 as normal, but x4= 92, and x5=3. For y, we define y1 as normal, but y2=992, and y3 to y4 as normal. hence f((x,y)) = 0.7 6 8 992 6 1 92 3 3.

My questions are a) is my hypothesis on why the original function is not a bijection correct? b) why does the special blocking in the new function make a bijection?

Answered by Victoria West.
A silo with a flat side 2008-07-14
From Amy:
Without using the Pythagorean Theorem, determine the capacity of a silo in cubic feet of grain if: the cylinder-shaped silo has one flat, rectangular face that rests against the side of the barn; the height of the silo is 30 feet and the face resting against the barn is 10 feet wide; the barn is approximately 5 feet from the center of the silo.
Answered by Harley Weston.
A four sided lot 2008-07-09
From michelle:
the front is 65.29
left side 162.26
the back 155.00
right side 101.01

right angle=50.00
left angle= 153.30

Answered by Harley Weston.
Neither prime nor composite 2008-07-08
From saloni:
Give the counting number which is neither prime nor composite ?
Answered by Penny Nom.
Divisibility by 11 2008-07-04
From Peter:
For what single digit value of n is the number n53nn672 divisible by 11?
Answered by Leeanne Boehm.
Composition of two functions 2008-07-04
From Kristian:
f(x)= the square of x add to 1 and g(x)=1/x

find: (f o g)x

Answered by Penny Nom.
Polynomial division and remainders 2008-07-02
From RAM:
If the polynomial x4-6x3+16x2-25x+10 is divided by another polynomial x2-2x+k, the remainder comes out to be x+a. Find k and a.
Answered by Janet Cotcher and Harley Weston.
Kilometers per liters to miles per gallon 2008-06-26
From margaret:
we travelled 130 kilometers on 10 liters of gas. How many miles to the gallon did I get
Answered by Penny Nom.
The smallest number divisible by 1 to 9 2008-06-26
From Peggy:
What is the smallest number divisible by each of the first nine counting numbers?
Answered by Penny Nom.
A six sided lot 2008-06-25
From Zack:
Please help me find the area of my lot. I am sending a fax of the drawing and dimensions. Zack
Answered by Harley Weston.
How many pounds in 300 grams? 2008-06-24
From Renelda:
How many pounds in 300 grams? Can you send the formula? Thanks
Answered by Penny Nom.
The sum of the digits of a number 2008-06-23
From Ben:
Question: Using mathematical induction, prove that if the sum of the digits of a number is divisible by three, then the number itself is also divisible by 3.
Answered by Penny Nom.
9(d+5)=3(d-3)+6d 2008-06-19
From davontay:
9(d+5)=3(d-3)+6d
Answered by Harley Weston.
Concrete blocks 2008-06-18
From marvin:
How many cement blocks do i need to build a 8 ft. tall wall x 1,300 ft. long?
Answered by Harley Weston.
How do you change bases? 2008-06-17
From Peter:
How do you change bases eg. 121 from base 3 to base 5?
Answered by Penny Nom.
-2(x - 1)^2 + 8 2008-06-12
From kasondra:
y=-2(x-1)2[thats a square] +8 that need to be put in standard form
the answer is y= -2x2 -4x + 6
i got y= -2x2 -4x -4
i just don't get how my teacher got that answer

Answered by Penny Nom.
The weight of concrete 2008-06-11
From Daniel:
I need to calculate the weight of Concrete in a slab that measures 2' x 4' x 4" thick. I am pricing out a job and have two different size stones to work with, the second stone measures 2' x 2' x 4".
Answered by Harley Weston.
What are the dimensions of the rectangle? 2008-06-11
From Denise:
A rectangle has a diagonal that is 3.6 feet longer than the length and 7.1 feet longer than the width. What are the dimensions of the rectangle?
Answered by Claude Tardif and Victoria West.
A recursive formula for 9, -18, 36, -27, ... 2008-06-05
From Rita:
I am having trouble understanding how to write formulas (I should say create formulas) from a given sequence. It does not matter what sort of sequence it is. The confusion remains. I have not found a textbook or easy math book that explains this process for the average Joe to grasp. Here are the two questions:

(1) Write a recursive formula for the sequence
9, -18, 36, -72, ...

(2) Write a recursive formula for the sequence
3, 3(sqrt{3}), 9, 9(sqrt{3})

Answered by Penny Nom.
A 4 sided lot 2008-06-03
From Pat:
If a lot of property is: front 127.87, back 156.17, one side is 129.73, the other side is 125.00, then how much total acreage is this.
Answered by Harley Weston.
The area of a regular pentagon 2008-06-02
From Jenna:
I would like to know if there is a formula to find the area of a regular Pentagon that is not inscribed in a circle only apothem and side length are given.
Answered by Penny Nom.
4x^3 - 6x + 2 = 0 2008-05-28
From paul:
4x^3 - 6x + 2 = 0
Answered by Janice Cotcher.
Convert cubic inches to gallons 2008-05-26
From Rosemarie:
How do you convert cubic inches to gallons? Thanks for your help.
Answered by Penny Nom.
.5 cu ft of sand to lbs. 2008-05-25
From rogersimmons:
needtocovert .5 cu ft. of sand into lbs.
Answered by Penny Nom.
3 radical 48 +11 radical 75 2008-05-24
From Maria:
Add 3 radical 48 +11 radical 75. Write answer in simpliest form.
Answered by Penny Nom.
Double the the dimensions of a rectangle 2008-05-22
From Destiny:
Double the the dimensions of a rectangle that is 3 by 4 in. What is the area?
Answered by Penny Nom.
Volume/weight of a salt shedweight 2008-05-21
From James:
How do I find the volume/weight of a salt shed. I have a 3000 ton barn, I need a better way of finding how much salt to order as it gets used. I know how to calculate volume, how do i get tonnage from there??
Answered by Harley Weston.
Nine digit numbers 2008-05-21
From Alex:
List of Nine digit numbers, that can be divided by nine?
Answered by Janice Cotcher.
1+1/1+1/x 2008-05-21
From nadean:
1+1/1+1/x
Answered by Penny Nom.
A weird shaped lot 2008-05-21
From James:
Need to find square foot of a residential lot on a cul-de-sac. Weird triangular shape. Measurements : 126.18 170.08 138.2 48.4
Answered by Harley Weston.
Converting a decimal to a fraction 2008-05-19
From judy:
please explain how to get from the decimal number .3437 to the fraction 11/32
Answered by Leeanne Boehm.
Guy wires for a tower 2008-05-19
From larissa:
a radio tower 500 feet high is located on the side of a hill ( the hill has an inclination to the horizontal of 5 degrees.) How long should two guy wires be if they are connected to the top of the tower and are secured at two points 100 feet directly above ( up the hill ) and directly below the base of the tower?
Answered by Penny Nom.
Fencing around a playground 2008-05-15
From Vanessa:
If the playground is a rectangle with it's width 10ft shorter than it's length, and it can be enclosed by exactly 140ft of fence, what is the length and width of the playground?
Answered by Penny Nom.
The fourth side of a property 2008-05-14
From Penny:
Property has 3 measurements and 2 angles and I want the altitude at a particular point. Corners are sw, nw, ne and se. 1 want the 4th saide measurement confirmed (we think 146.07') and the altitude measurement at 25' from sw (on the sw-se side) The angles provided are interior at nw 67.38 degrees and the opposing corner se interior appears to be 68.58 degrees. the 3 sides with measurements are: sw to nw=54.07'. sw to se 146.07' and se to ne 53.57'. If you would help, we would really appreciate it. Thanks! Penny
Answered by Harley Weston.
How much seed do I need per plot? 2008-05-13
From Jaye:
The plot size is 5 m X 2 m. The seeding rate is 12Kg/ha. How much seed do I need per plot? I have failed to get a proper answer. Thank you.
Answered by Penny Nom.
The weight of a concrete column 2008-05-11
From russell:
a cylindrical form is filled with a slow curing concrete. The base of the form is 10 ft in radius, and height is 25 ft. while the concrete hardens, gravity causes the density to vary from a density of 90 lbs/ft^3 at the bottom to a density of 50 lb/ft^3 at the top. Assume that the density varies linearly from the top to the bottom, and compute the total weight of the resulting concrete column
Answered by Harley Weston.
Selling price 2008-05-10
From Roxane:
I'm trying to figure out the formula for the following, I have a product that I want to receive a certain profit margin, but with this product I also have to pay a percentage of commission to someone. I need to know how to calculate my new selling price taking into account the markup percentage, but I don't want my gross margin to be lowered. I have calculated the markup percentage to get my new selling price with the commission, but it lowers my profit margin once I pay the commission and I don't want it to do that.
Answered by Penny Nom.
The sides of an octagon 2008-05-07
From Frank:
Question from Frank, a parent: Hi, Need to lay out a 10 ft. diameter octagon on the ground. What is the length of the sides ? And the angles ?
Answered by Harley Weston.
Four cookies 2008-05-03
From tashera:
if a serving is 5 cookies. how many servings can i make from 4 cookies?
Answered by Stephen La Rocque.
How many gallons are in 1,500 liters? 2008-05-01
From bambi:
[How many gallons are in 1,500 liters?
Answered by Penny Nom.
Division by zero 2008-05-01
From Bill:
Why does multiplication have a favored status so as to make division by zero undefined?
Answered by Harley Weston.
1/3-5/6=1/x, solve for x 2008-05-01
From Lavada:
1/3-5/6=1/x How do I solve this problem.
Answered by Penny Nom.
(9x^2-4)/(3x^2-5x+2) * (9x^4 - 6x^3 +4x^2)/(27x^4+8x) 2008-04-29
From Jonathon:
I don't remember how to solve an equation in the form of the following:

(9x^2-4)/(3x^2-5x+2) * (9x^4 - 6x^3 +4x^2)/(27x^4+8x) =

Answered by Stephen La Rocque.
Converting metric density 2008-04-28
From Hilary:
10g/100ml=?g/?l
52g/100ml=?g/?l
65g/100ml=?g/?l
100g/100ml=?g/?l
137g/100ml=?g/?l
0.15g/100ml=?g/?l

Answered by Penny Nom.
WHY 144 where does this figure come from 2008-04-19
From Sean:
Why do you use the number 144 to divide

EG. length of timber 2 inches x 8 inches x 14 foot = 1.5 cubic feet

Calculation is 2x8x14 =224 divide by 144 = 1.5 WHY 144 where does this figure come from

Answered by Leeanne Boehm.
Simplify 5x^5/8 / 15x^2/12 2008-04-18
From Kyler:
5x^5/8 / 15x^2/12 Divide the rational expressions. Simplify the results.
Answered by Penny Nom.
The three sides of a triangle 2008-04-16
From Bridgett:
The sum of the lengths of any 2 sides of a triangle must be greater than the third side. If the triangle has one side that is 11 CM, and the second side of the triangle is 4 cm less than twice the third side, what lengths do the 2nd and 3rd side have to be?

between 0000-00-00 and 9999-99-99


Answered by Stephen La Rocque.
What is 300% as a fraction? 2008-04-14
From Carli:
what is 300% as a fraction
Answered by Stephen La Rocque.
A 13 cubic foot cylinder filled with sand 2008-04-12
From Thomas:
I have a 100 pound bag of sand. I need to know how much sand I will need to fill a 13 cubic foot cylinder
Answered by Harley Weston.
The capacity of a silo 2008-04-12
From Candonn:
I have a cement silo that is 30 ft. high and has a diameter of 12 ft. I was wondering what the capacity of the silo is in tons of dry material. The silo is flat on the bottom also.
Answered by Penny Nom.
Some 4 sided polygons 2008-04-11
From kathy:
I am a 4th grade student and need to complete this question:

I need to draw 2 polygons with 4 sides and 2 right angles. Please help. Thanks

Answered by Penny Nom.
Side lengths and angles in a regular octagon 2008-04-09
From Lori:
I am a builder and need to find the length of the sides of an octagon. I know the length between the parallel sides (26 feet). What is the length of each side? What is the angle measurement?
Answered by Harley Weston.
The weight of 1 litre of milk 2008-04-07
From IMRAN:
WHAT WEIGHT IN KILOGRAMS DOES 1 LITRE OF MILK HAVE?
Answered by Penny Nom.
Composition of functions: feet, inches and yards 2008-04-05
From Coya:
write a function f(x) that converts feet to inches.Now write a function g(x) that converts yards to feet. Explain what the composite function f(g(x)) means. Then evaluate f(g(x)) for x = 3,6, and 9.
Answered by Stephen La Rocque.
Compatible numbers 2008-04-04
From dawn:
Need help. Estimate the quotient. Tell what compatible numbers you used.

817divided by 4 =

Answered by Penny Nom.
100 yards of cloth 2008-04-04
From Leroy:
(100 yards of cloth 48 inches = how many square yards)
Answered by Penny Nom.
Ounces and grams 2008-04-02
From Rakesh:
7.2 oz per sq. yard equals ? __ grams per sq. meter

How I can measure the weight of a Jeans fabric (in oz)? And what is the rellation between grams and oz?

Answered by Penny Nom.
Common fraction to decimal fraction 2008-04-01
From chadwick:
how do I turn 3/32 into a decimal
Answered by Penny Nom.
-2 ^ 2 + (6 - 9) / (-3) + 4 * (-2) 2008-04-01
From Jeth:
How do I simplify: -2 ^ 2 + (6 - 9) / (-3) + 4 * (-2)
Answered by Penny Nom.
Square feet and square yards 2008-04-01
From Victor:
6200 sq. ft.into yards
Answered by Penny Nom.
A car tire full of concrete 2008-03-27
From robert:
I want to build a volleyball net support. I am using a car tire 24"odx16"id filled with concrete. how much will this weigh? thanks
Answered by Penny Nom.
Similar triangles 2008-03-26
From Nisha:
In triangle ABC, LM is parallel to AB. If AL=x-3,AC=2x,BM=x-2,BC=2x=3. Find x
Answered by Penny Nom.
Light years to miles 2008-03-24
From Robert:
HOW MANY MILES ARE THERE IN 7.5 BILLION LIGHT YEARS.
Answered by Penny Nom.
Converting ounces to milliliters 2008-03-21
From kelly:
As I have to drink 12 - 8 ounce glasses of water a day and my water bottle holds 591 ml, how many bottles of water must I drink?
Answered by Penny Nom.
How would I graph: x - 2 divided by x^2 - 4? 2008-03-19
From Sean:
How would I graph: x - 2 divided by x^2 - 4 using a method in extreme values of functions or completing the square

x - 2/ x^2-4

Answered by Stephen La Rocque.
Determine the capacity of Farmer John's silo 2008-03-16
From paula:
Farmer John stores grain in a large silo located at the edge of his farm. The cylinder-shaped silo has one flat, rectangular face that rests against the side of his barn. The height of the silo is 30 feet and the face resting against the barn is 10 feet wide. If the barn is approximately 5 feet from the center of the silo, determine the capacity of Farmer John’s silo in cubic feet of grain.
Answered by Penny Nom.
The limit of (sin4x)/(tan9x) as x approaches zero 2008-03-14
From stev:
as x->0. How do you find the limit of f(x)=(sin4x)/(tan9X)
Answered by Harley Weston.
The length of a side of a square 2008-03-12
From laura:
if i had a square of 2.3 square meters what would its radius be?(in centimeters please)
Answered by Penny Nom.
Moving grain across the river 2008-03-12
From Janice:
Wesley needs to move 820 kg of grain across the river in a canoe that can carry no more than 120 kg. Wesley has a mass of 70 kg. What is the fewest number of trips he will have to make? A. 17 trips; B. 18 trips; C. 32 trips; D. 33 trips.
Answered by Penny Nom.
A ton of carbon dioxide 2008-03-11
From claire:
I want to know the size of a box if it contained a ton of Co2?
Answered by Stephen La Rocque.
Repeating decimals 2008-03-10
From Blaine:
Is it possible to put a repeating decimal number into a calculator to solve a problem?

EX: Write 39.3939... as a fraction.

Answered by Penny Nom.
Side lengths and area of a triangle 2008-03-08
From angela:
In the diagram of ABC the angle A is 65 the angle B is 70 and the side opposite vertex B is 7. Find the length of the side opposite vertex A and find the area of ABC
Answered by Penny Nom.
Composition of functions 2008-03-07
From sharon:
Find FoG(x)
F(x)= the square root of x-9
G(x)= x^2
and also find GoF and their domains

Answered by Penny Nom.
Cubic feet to liters 2008-03-06
From chase:
if 1cu.ft=6.229 imp. gal and1 imp.gal=4.546 liters how many liters does of 128cuft. hold
Answered by Penny Nom.
The dimensions of a rectangle 2008-03-06
From Julie:
Find the dimensions of a rectangle with an area of 400. Width x ft. Length (2x-7)ft.
Answered by Stephen La Rocque and Penny Nom.
An arithmetic expression 2008-03-05
From Kasani:
evaluate the expression: -2^-1[-14 - 4(-6)] - (7 - 3)
Answered by Penny Nom.
How large is a container that will hold 3.5 cu meters? 2008-03-05
From Robert:
If a company allows you to ship personal belongings while on an over seas assignment and the figure is 3.5 cubic meters. Just how much is the total area in square feet?? 10X10X10 or??? I just wanted to understand the size of a container that would hold 3.5 cubic meters!!
Answered by Harley Weston.
What was the ostrich's speed in miles per hour? 2008-03-03
From jacob:
If it took an ostrich 2.5 seconds to travel 110 feet and the average speed is 44 feet/ seconds, what was the ostrich's speed in miles per hour?
Answered by Penny Nom.
Convert from base 6 to base 12 2008-03-02
From Jade:
I am trying to convert 13405 base 6 to a base 12 number. I am not allowed to go into a base 10 number. How will can I base 12 without doing anything with base 10? Thank you kindly.
Answered by Stephen La Rocque and Penny Nom.
Simultaneous equations 2008-02-29
From CONOR:
I was wondering if you could help me with this problem

7x - 5y = -1
3y = 4x

Answered by Penny Nom.
150 decimeters is how many centimeters 2008-02-29
From gayle:
150 decimeters is how many centimeters
Answered by Penny Nom.
A new carpet 2008-02-28
From Rhiannon:
The Gatlins are buying a new carpet for their house. They need about 1,175 square feet of carpet. The carpet they buy is sold by the square yard. Estimate the number of square yards of carpet they need. Do I use the formula to find the volume or are?
Answered by Penny Nom.
Actural cost, selling price and commission 2008-02-21
From LiSa:
I have a question about a few things if anyone can answer... ex. actual cost (AC) = $1000 selling price (SP) = $ amount earned for company (C) = 15% of... selling price? commission 1 (C1) = 2.5% commission 2 (C2) = 2.5% How would you calculate how much the company's commission amount would be as well as the amounts for the other 2 that get 2.5%? And what would the selling price be? (the 15% $ + 2.5% $ + 2.5% $ ?) Is the 2.5% taken from the original amount? I am soooo confused! If anyone could help, that would be great!
Answered by Harley Weston.
Algebraic fractions 2008-02-21
From sergio:
how to simplify
(x^2 +5x+6)/(x^2 - 4) x (x^2 -5x+6)/(x^2-9)

Answered by Stephen La Rocque.
8% simple interest for 6 years 2008-02-20
From Tiffany:
cindy puts 5,025 in a bank account that earns a simple interest rate of 8%. How much money will be in the account at the end of 6 years?
Answered by Stephen La Rocque and Penny Nom.
The length of the third side of a triangle 2008-02-16
From mary:
I have an angle of 72 degrees and each of the sides are 5' long. What is the distance from each of the ends of the 5 feet to form a triangle.
Answered by Stephen La Rocque.
I am a 4-digit number 2008-02-12
From Nickie:
I am a 4-digit number with no repeating digits. I am divisible by 5, my first two digits (left to right) make a number divisible by 3, and my first three digits make a number divisible by 4. Also, my digits have a sum of 19 and I have the digit 7 in the thousands place. Who am I?
Answered by Penny Nom.
Drawing blocks from a box 2008-02-11
From Inez:
A box contains 3 red blocks and 1 green block. A block is chosen without looking and not put back. Then another block is chosen. What are the possible outcomes?
Answered by Penny Nom.
A flagpole and a statue 2008-02-09
From Krista:
A flagpole casts a ten meter (m) shadow at the same time as a six metre (m) statute beside it casts a two metre shadow. What is the height of the flagpole??
Answered by Steve La Rocque and Penny Nom.
A three sided shape 2008-02-09
From Tim:
Question from Tim:

How would I calculate the area in square feet of a "triangle" with sides:

Base is 36 feet; height is 70 feet; and the thirs side is an arch (curve) of 77 feet.

Answered by Harley Weston.
Why is the number 1 neither prime nor composite? 2008-02-07
From ronni:
why is the number 1 neither prime nor composite?
Answered by Harley Weston.
Cost per square yard 2008-02-05
From Terry:
MY cost for seed is $325.00 per acre...i want to bring that down to $ per sqyard. Whats the formula?
Answered by Penny Nom.
The size of a lot 2008-02-02
From linda:
Hi, could you please tell me how close to an acre of land my lot is. The lot size is 140 ft by 200ft. I am trying to see how close to an acre it is.
Answered by Penny Nom.
The dimensions of a rectangle 2008-02-01
From Evan:
you are given that the area of a rectangle is 21.6in sq. find the base and the height of the rectangle.
Answered by Penny Nom.
The sign of the quotient of two integers 2008-02-01
From michelle:
without actually dividing, how can you decide whether the quotient of two integers is A. positive B. negative C. 0?
Answered by Penny Nom.
Bubbles trapped in the volumetric pipet 2008-01-31
From Nursing:
In determining the density of a liquid, a student left air bubbles trapped in the volumetric pipet. Did this give a density less than expected or greater than expected? Why?
Answered by Penny Nom.
The volume of a tank 2008-01-27
From Fred:
A cubic tank holds 1,000 kilograms of water what are the dimensions of the tank in meters?
Answered by Harley Weston.
Linear feet to metres 2008-01-25
From Jillian:
i have a measurement of something in linear feet. i dont know what a linear foot is so i want to convert it to meters to have an idea. how do i convert.
Answered by Penny Nom.
Converting cubic inches to ounces 2008-01-22
From kim:
I am trying to find the formula to figure capacity in ounces for the following:

a rectangle 18" long, .75" wide, .75" deep ....

This is for an art project and the cost estimate. Thanks, Kim

Answered by Penny Nom.
How many combinations of 8614 are divisible by 7? 2008-01-22
From Rebecca:
How many combinations of 8614 are divisible by 7 equally (with no remainder)?
Answered by Penny Nom.
The cosine of an angle 2008-01-21
From Kristine:
Find measure of unknown side cosA=0.5
Answered by Harley Weston.
The angles of a triangle given the three sides 2008-01-17
From Lucy:
Is there a way to find the angles of a triangle just by knowing the lengths of it's sides? It seems like the would be a relationship between the two, but I'm not sure.
Answered by Stephen La Rocque and Harley Weston.
Converting mg's to cc's 2008-01-17
From sandy:
i need to find out how to convert mg's to cc's i need to give 10 mg of liquid and 5 mg of liquid and i only have a dropper that measures cc's please help
Answered by Stephen La Rocque and Harley Weston.
Fact families with multiplication and division 2008-01-16
From Donna:
I am trying to help my granddaughter with some math homework. I looked at what you have on fact families. Am I correct to assume that the same process applies to multiplication and division as well? Her question asks for fact families for 2, 8, & 16.
Answered by Penny Nom.
Explaining the factoring for the difference of cubes 2008-01-16
From Bill:
A student asked me where did the "difference of cubes" and "sum of cubes" come from. I did not have an answer for her. She is very bright and understands how they work but wanted to know where they derived from. Any help you can offer would be great. Thanks
Answered by Stephen La Rocque.
A 16 sided lot 2008-01-16
From R:
I am having a land of 16 irregular sides i know all the length of the sides how to calculate the area of the land
Answered by Harley Weston.
Balancing on a fulcrum (net torque and equilbrium) 2008-01-15
From Eric:
If I have a 4" bar that has no weight value with a 275lb. weight on one side and a 125lb. weight on the other side what would be the folcrum point? Please keep the answer very simple. Also is there a fixed equation to figure folcrum points? Is there an equation for if the 4" bar had a weight value?
Answered by Stephen La Rocque.
Two solutions using the law of sines 2008-01-14
From Kate:
I am working on the Law of Sines and I have a problem that says: Find a value for b so that the triangle has 2 solutions.

I am given that A = 36 degrees and a = 5. Now, I learned that for a triangle to have 2 solutions, h < a < b. BUT...my answer key says the answer is: 5 < b < 5/sin 36. I can't figure out how to make this fit with h < a < b.

Answered by Harley Weston.
(303,000,000,000,303x3,300,000,033)/1,000,100,010,001 2008-01-13
From Adrian:
Give the result of the following computation as an integer.

(303,000,000,000,303x3,300,000,033)/1,000,100,010,001

N.B. No calculators allowed

Answered by Penny Nom.
Forces on an inclined plane 2008-01-10
From Ron:
A body that weighs 540lbs is caused to slide up an inclined plane with a uniform velocity by a force that acts parallel to the plane. For each foot of horizontal distance, there is a vertical rise of 2in. If the coefficient of sliding friction is 0.16, what force is required to move the body?
Answered by Stephen La Rocque.
Base salary plus commission 2008-01-10
From mike:
i need some help with this word problem. in november 2001, hall kinion advertised an account executive position in savannah georgia for a salesperson with experience selling internet services. the job advertised a 3500 dollar base salary and 24000 dollar to 48000 dollar in commissions in the first year. Assuming a 20 percent commission rate, how many dollars in sales would u have to generate to earn 64000 dollars annually.
Answered by Penny Nom.
Sqrt(x^2 + y^2) 2008-01-10
From Darcie:
Can you simplify this problem?: square root of (x squared plus y squared). Note: x squared plus y squared is under the radical.
Answered by Penny Nom.
Conversion 2008-01-08
From Dom:
Hi , Is it possible to convert 175 grams per metered squared to cm ? tks Dom
Answered by Stephen La Rocque.
How far is the jet from the lighthouse? 2008-01-07
From Natalie:
Question: A ship spots a lighthouse that is 53m high, at an angle of elevation of 7 degrees that is directly north of the ship. The same ship spots a jet travelling N62E at an altitude of 1500m with an angle of elevation of 15 degrees. How far is the jet from the lighthouse?

Natalie

Answered by Harley Weston.
Sigma notation 2008-01-06
From jamelia:
what is this word problem in sigma style i am so lost at this point my first time ever attempting this subject 1+2+3+4+5
Answered by Penny Nom.
Lining up coins visually using geometry and trigonometry 2007-12-31
From Jessica:
a) In what order would you arrange a penny, a nickel, a dime, a quarter, and a half-dollar so that they all have the same apparent size? The diameters of the coins, in thousandths of inches, are as follows: penny, 750; nickel, 835; dime, 705; quarter, 955; half-dollar, 1205.
b) How should the coins be placed, if the distance between the dime and the half-dollar is 100 units? How far from thw dime should your eye be to see that all the coins have the same apparent size?
c) What angle do the coins subtend when they have the same apparent size?

Answered by Stephen La Rocque.
Smallest cone containing a 4cm radius inscribed sphere 2007-12-19
From Eva:
A sphere with a radius of 4cm is inscribed into a cone. Find the minimum volume of the cone.
Answered by Stephen La Rocque.
Simplifying square roots 2007-12-19
From Ciara:
How would you calculate 2 to the square root of 8 plus 4 to the square root of 2 minus 5 to the square root of 2?
Answered by Stephen La Rocque.
What size room would I need for 220 cubic feet? 2007-12-18
From Andy:
What size room would I need for 220 cubic feet? What is the calculation I would use to find the answer to this problem?
Answered by Penny Nom.
System of equations 2007-12-06
From Jenn:
change the equation,x-y=4 to form y=mx+b the solution to the system of equations y=2x and y=-x+3 is
Answered by Stephen La Rocque.
Chicken and goat feet 2007-12-05
From Kim:
Old McDonald raises goats and chickens. The animals have a total of 100 heads adn 360 feet. How many goats and how many chickens does Mr. McDonald have?
Answered by Stephen La Rocque and Penny Nom.
Third side of a triangle 2007-12-05
From Sudhir:
How can we find the all positive integral values for the third side of a triangle, whose other two sides measure 15 and 20 respectively. (No other information viz. the type of triangle, area, perimeter or angles etc. is given.)
Answered by Stephen La Rocque.
A decision based on data 2007-12-04
From manny:
You have administered a standardized test of manual dexterity to two groups of 10 semi skilled workers. One of these two groups of workers will be employed by you to work in a warehouse with many fragile items. The higher the manual dexterity of a worker the less likelihood that worker will break significant inventory. Because of a unique contract you must hire all 10 employees from one of the two groups and none of the employees from the other.

You must decide which group to choose. Choose at least two measures of central tendency and at least one measure of dispersion for each group and use those to make your choice. Be sure to justify your choice with at least one page of discussion and analysis.

Answered by Penny Nom.
Ramp height 2007-12-03
From Steve:
I need to find the height of a angle. 23 degree angle at 12 feet of length is how high from the ground?
Answered by Stephen La Rocque.
Converting grams of hair spray into milliliters 2007-12-02
From komal:
I am confused that i am converting 75grams of hair spray ( liquid) into ML? but i couldnt get the answer what is perfect answer for that please help me for this thing.
Answered by Penny Nom.
The dimensions of a box 2007-11-30
From nephi:
if i wanted to make a box that was 0.35 ~ 0.7 Cubic Feet on the inside how would i go about making the measurements for the outside?
Answered by Penny Nom.
Transformations and compositions 2007-11-29
From mary:
Is there any possible relationship between composite functions and the concept of function transformations?
Answered by Harley Weston.
Density 2007-11-29
From jessuca:
what is the density for 40g / 2 mL
Answered by Penny Nom.
The width of a property 2007-11-29
From Angie:
I need to know what the footage is to find 8.1 acres knowing that the North boundary is 1309'
Answered by Penny Nom.
The perimeter of Wisconsin 2007-11-26
From Kyle:
I am doing a project and I need to figure out the total perimeter of wisconsin in miles and only know the square miles of Wisconsin which is 65,503 and I need to get that to miles I have no clue how to do this. I've looked all of the computer for answers but got none. Thanks
Answered by Leeanne Boehm and Penny Nom.
A curve sketch 2007-11-22
From Ahson:
Find critical points, determine the monotonicity and concavity and sketch a graph of f(x) with any local maximum, local minimum and inflection points labeled:

1. f(x) = x^4 - x^3 - 3x^2 + 1

Answered by Harley Weston.
The definition of the sine function 2007-11-22
From Indrajit:
I need a explanation in this theory.......if sinθ = p/h...then
sin 90 deg. = p/h
or 1 = p/h
or p=h .....how can a perpendicular be equal to a hypotenuse.???

Answered by Harley Weston.
Area of a 17-sided lot 2007-11-21
From Lynda:
My uncle is wanting to buy this piece of land [a 17-sided polygon] but we are questioning the acerage total. the measurements are [on the attached diagram].
Answered by Stephen La Rocque.
I know it's heavy, but how heavy, per linear foot? 2007-11-19
From Gary:
I am trying to find out the weight of a linear foot of a steel bar with the following measurements.
Thickness - 1.125"
Length - 120"
Height - 18"

Density of carbon steel - .2837
I know it's heavy, but how heavy, per linear foot?

Answered by Stephen La Rocque.
Cubic feet and gallons 2007-11-19
From Leonard:
Every conversion I find to convert a cubic foot to a gallon is in liquid. Question, how would 1 cubic foot convert to gallons in dry measurement? Example say you have 1 Gallon of packing peanuts how many cubic feet would that be? I can do this and have done it for the heck of it, I took a box 12x12x12 inches, which is a cubic foot, filled it with packing peanuts the tried to fill a 1 Gallon container, that amount almost fills a 1 gallon bucket twice. I even ask a math teacher and was told that 1 cubic foot equaled 7.481 gallons, then I said that is liquid, teacher ansered yes, then i ask what about dry and got a response of I do not know.
Answered by Harley Weston.
Sides and vertices 2007-11-17
From derek:
What is the correct name for a 2D geometric shape that would look like one slice from a round pizza and how may sides and vertices would it have? I believe you would say it has one vertex and two sides with the curved boundary not resulting in vertices or a side. Your help is much appreciated.
Answered by Chris Fisher and Walter Whitelty.
How many centimeters equal one meter? 2007-11-15
From Sherrie:
How many centimeters equal one meter?
Answered by Penny Nom.
The third side of a triange 2007-11-12
From Christine:
I need the inequality to describe the third side of a triange if two sides are known. First side is 23 and second side is 15. What is the third side?
Answered by Stephen La Rocque and Victoria West.
A mole of hydrogen peroxide 2007-11-11
From John:
i need to get a mole of hydrogen peroxide, which i know is 34g, but i don't have a way to measure grams, so do you know how many mililiters i need to have 34g of hydrogen peroxide. Thanks for all the help
Answered by Stephen La Rocque.
How do i convert milliliters to grams for water and also for copper sulfate (CuSO4)? 2007-11-11
From jenn:
how do i convert milliliters to grams for water and also for copper sulfate (CuSO4)?
Answered by Harley Weston.
KWH and MWH 2007-11-10
From Julie:
Please provide the conversion of MWH to KWH or vice-versa. thanks
Answered by Penny Nom.
Cubic centimeters to cubic yards 2007-11-10
From Logan:
if i have 188.4cm^3 of concrete how do i change it into cubic yards?
Answered by Penny Nom.
Related Rates (streetlamp and shadow) 2007-11-09
From Casey:
A street light is mounted at the top of a 15ft pole. A man 6ft tall walks away from the pole at a rate of 5ft per second. How fast is the tip of his shadow moving when he is 40ft from the pole?
Answered by Stephen La Rocque and Penny Nom.
Increasing and decreasing for functions 2007-11-09
From David:
Direction: Identify the open intervals on which the function is increasing or decreasing.

f(x)=1/(x^2)
f'(x)= -2/(x^3)

i understand how to get up until there, and the undf. is x=0, but now i'm having problem setting up the number table chart. i cant remember how, and where to place the increase and decrease + - the chart, for example <---------0----------> where would the increase and the decrease be place?

Answered by Harley Weston.
Meters per second to miles per hour 2007-11-08
From Niecey:
If Greg was timed at 0.91 seconds for 10 meters running the 100 meter dash, at that speed, could he pass a car traveling 15 miles per hour in a school zone?
Answered by Penny Nom.
Why is the number 1 neither prime nor composite? 2007-11-08
From amarilis:
why is the number 1 neither prime nor composite?
Answered by Penny Nom.
Changing units and sizes of things 2007-11-08
From Carol:
volume of cube = 216 cubic inches how do I calculate cubic feet and calculate its surface area in square feet. a rectangular box has dimensions of 3 x 6 x 15 how do I calculate surface area in square yards and calculate volume in cubic inches. if I double the dimensions how does it affect the volume
Answered by Stephen La Rocque.
Simple interest 2007-11-08
From Lee:
If a bank pays 3% simple interest anually on savings, and you did not take any money out of your account, how much money would you have deposited to earn $45 in interest
Answered by Stephen La Rocque.
Two triangles 2007-11-08
From Barbara:
If the base of a larger triangle is 34 inches long, what is the length of side A of the smaller triangle?

(the small triangle has on top the letter a on the right side of the triangle it has the letter b, and at the bottom of the small triangle it has the number 17)

Answered by Penny Nom.
I have a piece of material 44 inches by 78 inches 2007-11-02
From Marianne:
I have a piece of material 44 inches by 78 inches. How many linear yards can I get from it?
Answered by Penny Nom.
A decimal to a fraction 2007-11-01
From Lisa:
i am trying to turn a decimal into a fraction. my example is to turn 10.5 into a fraction at its lowest form
Answered by Penny Nom.
A wiffleball field 2007-10-31
From Svitlana:
The Adam's family has set up a wiffleball field in their backyard. The bases are arranged like a typical baseball diamond, where the distance between consecutive bases is the same. First base is opposite of third base, and second base is opposite of home plate. The distance between consecutive bases is 50 feet. Now, the pitcher stands 25 feet from home plate and lies on the line between home plate and second base. How far is the pitcher from first base? Round your answer down to the nearest inch.
Answered by Penny Nom.
Density 2007-10-30
From Ali:
Would an object sink or float in pure water if it has a mass of 10g and a volume of 5 cubic centimeters? 314
Answered by Stephen La Rocque.
Constantly accelerated motion 2007-10-29
From Priyanka:
A, B and C are three points lying in that order on a straight line. A body is projected from B towards A with speed 3 m/s. The body experiences an acceleration of 1 m/s^2 towards C. If BC =20 m, find the time taken to reach C and the distance travelled by the body from the moment of projection until it reaches C.
Answered by Stephen La Rocque.
Converting millimeteres to inches 2007-10-27
From Alison:
I am hoping to buy a cabinet, the measurements are 1120mm x 560mm, what is that in feet & inches.
Answered by Penny Nom.
The length of a rectangle is twice the width 2007-10-26
From Tami:
The length of a rectangle is twice the width. The number of square units in it's arear is four times the number of units in it's perimeter. What are the dimensions of the rectangle?
Answered by Penny Nom.
A sixfold increase 2007-10-24
From Fred:
If I have $500 and it grew to $3,000, what is the correct description of the increase? Is it a sixfold increase (sextupled), or a fivefold increase (quintupled)? When I divide $3,000 by $500, the result is six, therefore, a sixfold increase?
Answered by Stephen La Rocque.
Converting bases 2007-10-24
From Johanna:
Is there an easy way to convert from base to base. For example, base 5 to base 9 or base 2 to base 4 etc
Answered by Stephen La Rocque and Penny Nom.
The nth term 2007-10-18
From shannon:
Ok , what i am having problems with is the nth term. I get how the numbers come together, but i am having trouble with finding the nth term.
Answered by Penny Nom.
Convert 13.5 oz per sq yard of fabric into grams per meter square 2007-10-17
From marcy:
I need to convert 13.5 oz per sq yard of fabric into grams per meter square.
Answered by Penny Nom.
Paying with silver - Part 2 2007-10-17
From Shanna:
The paying with silver problem. I understand how to do the problem, but could you please explain how you would use base 2 arithmetic to solve it.
Answered by Penny Nom.
Calculate the dimensions of the rectangle 2007-10-16
From Sara:
The perimeter of a rectangle is 62cm. The length of the diagonal is 25 cm. Calculate the dimensions of the rectangle.
Answered by Stephen La Rocque.
Surface area of an open-ended cone 2007-10-16
From Lorne:
What is the surface area of an open ended cone? Measured at 10' high, 16' diameter on the bottom and 2' diameter at the top.
Answered by Stephen La Rocque.
How many tons of asphalt are in 12 cubic yards? 2007-10-15
From Matthew:
I want to know how many tons of asphalt are in 12 cubic yards?
Answered by Penny Nom.
Four triangles in a square 2007-10-15
From Kristina:
A square with side lengths of 6 cm is divided into 3 right triangles and a larger isosceles triangle. If the three right triangles have equal area, find the exact area of the isosceles triangle.
Answered by Stephen La Rocque.
The dimensions of a rectangle 2007-10-14
From Elizabeth:
The perimeter of a rectangle is 20m. The area is 20sqm. Calculate the dimensions of the rectangle. Give your answers correct to two decimal places.
Answered by Penny Nom.
Substitution method 2007-10-11
From Kevin:
3xx+2y=-36-y=11
Answered by Stephen La Rocque.
Area of a quadrilateral 2007-10-10
From Courtney:
how would i find the area of a quadrilateral..
the sides are a (/) is 6cm, b (—) is 9 cm, and c (\) is 7 cm..
the angle between a and b is 140 degrees and b and c is 115 degrees..

Answered by Stephen La Rocque.
Parabolic suspension bridge 2007-10-09
From Jessica:
A suspesion bridge with weight uniformly distributed along its length has twin towers that extend 75 meters abouve the road surfce and are 400 meters apart.The cables are parabolic in shape and are suspended from the tops of the towers. The cables touch the road surface at the center of the bridge. Find the height of the cables at a point 100 meters from the center. (Assume that the road is level.)
Answered by Stephen La Rocque.
Simplifying algebraic expressions 2007-10-09
From Sakeena:
(2^2*3)^x+1/2^2x*3x
Answered by Stephen La Rocque.
Simplifying rational expressions 2007-10-09
From Bama:
x(x-2)+1 divided by x^2-3x+2
Answered by Stephen La Rocque.
Subtracting rational expressions 2007-10-09
From Pamela:
Subtract:
6p – w     __     4c + p
 w – 4             w – 4

Answered by Stephen La Rocque.
Area and side length of a hexagonal tent 2007-10-09
From Mark:
Trying to buy a tent with 6 sides, in the catalog they give me sq. ft., what is the distance from side to side of a hexagonal tent with an area of 90 sq. ft.
Answered by Stephen La Rocque.
Coin jar 2007-10-07
From a student:
Sally empties his jar of coins. It contains $3.75 in nickels, dimes, and quarters. The number of dimes is twice the number of nickels and the number of quarters is three less than the number of nickels. Determine how many nickels, dimes, and quarters were in the in the jar.
Answered by Stephen La Rocque.
Solving four simultaneous equations (system of four linear equations) 2007-10-07
From Johan:
I need some help in solving this question
x + 2y - 3z + 4w = 12
2x + 2y - 2z + 3w = 10
0 + y + z + 0 = -1
x - y + z - 2w = -4

Answered by Stephen La Rocque.
Motion along a line 2007-10-02
From Claudette:
If the position function of a particle is x(t) = sin^2(2t), find the distance that the particle travels from t = 0 to t = 2
Answered by Harley Weston.
3 times x to the power -2 2007-10-01
From Jennifer:
The problem is 3 times x to the power -2.

I was just supposed to simplify, and I have no idea how.

Answered by Penny Nom.
I have a 6 digits 2007-09-28
From Kim:
I have a 6 digits. My hundred-thousands digit is 1 less that my ones digit, 8 more than my thousands digit, twice my tens digit, and 4 times my hundreds digit. My ten-thousands digit is 0. What number am I?
Answered by Stephen La Rocque.
Nine dots 2007-09-25
From al:
Connect the dots.
There is nine dots, three across and three down.
Connect the nine dot with the pencil only using four line and never lifting the pencil off of the paper.

Answered by Stephen La Rocque and Harley Weston.
Exponential regression 2007-09-24
From Anonymous:
I need help in finding this. Find a formula for exponential regression. Determine how to calculate a and b in the formula. Y = ba^x where y is as a function of x.
Answered by Penny Nom.
A 3 ounce piece of gold 2007-09-24
From Helen:
What would the volume (in mL) of a 3 ounce piece of gold be?
16oz= 1lb
1lb= 454oz.
first i converted 3oz to lbs and then to grams to get the mass. i got 85.125 g. now i don't know how to get the density to get to the volume.

Answered by Stephen La Rocque.
Finding equations, intersection point of two lines at right angles 2007-09-22
From Yaz:
Find the equation of the line joining A(-1,-9) to B(6,120). Another line passes through C(7,-5) and meets AB at rigth angle of D. Find the euation of CD and calculate the co-ordinates of D.
Answered by Stephen La Rocque.
Metres per minute to miles per hour 2007-09-20
From Angela:
If a person is traveling 150 meters per minute, what is their speed in miles per hour?
Answered by Stephen La Rocque and Harley Weston.
(5C squared + 14C + 11) divided by (5C + 4) 2007-09-20
From John:
(5C squared + 14C + 11) divided by (5C + 4)
Answered by Harley Weston.
Area of an irregular lot 2007-09-20
From Joanne:
I found your web site through ask.com & it's great. Really hoping you can help me. We are looking at purchasing an existing home with a lot size that is advertised at 2.04 acres, however, even with my limited math skills I know this is not correct.

Their calculation takes the deepest part of the lost (179 feet) multiplied by the widest part of the lost (497.26 feet) as if it was a perfect rectangle. As you can see from the faxed diagram it is far from rectangular in shape.

Can you advise as to the true lot size? As I mentioned we are looking at this property as a purchase so time is of the essence. I have also faxed this information (with the diagram) to the number listed on your web site. Thank you so much for your help.

Answered by Harley Weston.
how do you simplify 3a + 13b = 37 2007-09-16
From brandon:
how do you simplify 3a + 13b = 37
Answered by Penny Nom.
Volume, mass and density 2007-09-14
From Nathan:
I have a sample of material that is 10cmx10cmx5cm and has a mass of 29 grams. How many grams would a sample of the same material that was 96cmx188cmx5cm be?
Answered by Penny Nom.
How much stone will be needed to fill the trench? 2007-09-14
From Debbie:
A trench 100metres long 1ft wide*18inches deep with a 4inch pipe laid. How much stone will be needed to fill the trench.
How much weight in stone would I need to fill it?

Answered by Penny Nom and Victoria West.
Is it platinum or silver? 2007-09-14
From Jordan:
In Chemistry we are doing a lot of problems that are much more math related than anything. One of my questions reads: A certain medallion has a mass of 55.64 g. When placed in 75.2 mL of water in a graduated cylinder, the water level rises and then reads 77.8 mL. Is the medallion made of platinum (D=21.4 g/mL) or silver (D= 10.5 g/mL)?
Answered by Penny Nom.
The contents of a silo 2007-09-14
From Steven:
I need to know how to find the weight of the contents of a silo that is 4 feet across and is funnel shaped at 60 degrees
Answered by Penny Nom.
Composite angles 2007-09-13
From Gilbert:
what is a composite angle? please help
Answered by Stephen La Rocque.
What was the water level in the cylinder? 2007-09-11
From glen:
After a 5.63-g sample of Wood's metal has been added to the water in a 10-mL graduated cylinder, the new water level is 8.7mL. What was the water level in the cylinder before the sample was added?
Answered by Stephen la Rocque.
Markup to include 6% commission 2007-09-07
From Steve:
I estimate the cost to build a house for $250,000.00. I mark it up by 20%: $250,000 x 1.20 = $300,000 base price (BP) without sales commission. I want to set the sale price (SP) to include a 6% commission to pay the real estate fee. After the 6 % fee is paid I want to net the $300,000 base price. The formula is basically: $300,000 + 6% of SP = SP. But I can't figure out how to solve for SP.
Answered by Penny Nom.
2x+1+x-4+4x+1 2007-09-06
From laurie:
2x+1+x-4+4x+1
Answered by Penny Nom.
How many two digit numbers contain at least one 7? 2007-09-06
From Janet:
How many two digit numbers contain at least one number seven?
Answered by Penny Nom.
Variable expressions - part 2 2007-09-04
From lisa:
how do I write a variable expression for word phrase example x less than = 2 or 2 less than p 4 fewer than job please help
Answered by Penny Nom.
Variable expressions 2007-09-04
From Eric:
how so i write a numerical or variable expression for each quantity. example two dozen eggs or d dozen eggs number of quarts ub a 3 gallons
Answered by Penny Nom.
16x^2+4xy^2+8x divided by 4xy 2007-09-02
From Ricardo:
how can i solve that problem of polynomial division?

16x^2+4xy^2+8x divided by 4xy

Answered by Penny Nom.
The quotient of thirty and ten times a number. 2007-09-02
From Jenna:
I need help on How to translate a phrase into an algebraic expression: The quotient of thirty and ten times a number.
Answered by Penny Nom.
george, thomas, abe, alexander... 2007-09-01
From lori:
george, thomas, abe, alexander... come up with the next few im confused and need help!!
Answered by Stephen la Rocque and Harley Weston.
Similar triangles 2007-08-29
From James:
Question: In the triangle ABC, X is a point on AC. AX = 15 m and XC = 5 m. The angle AXB is 60 degrees and the angle ABC is 120 degrees. Find the length of BC.

I am sure to an extent that this has to do with similar triangles, but I am not certain.

Answered by Harley Weston.
Converting a repeating decimal to a common fraction 2007-08-22
From isabelle:
how do you turn 6.333... into a fraction in simplest form?
Answered by Stephen La Rocque and Penny Nom.
More on the sixth degree equation 2007-08-21
From Farzan:
Equation 1 :
(R + (6.67*5.98*10^13/((R^2)*2)))^2 - (R^2) = (7.27*(10^-5)*R)^2
Equation 2 :
(6.67*5.98*10^13/(R^2*2))^2 + 6.67*5.98*10^13/R = (7.27*10^-5*R)^2
As you see if we expand the left part of the first equation and simplify, the equations become same, but these two equations have different answers in my math software.The first one has 3 amounts for R, but the second one has 6 amounts. Why are the answers different ?

Answered by Stephen La Rocque.
A sixth degree equation 2007-08-20
From Farzan:
Dear friends I need to solve this equation with an understandable method for high-school students : (6.67*5.98*10^13/(R^2*2))^2 + 6.67*5.98*10^13/R = (7.27*10^-5*R)^2 please solve this problem if you have time.
Answered by Harley Weston.
The area of a five sided lot 2007-08-19
From Karyn:
I have been looking at your site, but I am still confused as to how to figure out the exact square footage of my irregular lot (since it is a gradual fan lot from the front, and triangles out in the back).
Answered by Harley Weston.
What is 240mL of water in cups? 2007-08-16
From Gareth:
What is 240mL of water in cups?
Answered by Penny Nom.
Angle of depression to a football 2007-08-15
From Tosin:
The angle of depression to a football from the top of building 10m high is 17 degrees. Find the distance of the ball from the foot of the building.
Answered by Stephen La Rocque.
Division by decimals 2007-08-15
From Brooklyn:
I am studying for an exam, and I will not be able to use a calculator. I can not figure out how to do division is the number on the outside of the box is a decimal. Ex: 2/6.3=X Can you please help me figure this out?
Answered by Penny Nom.
The length of the third side of a triangle 2007-08-15
From Brooklyn:
What is the equation to find the length of the third side of a triangle if you have the length of A, B, and the angles(s)?
Answered by Stephen La Rocque.
The tension in a single strand of wire 2007-08-13
From Gene:
Seeking an equation that calculates the tension in a single strand of wire or rope. Conditions: the wire is fixed at both ends, A known weight is suspended from the center of the span. The weight will dipslace the strand downward some distance Y. Let the distance between the fixed ends be X= 500mm. Weight W=0.5kg. Original tension in the strand is T= 10 newtons.Your help sincerely appreciated.
Answered by Gabriel Potter and Stephen La Rocque.
Water bill woes 2007-08-12
From Terry:
I know that a cubic foot of water=7.48051945 gals., but I keep getting my water bill and they go by thousands of a gal. per cubic foot. Is there a conversion that says that shows this? Water department=.763 x cubic foot used=thousands of gallons water used that month. Exampel: 72cu. ft meter reading x .763+ 58.--- galons of water I use in a month. Total 58=thousand = +gallons. This is wrong. There are only two people in the house.
Answered by Stephen La Rocque.
The swaying of a building in the wind 2007-08-11
From San:
During a strong wind, a tall builing, such as the CN Tower, can sway back and forth as much as 100cm, with a period of 10 seconds. Please help me to determine the equation for this function, in the form y=asinkx
Answered by Stephen La Rocque.
Induction - divisibility 2007-08-04
From Jerry:
How would you prove that for any positive integer n, the value of the expression 3^(2n+2) - 8n -9 is divisible by 64.
Answered by Chris Fisher and Penny Nom.
Rolling a metal plate 2007-07-31
From k.a.suresh:
How to calculate the plate size for rolling (2010mm OD)plate thick is 6mm
Answered by Harley Weston.
Subdividing a property into half acre parcels 2007-07-31
From Pamela:
We need to divide a large property with five sides and strange angles into 1/2 acre parcels from the top down. We want to know where along the eastern and western borders of the large property to make the divisions.
Answered by Stephen La Rocque.
Vehicle collision 2007-07-31
From Tania:
If a driver falls asleep at the wheel of a newer model car and slams into the rear of a parked classic mustang (2860-3280lb) and the parked car is thrown 100 ft across the street, what would be the minimum speed required by the newer car in order to move the mustang?
Answered by Penny Nom.
320 acre lots 2007-07-29
From melisa:
we are splitting a 320 acre parcel into five 35 acre lots with one 145 ace lot. I have the footage of each side there are eight: 5280 x 2640x2640then out1320x1320up x2640back inx1320upx1320this now would meet back to the 5280foot side i know there is a half of a square mile and 320 acres.what would the measurements be in feet for length to make a square or a rectangle to make a 35 acre lot?
Answered by Harley Weston.
Angle of depression 2007-07-23
From joyce:
hello, here is my problem......
As you stand on a bridge w/c is 100 ft. above the water you are looking @ an approaching barge. If the A of top of the front of the bridge is 29.04 degrees and the angle of depression of the rear is 17.36 degrees . Find the length of the barge?

Answered by Harley Weston.
f(x) = (x^4) - 4x^3 2007-07-22
From Michael:
I'm a student who needs your help. I hope you'll be able to answer my question. Here it is: Given the function f(x)=(x^4)-4x^3, determine the intervals over which the function is increasing, decreasing or constant. Find all zeros of f(x) and indicate any relative minimum and maximum values of the function.
Any help would be appreciated. Thank you for your time.

Answered by Harley Weston.
Angle of depression (declination) between sailboat observations 2007-07-18
From Joyce:
From a cliff 150 ft above a lake, we see a boat sailing directly towards us. The angle of depression of the boat is seen to be 5 degrees and 7 inches and 11 degrees and 18 inches. Find the distance sailed between observations.
Answered by Stephen La Rocque.
The area of a property 2007-07-17
From Mary:
I currently have an opportunity to sell my property to a business and they would like me to send a proposal to them with details. I need to know the square footage of my property and have tried using your formula from similar questions. The dimensions of my property are as follows
front line facing street: 19.908 m
right side : 37.338 m
back line : 29.779 m
left side : 43.983m
The front line and the right side are at right angles to each other. Can you formulate and reply? Thank you Mary

Answered by Harley Weston.
Sample size 2007-07-17
From Nurse:
If a population has standard deviation 20, what is = the minimum sample size to be 95% confident that the error should be = accurate to within 4?
Answered by Harley Weston.
Radical 96 plus radical 27 divide by radical 3 2007-07-17
From linda:
radical 96 plus radical 27 divide by radical 3
Answered by Penny Nom.
Trig functions for angles not between 0 and 90 degrees 2007-07-16
From Tim:
My question: Why is the value of a trigonometric function, the same, for an angle over 90 degrees and its reference angle? How are the angle and its reference related? Do they both form a triangle that has equal sides?
Answered by Penny Nom.
The two towers (angles of elevation trigonometry) 2007-07-14
From joyce:
The angle of elevation of tower B from the top of tower A is 28 degrees and the angle of elevation of the top of tower A from the base is 46 degrees Find the height of tower A if tower B is 120 m high?
Answered by Stephen La Rocque.
Any regular polygon inscribed in a circle 2007-07-12
From DJ:
Circle with r=12" is inscribed in a regular octagon. What is the length of each octagon segment? Note: Our answer works for any regular polygon inscribed in any circle.
Answered by Stephen La Rocque.
Converting measurements 2007-07-11
From Nicole:
if you have 8 oz of water how many cc: will there be.
Answered by Stephen La Rocque.
The probability of rolling double sixes twice in a row 2007-07-10
From Matt:
what is the probability of rolling double sixes twice in a row?
Answered by Stephen La Rocque.
Grams of vodka 2007-07-10
From Andrew:
milliliters to grams..vodka 80 proof?
Answered by Stephen La Rocque.
Derivative of a Function 2007-07-09
From Bob:
What is the derivative of the function a sub n = [n/(n+1)]^n ?
Answered by Stephen La Rocque.
Proof that any side of a triangle is less than half the perimeter. 2007-07-07
From Omkar:
Any side of a triangle is smaller than half of its perimeter, prove this in short ?
Answered by Stephen La Rocque.
Finding the radius of an inscribed circle 2007-07-05
From Maria:
I need to find the radius of a circle which is inscribed inside an obtuse triangle ABC. I know all the angles and all the lengths of the triangle.
Answered by Stephen La Rocque and Chris Fisher.
Tons of concrete 2007-07-03
From Trey:
I have 300,000 square feet of concrete and want to know how to convert this to tons?
Answered by Harley Weston.
A statistical difference 2007-07-03
From a student:
I am writing a paper for a grad class in which I am comparing achievement levels of boys and girls. I have data that shows the average marks for girls and boys in each grade 12 subject for the entire province. My prof want me to show there is a statistical difference. What do I have to do to show this? I don't have access to the raw data, so don't know the SD. Is there anything I can do mathematically with the data that I have? (I can determine n for each set of data, it ranges between 2400 and about 13000 for each subject area.)
Answered by Chris Fisher and Harley Weston.
Expansion on a steel boat 2007-06-28
From JOHN:
that is the expansion on a steel boat that is 300' long by 100' wide. if the water temp is 72deg f. and the deck temp is at 150deg f. I think the deck would grow in all dir. and by what dist? do I still use 0.00000645
Answered by Stephen La Rocque.
How do I convert milliliters(powder form) to grams? 2007-06-28
From ed:
How do I convert milliliters(powder form) to grams
Answered by Harley Weston.
How do I convert inches to mm in my head? 2007-06-27
From philip:
Hi, I would like to know how to convert inches to mm in my head, I use socket sets and want to know an easy way to work out what 3/16, 7/32 is in mm. Thanks Phil
Answered by Harley Weston.
sin|x| and cos|x| 2007-06-25
From Mac:
Can anyone tell me whether sin|x| and cos|x| is differentiable at x=0 ? As far as i know, cos(x) and sin(x) is differentiable at all x.
Answered by Penny Nom and Stephen La Rocque.
Volume in a triangular water trough 2007-06-24
From David:
A water trough has sloping sides of length 500mm making it triangular in cross section, with vertical ends. The width at the top is 600mm and the length is 2.0 metres.
(i) Calculate the capacity of the trough, giving your answer accurate to the nearest litre.
(ii) Find out the depth of the water when the trough is half full.

Answered by Stephen La Rocque.
Average speeds 2007-06-22
From Dee:
A guy goes 3.1m a second for 29 seconds in a golf cart. He then walks 1.5 meters a second. How long does he have to walk to get the average speed of 2.1m a second?
Answered by Stephen La Rocque.
Simplify 2007-06-21
From A student:
what is the answer of 3y+5/2(4y+1)+5y/2
Answered by Penny Nom.
Simultaneous equations : the Elimination method 2007-06-21
From Patricia:
I need to find the value of X and Y using the Elimination method.

5/x + 3/y=4
25/x-2/y=3

Answered by Stephen La Rocque.
New wood siding 2007-06-20
From Angela:
I have a trailer that is 14x70 and I took off the wood siding that was about 8 feet tall. They want $8.60 a cubic yard. How many cubic yards do I have and how much will it cost me. Thanks to all!
Answered by Stephen La Rocque and Penny Nom.
Making a box with a particular volume 2007-06-20
From Kimberly:
How do you convert 487 cf into box?
Answered by Stephen La Rocque.
How many tons of sand? 2007-06-19
From Richard:
I need to know how many tons of sand it would take to fill a container 20 feet long, 20 feet wide, and 6 inches deep. I am not sure of the makeup of the sand as far as dry, loose, wet, etc. If you could give me calculations of all the variants it would be appreciated.
Answered by Stephen La Rocque and Penny Nom.
Simultaneous inequalities 2007-06-18
From Freddy:
Watson Electric has production facilities in Valley Mills, Marlin,and Hillsboro. Each one produces radios,stereos,and TV sets. There production capacities are

Valley Mills: 10 radios, 12 stereos, and 6 TV sets per hour Marlin: 7 radios, 10 stereos, and 8 TV sets per hour Hillsboro: 5 Radios, 4 Stereos, amid 13 TV sets per hour

QUESTION

How many hours should each plant be scheduled to fill an order of 1095 radios, 1230 stereos, and 1490 TV sets?

Answered by Stephen La Rocque.
Simplifying a quartic rational expression using long division 2007-06-14
From Megan:
x+2/12x^4+17x^3+0x^2+8x-40=
Answered by Stephen La Rocque and Penny Nom.
The composition of functions 2007-06-14
From Gilligan:
Question from Gilligan, a student:

Let f(x) = x^2, g(x) = 3x and h(x) = (sqrt{x}) + 1. Express each function as a composite of f, g and/or h.

(1) p(x) = 3(sqrt{x}) + 3

(2) Q(x) = (sqrt(sqrt{x}) + 1}) + 1

Answered by Penny Nom.
Angles of depression 2007-06-13
From Phonda:
The pilot of a small private plane can look forward and see the control tower for a small airstrip. Beyond that is a large factory that is 3 milies from the airstrip. The angles of depression are 12.5 degrees and 4.8 degrees respectively. Find the airplane's altitude, to the nearest ten feet.
Answered by Stephen La Rocque.
Standard Deviation 2007-06-13
From Adrian:
If you are told that the mean salary of a certain group of workers is $30,000 with a standard deviation of $4000, what proportion of workers earn over $38,000? What proportion of workers earn less than $18,000? Assume the distribution of wages is normal.
Answered by Stephen La Rocque.
Composition functions 2007-06-12
From Gilligan:
Find functions f and g so that f(g(x)) = H.
(1) H(x) = (1 + x^2)^(3/2)
(2) H(x) = int(x^2 + 1)
I don't know where to start.

Answered by Stephen La Rocque.
Converting dollars to gallons 2007-06-12
From Cynthia:
How do I solve this problem? If a family uses .20 per 100 gallon of water. How many gallons would $10.00 cost, divided amoung 3 people? Thanks, Cynthia
Answered by Stephen La Rocque.
How high does the ladder reach? 2007-06-11
From Madi:
A ladder 8 ft long resting on a house makes a 60 degree angle with the ground. how far up the house does it reach?
Answered by Stephen La Rocque.
The area of a quadrilateral 2007-06-10
From Lucy:
Calculate the area of the quadrilateral ABCD. AB= 4.1cm, BC = 7.6cm, AD= 5.4 cm, CD= ? Angle ABC = 117, Angle ADC = 62. Give your answer correct to 3 significant figures.
Answered by Stephen La Rocque and Penny Nom.
Rational polynomial expressions 2007-06-09
From a student:
I have a question that continues to lead me to the answer x/(x - 1) but according to my math book, the answer should be 1/(x - 1). Who is right? Who is wrong? Here is the question: (x^2 - 6x - 27)/(x^2 - 11x + 18) DIVIDED by (x^3 + 2x^2 - 3x)/(x^2 - 2x)
Answered by Stephen La Rocque.
The law of sines 2007-06-09
From Felicia:
A parallelogram has one side that is 12.0 cm and one angle that is 65°. The shorter diagonal is 25.0 cm. To the nearest tenth of a centimetre, how long is the other side of the parallelogram? Use the sine law.
Answered by Penny Nom.
Finding the area of a lot without knowing the angles 2007-06-05
From Cristin:
I have a land-locked piece of land. I need to know the square footage in order to determine the acreage. My deed only gives the lengths of its five sides.
Answered by Stephen La Rocque.
Cubic inches and square feet 2007-05-31
From keith:
is it possible to convert cubic inches to square feet.thanx
Answered by Stephen La Rocque.
The number of grams of sulfuric acid 2007-05-31
From cookie:
Hi,
Can you please let me know if I've done this correctly?

Calculate the number of grams of sulfuric acid in 0.500L of battery acid if the solution has a density of 1.28 g/mL and is 38.1% sulfuric acid by mass.

Answered by Stephen La Rocque.
I want to divide my land 2007-05-27
From alfredo:
i want to divide my land. It is north is 11.86 ft, the S is 110.00 ft, the W is 644.34 ft the E is 644.58 ft. which =to1.606 acre. how many feet going from N to S would it be to make 1.5 acre
Answered by Penny Nom.
Area of a hexagon (three ways to find it) 2007-05-24
From Josh:
I have to work out the area of a hexagon when I know that the perimeter of it is 1000m. I have tried many different ways but I get different answers.
Answered by Stephen La Rocque.
Simplifying a square root 2007-05-24
From Leigh:
I am trying to help my son. Simplify sq root 81 x to the 2nd power y to the 12th power.
Answered by Stephen La Rocque.
System of equations 2007-05-24
From Chris:
Find all real solutions (x,y,z,w) of the system of equations:
2y= x + x/17, 2z= y + y/17, 2w = z + z/17, 2x= w + w/17

Answered by Penny Nom.
How many tons of dirt is this? 2007-05-23
From Deb:
I need to know how many tons of dirt(6 inches deep)would it take to cover 8 acres?
Answered by Stephen La Rocque.
Converting yogurt 2007-05-21
From Annie:
I'm trying to follow a diet plan. The diet says I should have a 125ml tub of natural low fat bio yogurt. The only problem is, the yogurt pot lists its weight as 150g (with a large 'e' after it). Can you tell me how much of the yogurt pot I'm suppost to have please, or give me a conversion chart to change from g to ml for natural bio yogurt?
Answered by Stephen La Rocque.
Converting mass of sand to volume (a sand ceremony) 2007-05-21
From Linda:
I'm getting married. We are having a sand ceremony. The empty bottle we are using holds 800ml. I have bags of colored sand. Each bag is 30g. There is 7 people participating in this ceremony. What I need to find out is how many grams of sand i need for 7 people to participate to fill this jar.
Answered by Penny Nom.
Set up two simultaneous equations 2007-05-21
From Admire:
The cost of producing windscreen wipers blades at a factory ais partly fixed (due to operating overheads) and partly dependent on the number of blades produced. It costs $300 to produce 1000, and $600 to produce 5000 blades. How much would it cost to produce 24000 blades?
Answered by Penny Nom.
Divisibility 2007-05-18
From Ashish:
A number is divisible by 2^n if the last n digits of the number are divisible by 2^n. Why?
Answered by Penny Nom and Claude Tardif.
Null hypothesis 2007-05-13
From Dani:
A poll of 1068 American voters indicated that 48% supported te Democratic candidate for presidency. At the 0.05 level of significance, test the claim that less than 50% of American voters preferred the Democratic candidate.
Answered by Penny Nom.
Find the sample size needed 2007-05-13
From Mini:
Find the sample size needed to be 98% confident thata marketing survey on the proportion of shoppers who use the internet for holiday shopping is accurate within a margin of error of 0.02. Assume that the conditions for a binomial distribution are met, and that a current estimate for a sample proportion does not exist.
Answered by Penny Nom.
An octagonal sign 2007-05-13
From Cathy:
An octagonal sign is made by cutting four identical triangles made from a rectangle calculate area of one of the discarded triangles. length of square=26cmx22cm. length of octagonal sides are 10cm x 13cm
Answered by Penny Nom.
Finding the hypotenuse without Pythagorus 2007-05-11
From Shelbie:
How do i find the hypotenuse of a right traingle not using the pythagorean thereom if i have the measurements of the legs?
Answered by Stephen La Rocque.
Angle of Elevation 2007-05-10
From Micky:
Two Buildings are on opposite sides of a street 40 feet wide. The taller of the two buildings is 580 feet tall. The angle of depression from the top of the tallest building to the shorter building across the street is 57 degrees. Find the height of the shorter building.
Answered by Stephen La Rocque.
A ton of sawdust 2007-05-10
From David:
I am trying to do a conversion. A tractor trailer is loaded with 165 cubic yards of sawdust (I do not know the actual weight). I knw the wholesale cost as $1,000 for this amount, but would like to convert this to find out what the equivalent cost for a metric ton would be.
Answered by Stephen La Rocque.
Area of irregular surfaces 2007-05-09
From Dustan:
I am working on a way to compute very accurate areas for irregular surfaces by using the idea of a largest possible circle...
Answered by Chris Fisher.
Converting from mm to cm 2007-05-08
From Amanda:
How do you convert mm into centimeters?
Answered by Gabriel Potter and Steve La Rocque.
Probability of H.I.V. tests 2007-05-07
From Danielle:
A medical test detects H.I.V. Among those who have H.I.V., the test will detect the disease with probability 0.95; among those who do NOT have H.I.V., the test will falsely claim that H.I.V. is present with probability 0.0125. Among those who take this test, 4% have H.I.V. The test is given to Lucille, and indicates that she has H.I.V. What is the probability that Lucille actually has H.I.V.?
Answered by Stephen La Rocque.
Evaluating sine and cosine 2007-05-06
From Selimovic:
How can i solve sine or cosine for angle of, lets say 10°....Maybe it's easy but i don't know how...
Answered by Penny Nom.
A speaker box 2007-05-05
From darell:
I am trying to build a speaker box, and i am trying to find out the measurements of a 1.5cubic feet box.
Answered by Penny Nom.
Square footage of a lot 2007-05-03
From Linda:
I have two lots which I am trying to calcuate the square footage of

one lot is 50 x 285.72 x 50 x 270

2nd lot is 50 x 285.72 x 50 x 277.61

Any help would be appreciated. The answers would be GREATLY appreciated.

Answered by Penny Nom.
My commsion is 2.5% 2007-05-02
From susan:
How do you do percentage say i sold a house for 350,000 my commssion is 2.5% how do i figure that out can you show the formula for calculator please thank you very much
Answered by Penny Nom.
Simplify -36x squared over 18x 2007-05-01
From Emily:
Simplify -36x squared over 18x
Answered by Penny Nom.
A tugboat's speed 2007-04-30
From Amanda:
a tugboat must travel 24 miles against a 4 mile per hour current on the Potomac River and return. At what constant speed must the tugboat travel to make the trip in 12 hours. Round answer to the nearest tenth mph.
Answered by Stephen La Rocque.
A couch sliding off a truck 2007-04-30
From William:
A couch with a mass of 1 X 10^2kg is placed on an adjustable ramp connected to a truck. As one end of the ramp is raised, the couch begins to move downward. If the couch slides down the ramp with an acceleration of .70 meters per second when the ramp angle is 25 degrees, what is the coefficient of kinetic friction between the ramp and couch? I drew a force diagram and if I did it correctly I identified the forces involved as "mg" (mass x gravity), "Fn" (normal force) and the "Ff" (frictional force). I know that we have the couch sliding down the ramp a .70 m/s but I don't think this a force and I'm not sure how this info fits into the problem. I know that the formula for calculating the coefficient of friction is Ff/Fn. Based upon the force diagram I drew and calculated Fn to be 1082N. I can't seem to get past this point. How do I determine what the frictional force is?
Answered by Stephen La Rocque.
Simplifying a rational expression 2007-04-29
From Tamika:
X^2 + 10X + 24
______________
X + 6
SIMPLIFY EACH RATIONAL EXPRESSION (IF YOU CAN'T SIMPLIFY IT, WRITE ALREADY SIMPLIFIED)

Answered by Stephen La Rocque.
How many tons of sand is needed 2007-04-25
From Jennifer:
How many tons of sand is needed to cover 350 cubic square feet?
Answered by Penny Nom.
How can I convert milliliters to grams? 2007-04-18
From April:
What if I'm using cow's milk, how can I convert milliliters to grams?
Answered by Stephen La Rocque.
By how many feet was each dimension increased? 2007-04-18
From Becca:
A rectangle is 8 feet long and 6 feet wide. If each dimension is increased by the same number of feet, the area off the new rectangle formed is 32 square feet more than the area of the original rectangle. By how many feet was each dimension increased? I need a diagram and 5 or more sentences for an explanation. Thanks!
Answered by Stephen La Rocque.
Simultaneous equations 2007-04-16
From kyrie:
simultaneous equation 4x + 3y = 21 2x * y = 8
Answered by Penny Nom.
Ounces and liters 2007-04-16
From Simone:
I want to know how much is 80 ounces of water in liters?
Answered by Penny Nom.
Calculating interest 2007-04-15
From Gerry:
Calculate the amount of interest received on £120.00 desposited in an account paying 3% simple interest for three years.
Answered by Stephen La Rocque.
The angle of depression 2007-04-14
From Mary-Beth:
Two towers are 30 m apart. From the 15th floor, 40 m up, find the angle of depression to the base of the taller tower? I think the answer is 53, but the answer book says 37.
Answered by Penny Nom.
A hypothesis test 2007-04-09
From Katrina:
I have already tried to do this problem but im having a very had time with it. Can you please help me.

Glamour Magazine sponsored a survey of 2500 prospective brides and found that 60% of them spent less than $750 on their wedding gown. Use a 0.01 significance level to test the claim that less than 62% of brides spend less than $750 on their wedding gown. How are the results affected if it is learned that the responses were obtained from magazine readers who decided to respond to the survey through an Internet Web site?

Answered by Penny Nom.
Yards of sand in a ton 2007-04-09
From Teresa:
how many yards of sand are in a ton and how many yards of limestone does it take to make a ton
Answered by Penny Nom.
3 divided by 3 to it's fifth root 2007-04-06
From Annie:
How do I transform the equation 3 divided by 3 to it's fifth root to simple radical form (getting the radical out of the denominator)?
Answered by Penny Nom.
Simplify (1/3 + 1/3x) / (1/x + x/3x) 2007-04-04
From Farina:
how do i simplify (1/3 + 1/3x) / (1/x + x/3x)
Answered by Penny Nom.
What is the intensity 5m below the surface? 2007-03-31
From david:
I have this question which I am supposed to set it up and solve as a differential equation. I know how to solve the diffrential equation but I am having hard time understanding this question. Here is the question: The intensity of light in the ocean decreases the deeper you dive. In fact, the rate at which the intensity decreases is proportional to the current intensity. Setup the corresponding differential equation and solve for I(Y), the intensity I as a function of current intensity Y. If the light intensity 2m below the surface is 25% of the intensity at the surface, what is the intensity 5m below the surface. Can you please explain to me what does it mean by current intensity and how do I set this equation up. Thanks for the help.
Answered by Penny Nom.
The size of a lot 2007-03-30
From Phyllis:
Could you please figure out the square footage and acreage of my lot? One side is 480ft One side is 410ft One side is 480ft The last side is 410ft Thank you.
Answered by Penny Nom.
6- team schedule for softball 2007-03-29
From Angie:
I have 6-teams that can only play 3 days a week. Each team must get 18 games in for the season. I have to split them being the home team and the away team sometimes.
Answered by Penny Nom.
Cut a cake into four pieces 2007-03-27
From haley:
A cake measures 10 inches x 10 inches. The cake was cut into 4 pieces that were the same size. What could the size be of each piece?
Answered by Penny Nom.
Factoring a trinomial 2007-03-27
From Kim:
Hi, could you please help me solve this.

3x(squared) +20x - 7

Answered by Leeanne Boehm.
The distance between two fire towers 2007-03-23
From tony:
Two fire towers are 30km apart, tower A is due west of tower B. A fire is spotted from the towers, and the bearing from A and B are N76degreesE and N56degreesW, respectively. Find the distance from the fire to the straight line connecting tower A to tower B.
Answered by Stephen La Rocque.
The law of cosines 2007-03-23
From chetna:
Q 1) In triangle LMN, l=7, m=5 , n=4. find ANGLE N. After applying the rule and substituting values i'm getting Cos n= 58/40. Is there something wrong. The answer at the back of the book is 34 degrees.
Answered by Penny Nom.
Adding and Subtracting Rational Expressions 2007-03-21
From Pricilla:
Pricilla

I do not know how to do this problem. Don't know where to start.

[(5)/(2x-8)]+[(3x^2+15)/(x^3-4x^2+5x-20)]

Answered by Stephen La Rocque.
Angles of depression 2007-03-21
From romaine:
a woman of height 1.4m standing on top of a building of height 34.6m veiws a tree some distance away. she observes that the angle of depression of the bottom of the tree is 35 degrees, and the angle of depression of the top of the tree is 29 degrees. assume that the building and the tree is on level ground.
(a) calculate the distance of the woman from the top of the tree measured along her line of sight.
(b) determine the height of the tree.

Answered by Stephen La Rocque.
Synthetic division 2007-03-21
From Edward:
Hi there, please, explain me synthetic division of polynomial functions. for examp.: 3x3 + 2x2 +4 divide by x-2 (number after variable x is the powre of x ) Thank you. Edward.
Answered by Melanie Tyrer.
n-1/(n+1)! + n+1/n! 2007-03-18
From Cody:
How do you go about simplifying something like this; n-1/(n+1)! + n+1/n!?
Answered by Steve La Rocque and Claude Tardif.
Percentage markup 2007-03-16
From Debbie:
What formula or formulas do I use if I want to ultimately charge a customer $300 for a service, but need to factor in a 15% commission for a sales rep, $20 to cover the cost of our overhead, the cost of a $25 coupon that we will be sending to the customer to apply to future services. In other words, what amount do I initially quote the customer to be able to cover these costs?
Answered by Stephen La Rocque.
Conversion 2007-03-16
From Andrew:
If a lawn is 95.3 square yards, how do I convert that to square meters?
Answered by Stephen La Rocque.
Concrete sand 2007-03-08
From david:
Can you tell me how much 4 cubic yards of concrete sand ways in tons? THANK YOU.
Answered by Stephen La Rocque.
Rotor-ride 2007-03-07
From Jessie:
In a "Rotor-ride" at a carnival, people are rotated in a cylindrically walled "room". The room radius is 4.6m and the rotation frequency is 0.550 revolutions per second when the floor drops out. What is the minimum coefficient of static friction so that people will not slip down? People on this ride say they were "pressed against the wall". Is there really an outward force pressing them against the wall? If so, what is its source? If not, what is the proper description of their situation (besides scary)?
Answered by Stephen La Rocque.
The volume of a trailer 2007-03-04
From Ron:
I just can not seem to work this out, i think it is old age catching up with me, I am buying a trailer that is 25 feet long 7.5 foot wide and with 4 foot sides and i am trying to work out how many m3 it will hold can anyone help. Thank you in advance for any help. Regards Ron
Answered by Penny Nom.
A square 2007-03-02
From Jake:
how many sides does a square have?
Answered by Sara Ulmer.
Grams to millilitres 2007-03-02
From anton:
I am trying to convert 28 grams of weight loss powder into mils
Answered by Gabriel Potter.
Mutually exclusive events 2007-03-01
From kalyssa:
will you me an example of two events that are mutually exclusive and could you explain to me what mutually exclusive means?
Answered by Steve La Rocque, Pam Fowler and Penny Nom.
How many yards of concrete? 2007-02-28
From jeremy:
I need to figure how many yards of concrete to use. I work for an asphalt company, and we are expanding into concrete. Currently we measure everything in sq. ft. So we came up with 6,960 sq ft of concrete to remove and replace. Currently there is 6" of concrete, and 2" of bedding material. We would like to replace only 4.5" of concrete, the rest in bedding. Here is how I figured, am I correct. 6,960 Sq ft (to simplify I made the Sq ft 350 X 20 ) 350 ft X 20 ft X .33 (1/3 ft or 4") = 2,310 cubic feet 2,310 cb ft / 27 = 85.55 cubic yards Is this correct?
Answered by Penny Nom.
Simultaneous equations with fractions 2007-02-28
From Alyca:
Hello Math Central, I am a grade 10 student taking Academic math. Our unit right now is method of substitution and elimination. I'm stuck on this one question that I've been doing forever. Please help =)

*For this equation I have to do method of elimination, but it's so much harder with fractions...could some one please explain to me how to do it step by step?*
x y 2
-- - -- = - --
3 6 3

x y 1
-- - -- = 1---
12 4 2

Answered by Steve La Rocque and Ashley Mang.
Similar triangles 2007-02-21
From Vivienne:
The lengths of the sides of a triangle are 8, 15, & 17. If the longest side of a simlar triangle is 51, what is the length of the shortest side?
Answered by Stephen La Rocque and Penny Nom.
The length of the side of a hexagon 2007-02-19
From Nick:
if the area of a regular hexagon is 259.807 how do i find the length of each side, and the apothem? thanks if you can help
Answered by Penny Nom.
What is the remainder when 3^2007 is divided by seven? 2007-02-18
From Chris:
What is the remainder when 3^2007 is divided by seven??
Answered by Leeanne Boehm.
Optical illusions 2007-02-18
From Jami:
Hi, I'm Jami and I'm in 10th grade.I'm doing a geometry research project on optical illusions and need to know how math is involved.I have an idea already of how our eyes percept 2 dimensional images and construct them into 3D images but, that isn't my question.There are many books that contain optical illusion pictures that have secret messages in them or have objects popping out. Is there a mathematical way in that optical illusion pictures are constructed?
Answered by Walter Whiteley and Harley Weston.
A gymnast is swinging on a high bar. 2007-02-14
From William:
A gymnast is swinging on a high bar. The distance between his waist (center of mass) and the bar is 1.1 m. If his speed at the top of his swing is zero and his gain in speed is due entirely to his change in gravitational potential energy, find his speed at the bottom of his swing.
Answered by Stephen La Rocque.
Centimeters to cubic meters and grams to kilograms 2007-02-14
From Colleen:
A box of balls measures 120cm by 20cm by 20cm and weighs 48 g.
One thousand boxes = x cubic meters and weighs x kilograms.
Also, one million boxes = x cubic metes and weighs x kilograms.
One billion boxes = x cubic meters and weighs x kilograms. Solve for x.

Answered by Stephen La Rocque.
Density 2007-02-09
From Judy:
How do I calculate the mass in grams of 125mL of chloroform when d=1.49g/mL.
Answered by Stephen La Rocque.
Conversions 2007-02-09
From Donna:
Hi! I hope you can help me... What is the cost of 10 grams platinum if one ounce = $1200 (troy weight)? I REALLY appreciate your help. Can you please send me the formula to figure it out too? Thank You! Donna :)
Answered by Stephen La Rocque.
How many grams are in a 12 ounce coconut? 2007-02-08
From Tristan:
I have a problem where I need to know how many grams are in a 12 ounce coconut. Do I use "dry measure" with figuring out how many grams are in a ounce? And is that about 28 grams? Thanks.
Answered by Penny Nom.
Converting sq inches to sq millimeters 2007-02-08
From Michael:
If I have 1/32 sq inches, what does that equal in sq millimeters?
Answered by Stephen La Rocque.
Simultaneous equations with envelopes 2007-02-08
From Mick:
There were 17 envelopes bought, some were brown, some were white. The brown envelopes cost one cent more per envelope than the white ones. The total cost was 80 cents. How much of each type of envelope was bought? --Many thanks!
Answered by Stephen La Rocque.
Find the next 3 terms of the sequence 2007-02-07
From rose:
find the next 3 terms of the sequence 2,3,9,23,48,87
Answered by Penny Nom.
Multiplying and dividing by zero 2007-02-05
From colin:
i would like to if u multiply 1,000,000 by 0 does it equal o or 1.

if you have a number eg 15689 and you divide by o what would the answer be is it 1

Answered by Penny Nom.
Driving on a banked roadway 2007-02-05
From William:
A race track is shaped like an inverted cone. Race cars drive around this track in a horizontal path. If a drivers speed is 34 m/s, at what distance from the tip of the cone should he drive his car if there is no friction?

THANKS IN ADVANCE FOR ANY ASSISTANCE YOU CAN PROVIDE!

Sincerely,
William Bush

Answered by Stephen La Rocque.
The substitution method 2007-01-31
From Victoria:
how do i solve this problem using the substitution method?
2x-5= -14
-7x+14y= -5

Answered by Stephen La Rocque.
The elimination method 2007-01-31
From Addrianna:
x-2y=2
3x-5y=7

Answered by Stephen La Rocque.
The density of an irregular wooden block 2007-01-30
From Carlos:
I'm trying to help my son but I don't really have too much knowledge about this topic. he is been asked to convert 10 milliliter to gram, I think that since 1 g = 1ml the answer will be 10 g also there is another question of how of the density of an irregular wooden block is affected if:
a) the center is hollow:
b) the block is not completely submerged when determining the volume.

Answered by Stephen La Rocque and Penny Nom.
Simplify 18e^2fg/32eg^2 2007-01-30
From Morgan:
how do you simplify 18e^2fg/32eg^2
Answered by Penny Nom.
Height of a right triangle? 2007-01-29
From Engelbert:
The base of the right triangle is 50 ft. At the angle (on the base of the triangle) across from the right angle is labeled portion (a triangle within a triangle), the base of this little triangle is 10 ft. and the height is 6 ft. What equation can i use to solve this?
Answered by Steve La Rocque and Haley Ess.
A curve on a cylinder 2007-01-27
From John:
think of a tube,say 50mm in diameter made out of cardboard, project vertical lines at right angles from the base at say every 2mm right round the tube in pencil.No pretend you can put this cardboard tube in a saw and cut it at 45degrees. Get a pair of scissors and cut it at the lowest end and lay it out flat.It now looks like a graph,how do you work out each of these vertical lengths possibly chord lengths
Answered by Stephen La Rocque and Penny Nom.
The altitude of regular n-side pyramid 2007-01-26
From Wendimu:
The altitude of regular n-side pyramid is half of it's base side.What is the angle between the lateral face and it's base?
Answered by Penny Nom.
Grams and mls 2007-01-26
From Larissa:
Is there a formula to convert grams to mls..?
Answered by Stephen La Rocque.
What is the point of intersection of two sides of a polygon? 2007-01-23
From Tyler:
What is the point of intersection of two sides of a polygon?
Answered by Stephen La Rocque.
Is -2^2 = -4 or 4? 2007-01-23
From Joan:
Is -2 squared, when written without parentheses around the -2, -4 or could this correctly be solved by squaring -2 (-2 x -2) for an answer of 4? Or, to correctly get an answer of 4, would the problem have to read (-2) squared?
Answered by Stephen La Rocque.
Simplify (2x)^4 (1/2x^3)^2 2007-01-21
From Sarah:
Simplify (2x)^4 (1/2x^3)^2
Answered by Penny Nom.
Why zero is neither prime nor composite? 2007-01-17
From Brenda:
Why zero is neither prime nor composite?
Answered by Penny Nom.
In the shadow of a building 2007-01-11
From Bill:
if a building b feet cast a shadow f feet long, then, at the same time of the day, a tree t feet high will cast a shadow, how many feet long?
Answered by Stephen La Rocque.
How do the perimeters of the two pentagons compare? 2007-01-02
From Robert:
If the area of one pentagon is eighty-one times the area of another pentagon, how do the perimeters of the two pentagons compare?
Answered by Penny Nom.
Who invented the division symbol? 2006-12-29
From Alex:
Who invented the division symbol?
Answered by Penny Nom.
6/x-2y - 15/x+y = 0.5 2006-12-21
From Emeka:
6/x-2y - 15/x+y = 0.5
Answered by Stephen La Rocque.
7.2 oz per sq. yard equals ? __ grams per sq. meter 2006-12-20
From Carol:
7.2 oz per sq. yard equals ? __ grams per sq. meter
Answered by Penny Nom.
Splitting A Circle Evenly 2006-12-20
From Joe:
I'm trying to make a game board and instead of having it square, I would like to give it a curve (the game is Parcheesi). The attached diagram is pretty much completed (done in AutoCAD). What I would like to know is how to manually find the points that intersect the red line. In other words, evenly split the semi-circle into 8 pieces.
Answered by Penny Nom.
I want to convert milliliters into grams. 2006-12-14
From Kenchotenzin :
I want to convert milliliters into grams.
Answered by Penny Nom.
Drilling holes in a cube 2006-12-12
From Liz:
What is the volume of the solid remaining if a unit cube has a hole drilled thru one face to the opposite side and the hole that is drilled is a cylinder of unit diameter. Then the cube is rotated and an identical hole is drilled from an untouched face to the opposite side. Then the cube is rotated and an identical hole is drilled between the last two untouched faces. This will result in the cube vertices and the remaining volume of the cube falling into 8 parts. What is the total volume of these 8 parts. How is this volume calculated?
Answered by Penny Nom.
Base 5 2006-12-11
From Shelley:
how do you convert decimal numbers to a given base for example 2,875 to base five?
Answered by Stephen La Rocque.
lim x-->infinity cos x 2006-12-07
From Katie:
I was wondering if it was possible to find: lim x-->infinity cos x
Answered by Stephen La Rocque.
The square footage of an irregular lot 2006-12-07
From Lori:
If I have a rectangular irregular sized lot with the following:

A - 110.53'
B - 159.87'
C - 127.28'
D - 165.86'


How do I determine the square footage?

Answered by Penny Nom.
What is the diameter of the planet? 2006-12-07
From Niall:
In April 2005, scientists claimed to have captured light from an extrasolar planet orbiting a brown dwarf in the constellation Hydra some 230 light-years from Earth. The scientists believe the planet is about five times the mass of Jupiter.

Assume the planet has the same density as Jupiter. What is the diameter of the planet?

Answered by Stephen La Rocque.
Sigma notation 2006-12-05
From Marie:
how do i get this in sigma notation...i only need an explanation 1^2 + (2+3)^2 + (4+5+6)^2 + (7+8+9+10)^2....
Answered by Penny Nom.
solve; 2006-11-28
From Monique:
solve; f(f(x)) = 2x + 4 ; if f(x) = 1
Answered by Penny Nom.
(x - 5)/3 - 3/2 = (4x + 7)/6 2006-11-23
From Terri:
I have never had algebra before and I am finding it difficult.Can you please help me work this problem out?

(x - 5)/3 - 3/2 = (4x + 7)/6

Answered by Penny Nom.
Dimensions of a rectangle 2006-11-22
From Terri:
How can you find the dimensions of a rectangle that has a perimeter of 18 feet?
Answered by Stephen La Rocque.
cos2x=1 2006-11-21
From Christina:
I'm have a hard time solving cos2x=1 for exact values between 0<_x <360
Answered by Stephen La Rocque and Penny Nom.
The height of a building 2006-11-19
From Sweetie:
I have to figure out the measure of the water towers antenna on my schools campus using up to two items created by me and a manual. I don't know far away we are going to have to be. I need to have a fool proof way to figure out the distance to the tower. I have an idea using trigonometry but its really roughly estimated. Do you have any suggestions?
Answered by Stephen La Rocque.
Composition of functions 2006-11-19
From RJ:
Let f0(x) = 2/2-x and fn+1 = f0 o fn for n greater than or equal to 0. Find a formula for fn and prove it by mathematical induction. Recall that o represents function composition. i.e., (f o g)(x) = f(g(x)).
Answered by Stephen La Rocque.
Composition of functions 2006-11-18
From Oryan:
Given f(x)=-2x^3 and g(x)-4x-5, find g(f(-1))
Answered by Stephen La Rocque.
An aircraft and a missile 2006-11-18
From Sarah:
an aircraft is flying at a constant altitude with a constant speed of 600mph. an antiaircraft missile is fired on a straight line perpendicular to the flight path of the aircraft so that it will hit the aircraft at a point P. at that instant the aircraft is 2 miles from the impact point P the missile is 4 miles from P and flying at 1200 mph. at that instant, how rapidly is the distance between missile and aircraft decreasing?
Answered by Stephen La Rocque.
Converting cubic meters to gallons and liters 2006-11-16
From Lorne:
I have a large tank and I would like to know the formula for the conversion. And do you have any other info on web sites that have conversion tools as most of the tools are in u.s and I live in Canada and need the metric conversions.
Answered by Stephen La Rocque.
How to show my work when I simplify this 2 - 2[4 - 9] ÷ (-5)? 2006-11-14
From Remada:
How to show my work when I simplify this 2 - 2[4 - 9] ÷ (-5)
Answered by Penny Nom.
Motorcycle expansion chamber design 2006-11-14
From David:
I'm interested in calculating cone information regarding motorcycle expansion chamber design for example. I guess it's called a truncated cone, from what I've read so far. If I know the center line height, small radius, and large radius of a truncated cone then, how can I calculate the angle (included angle?) the cone forms? I'd like to know the variations of the formula so I can calculate for angle, or length, or one of the diameters if I know the other two measurements.
Answered by Stephen La Rocque.
15g/50g^3 2006-11-12
From Nicole:
How do u simplify the variable expression? 15g/50g^3
Answered by Penny Nom.
Simplifying exprressions 2006-11-06
From Jessica:
-21(a+2b)+14a-9b
Answered by Stephen La Rocque.
Simultaneous equations 2006-11-06
From An other:
e^2y-x+2=0
ln(x+3)-2y-1=0

Answered by Penny Nom.
3x+4y+7x+9+2 2006-11-03
From Chavon:
3x+4y+7x+9+2 I don't get it please help
Answered by Stephen La Rocque.
The dimensions of a rectangle 2006-11-02
From Ben:
A rectangle's length is 8 cm. greater than its width. Its perimeter is 128 cm. Find its dimensions.
Answered by Penny Nom.
3-6+4X3= 2006-10-25
From Tom:
3-6+4X3=
Answered by Penny Nom.
(2/3)^2/(4/2)/1/3 2006-10-25
From Jena:
how do i simplify this problem? (2/3)^2/(4/2)/1/3
Answered by Stephen La Rocque.
how many feet in 200 yards? 2006-10-24
From Audrey:
how many feet in 200 yards?
Answered by Karen McIver.
Simplifying radicals 2006-10-23
From Christin:
i would like for you to solve this problem?? 6 square root 8
Answered by Stephen La Rocque.
Octagonal Stop Sign 2006-10-23
From Phil:
My colleagues and I are in a discussion about how many true "sides" an octagonal stop sign has. I say that it has 10 true sides because of the front back sides of a stop sign (3D octagon). Is this correct terminology or should the front and back "sides" be referred to as faces -- equaling 2?
Answered by Paul Betts, Steve La Rocque and Harley Weston.
how many ounces are in 750 cc's. 2006-10-14
From Not a teacher:
how many ounces are in 750 cc's.
Answered by Stephen La Rocque.
How do you write a variable expression 2006-10-11
From Josh:
How do you write a variable expression for the following:

The amount of money Waldo has if he has $10 more than Joe and

How much weight Kirk can lift if he lifts 30 lbs more than his brother.

Answered by Stephen La Rocque.
Metric conversion 2006-10-08
From Eve:
45.6 microliter to megaliter?
Answered by Stephen La Rocque.
Two problems involving density 2006-10-06
From Patricia:

1. Nick has to ship 650,000 g of substance A to Singapore. The density of substance A is 3.23g/cm^3. If the delivery charge is 0.25 centavo per cm^3,how much does he need to pay?

2.Ann ordered 150 balloons for a birthday. Each balloon has a radius of 20cm. If the density of helium is 0.124g/cm^3,how many grams of helium is needed to meet the order?


Answered by Stephen La Rocque.
Divisibility by 9 and 11 2006-10-04
From Prakai:
can 818991 divisible by 9, or 11?
Answered by Penny Nom.
An expansion and a translation 2006-09-25
From meghan:

Write the equations for f(x) = squareroot(4 - (x - 2)^2) after:

a) a horizontal expansion by a factor of 2
Answer: f(x) = squareroot(4 - (1/2x - 2)^2)
b) a horizontal translation 3 units left
Answer: f(x) = squareroot(4 - (x + 1)^2)
c) the expansion in part a), then the translation in part b)
d) the translation in part b), then the expansion in part a)

I understand how to do a) and b), but I'm not sure what I'm supposed to do for the equations in a specific order (expansion, then translation vs. translation, then expansion).


Answered by Penny Nom.
How do I determine the acreage? 2006-09-20
From Charlene:
I have a survey of property with various points on it; how do I determine the acreage?
Answered by Penny Nom.
How large is the property? 2006-09-20
From Jahn:
a triangular lot has 750 feet on one side of a 90-degree angle and 500 feet on the other side of the angle. how large is the property?
Answered by Penny Nom.
The square footage of my property 2006-09-17
From Jack:
I have been trying to calculate the square footage of my property, without success. The measurements are below. Is there a simple formula that can be used based upon the outside parameters? Or, is there a simple way to calculate the square footage?
Answered by Stephen La Rocque and Penny Nom.
The length of 2 sides of a triangle 2006-09-15
From Lonnie:
I need to know how to figure the length of 2 sides of a triangle, as the following example:

The length of the bottom is 12' and the angles are 45, 45 I need to know how long the other 2 sides must be to get an angle of 90 at the top.
Answered by Stephen La Rocque.

Square roots in a binomial expansion 2006-09-11
From Sydney:
(√x + 5)4 expanded using the binomial theorem
Answered by Penny Nom.
Plotting co-ordinates on a number plane 2006-09-08
From Victor:
I need to plot co-ordinates (0,2), (2,-2), (-2,0), and (0,0) on a number or co-ordinates plane, I am not sure how to do this, and get lost after the first point, can you please help?
Answered by Penny Nom.
Metric Conversions 2006-09-02
From Patricia:
I have a new difficulty, metric conversion when it involves the prefixes. Some problems I need help with are:
Answered by Stephen La Rocque.
Cubic yards of trash 2006-08-23
From Chris:

I am journalist in California trying to convert cubic yards of trash (appliances, furniture, tires and household trash) into the space of a football field.

Is there a formula for this?

For example, how much of an American football field would 45,000 cubic yards of said trash fill up? The typical American football field I am told is 120 yards (110 meters) long by 53 1/2 yards (49 meters) wide

Ideally, I'd like to be about to say this amount of trash would fill up the football field XX-feet deep


Answered by Penny Nom.
Approximately what is the acreage? 2006-08-17
From Lisa:
An ad describing a lot size of 73x150. Approximately what is the acreage?
Answered by Stephen La Rocque.
How fast is the water level rising 2006-08-12
From Erin:
Water runs into a conical tank at the rate of 9ft3/min. The tank stands point down and has a height of 10 ft. and a base radius of 5 ft. How fast is the water level rising when the water is 6 ft. deep? (V=1/3 pi r2 h).
Answered by Penny Nom.
The area of a lot 2006-08-05
From Lyn:
I have no idea how to find the square feet of an uneven polygon. I am in the process of making a land agreement and need to know.
Please tell me how to figure out the footage and acreage of land as follows.
road-front 360'
back 360'
left side 220'
right side 180'

Answered by Penny Nom.
Grams and milliliters 2006-07-27
From Poh:
We would like to know the conversion of 1 gm of cream/powder to ml.
Answered by Stephen La Rocque and Penny Nom.
Converting fractions to decimals 2006-07-25
From Diane:
HOW DO I CONVERT A FRACTION TO A DECIMAL? (E.G,. 8/14 to ? )

And HOW DO I DIVIDE A FRACTION (e.g., 1/14 divided by 3) and then convert that answer to a decimal?

Answered by Penny Nom.
How many miles are there in 30 furlongs? 2006-07-22
From Jennifer:
If there are 8 furlongs in a mile, how many miles are there in 30 furlongs?
Answered by Steve La Rocque and Paul Betts.
The lot is a very odd shape 2006-07-20
From Liz:
I have a square footage question. I'm looking at buying a piece of land and would like to know how to calculate the square footage of it. The lot is a very odd shape and I have attached a drawing of the lot:
Answered by Stephen La Rocque.
The dimensions of a rectangle 2006-07-09
From Lori:
The length of a rectangle is 1 cm more than 3 times its width. If the area of the rectangle is 61 cm2, find the dimensions of the rectangle to the nearest thousandth.
Answered by Penny Nom.
A problem with arc sine 2006-07-07
From Scott:
How to prove arc sin x = arc tan( (x)/√(1-x2))
Answered by Penny Nom.
Two algebra questions 2006-07-02
From Majeedah:

I'm upgrading thru correspondence and haven't been in school for a long while, so I have no class or teacher to explain the basics to me.

I have a couple of questions:

The problem is this:

  1. f(x) = x2 - 2, Find the expression.

    Q f(-x)
    A x2 - 2

    Why is the answer not -x2 - 2?

  2. A relation f, is given by f(x) = x - 2/x. Find the expression.

    Q 2/f(3)
    A 6

    How do you get the answer.


Answered by Stephen La Rocque.
An octagon shape flower bed 2006-06-24
From Brandy:
hello my name is brandy my husband and I would like to build an octagon shape flower bed to put around the tree in the front yard we would hope to have the whole shape about 4 ft-5 ft around the tree. what would be the way to find out how to cut each side to that they fit together equally
Answered by Penny Nom.
A side segment of an octagon 2006-06-24
From Ed:
I would like to know how to find a side segment of an octagon when I know that my octagon is 9 feet from side to side and top to bottom?
Answered by Stephen La Rocque.
A trig problem 2006-06-24
From Greg:
A and B are two towers, B being 4 km due east of A. The true bearings of a flagpole, C, from A and B are α east of north and α west of north respectively. The true bearings of a second flagpole, D, from A and B are (α + β) east of north and (α - β) west of north respectively. Assuming A, B, C, and D are on level ground, and that α = 25, β = 10, find the distance between C and D.
Answered by Penny Nom.
simplify 7x + 4y + 2x - 6y 2006-06-21
From Alan:
simplify 7x + 4y + 2x - 6y
Answered by Paul Betts.
What should the length of the other sides? 2006-06-20
From Kevin:
if a property is basically square for 5 acres and one side is 340 feet what should the length of the other sides?
Answered by Stephen La Rocque.
A 52 cubic foot box 2006-06-14
From Konstanze:
I need to figure out what LXHXW I need to create a 1.5 cubic meter or 52 cubic foot volume--there is an answer in your database that relates to this..but it does not give the formula to go from cubic feet/inches to a measurement for a box.

Empirically I can figure out that 3 x 3 x 3 equals 27 and that 3.5 feet (42") x 3.5 x 3.5 equals 42.87 and 3.75 x 3.75 x 3.75 equals 52.73 which is about 1.5 cubic meters (1cubic meter = 52.971643 cubic feet) i.e a box 45 x 45x 45 "

There must be an easier way.

Answered by Penny Nom.
The chord length of a polygon 2006-06-14
From Krishna:
I have to find out the chord length of a polygon - Tetradecagon ! The Radius of the Circle is 11.5 Cms. The Circle is intersepted by 14 arcs. Then how to find out the chord length?
Answered by Stephen La Rocque.
Fahrenheit and Celsius 2006-06-12
From Doris:
I just know how to do problems with fahrenheit or celcius. Could you show me step by step how to do these? Then I can do it if you can show me each step of the way.
Answered by Stephen La Rocque and Penny Nom.
Designing a garage 2006-06-08
From A builder:
I'm currently designing a garage and came upon this interesting math problem. I've tried using various methods to solve it but have so far been unsuccessful. I've included a picture as its far easier to show you my question than explain it verbally. I realize it could be done by trial and error but i'm looking for a real solution.
Answered by Stephen La Rocque and Penny Nom.
Buying some carpet 2006-06-05
From Russ:
My room is 24 ft. long and 17 ft. wide. I need to know how many square feet are in 1 yd.²
Answered by Penny Nom.
Simultaneous Equations 2006-05-24
From Angie:

Question: solve the equations
2x-3y-z=0
3x-2y+z=-5
x+3y-2z=14

for x,y,z


Answered by Stephen La Rocque.
The sides of a hexagon 2006-05-24
From Emilio:
I need to find the measure of the sides of the hexagon that the area is 58 square units.
Answered by Stephen La Rocque.
The interior angles of a right triangle 2006-05-20
From Greg:
I am wondering if there is a way to figure out the interior angles of a right triangle if we know ONLY the side lengths, and the trick is, we CANNOT use arctangent!
Answered by Leeanne Boehm and Penny Nom.
What are the dimensions of the field? 2006-05-14
From Angie:
The length of fence required to enclose a rectangular field is 3000 metres. What are the dimensions of the field if it is known that the difference between the length and width is 50 metres?
Answered by Penny Nom.
What is 2/3 divided by 5? 2006-05-11
From KH:
What is 2/3 divided by 5?
Answered by Penny Nom.
8-digits numbers with their digits in decreasing order 2006-05-09
From Headmaster:
How many 8-digits number are there with their digits in decreasing order
Answered by Penny Nom.
Cutting a piece of PVC pipe 2006-05-03
From Shonda:
A plumber has a 10' piece of PVC pipe. How many 9/5 foot pieces can be cut from the 10' piece?
Answered by Penny Nom.
Simplifying with negative exponents 2006-05-02
From Kristine:

I am having a hard time understanding factoring and multiplying polynomials. I have a couple of problems I just can't get, can you please help me?

Simplify, do not use negatives in the answer:
(5a-1b-7)(-2a4b2)

and:
(5x2y-7z)(-4xy-3z-4)


Answered by Stephen La Rocque.
How much does 1300cc of liquid weigh? 2006-04-28
From Debra:
How much does 1300cc of liquid weigh, and how is the formula equated?
Answered by Stephen La Rocque.
Odd-man out coin tossing and probabilities 2006-04-27
From Kalyan:
When four people toss fair coins, what is the probability that in a given toss there will be one "odd man". That is one person whose coin does not have the same out comes as that of any other members?
Answered by Stephen La Rocque.
How many gallons in a cubic foot? 2006-04-24
From Jana:
How many gallons in a cubic foot?
Answered by Stephen La Rocque.
What's the difference between percentages and points? 2006-04-21
From Sayed:
What is the difference between percentages and points?
Answered by Stephen La Rocque.
The perimeter of a regular octagon 2006-04-20
From Martin:
I would like to make an octagon out of 2x4 lumber. I know that the lumber needs to be cut at 67.5 degree angles, but how do I determine the length of each piece if I want to make, say, a 2.5 ft diameter octagon?
Answered by Stephen La Rocque.
How much money can I borrow if I have $5 000 for a deposit? 2006-04-19
From Chris:
I have a sum of cash that I want to use as a deposit ($ 5 000) and a lender will lend me 65% of the purchase price. How do I calculate the maximum value I can borrow?
Answered by Stephen La Rocque.
Given three angles and a side 2006-04-09
From Jon:
How do you figure out the length of all sides of a scalene triangle if given the measure of all angles, and one side?
Answered by Stephen La Rocque.
How many people are in the race? 2006-04-07
From Marty:
You're in a race with 1/5 of the racers ahead of you and 5/6 of them behind you. How many people are in the race?
Answered by Penny Nom.
A region described by the lengths of four sides 2006-04-06
From Mark:
I have an area that is 77ft in length at the top, 83ft at the base. One side is 46ft and the other side is 20ft. What will be the square feet for this area?
Answered by Stephen La Rocque.
2x+5y=3 And -x+3y=-7 2006-04-03
From Lloyd:
simplify 2x+5y=3 And -x+3y=-7
Answered by Penny Nom.
The size of a piece of property 2006-04-02
From Nancy:
My name is Nancy and we purchased a piece of property and I am trying to determine whether it is an acre or less. Could you figure this out for me.
Here are the measurements of the property:
The front is 128.02'
The back is 210'
The left side is 220'
The right side is 251.61'
I am very anxious to find out if it is over an acre.
Thank you for any help you can give.

Answered by Penny Nom.
The centre and radius of a circle 2006-04-02
From Kaye:
I need to calculate Dimension E and F. I am given A, B, C, (or over all A+B+C), D, G. The radius is one continuous unknown radius. Example: A = 23.50
B = 35.50
C = 0.50
D = 11.50
G = 23.50
I have calculated this for angles but my mind is drawing a blank for the radius calculation. I can draw it but I need to put into Excel spreadsheet.

Answered by Harley Weston.
Superimposing images 2006-03-30
From Laurie:
I am a parent and trying to find real world applications for the math strand that includes superimposing one item on another to determine identical. The Alberta strand is match size and shape of figures by superimposing one on top of the other. I am trying to find a way to put this into context for everyday life. Any ideas?
Answered by Penny Nom.
Solve the equation cos x = sin 20 where x is acute. 2006-03-26
From Elle:
Solve the equation cos x = sin 20 where x is acute.
Answered by Stephen La Rocque.
The area of a block of land 2006-03-26
From Ronald:

I have a building block of land with four unequal sides and only one right angle. I want to know the total area (in metres) and how the calculations were carried out.

The four sides are: Rear of property: 9.14 metres
left side: 36.9 metres
Right side: 32.61 Metres
front to street: 27.43 Metres

The front to street and right side constitute a right angle. but there are no others.


Answered by Penny Nom.
The volume of water in a cone 2006-03-21
From Ghulam:
A vessel has the shape of an inverted cone.The radius of the top is 8 cm and the height is 20 cm. Water is poured in to a height of x cm.Show that if the volume of the water is V cubic cm,then V=(4/75)pi x3.
Answered by Penny Nom.
Three towns are located at the vertices of an equilateral triangle 2006-03-20
From A student:
three towns are located at the vertices of an equilateral triangle. The towns are 8, 5, and 3 miles, respectively, from a store. How far apart are the towns?
Answered by Chris Fisher.
A proof by contraposition 2006-03-16
From Eban:

1)by mathematical induction prove that 12 + 32 + 52 + ...... + (2k-1)2 = (1/3)k(2k-1)(2k+1) for all positive integers k.

2)show that the contrapositive of the following statement is true. if 1 + M7 is even, then M is odd.


Answered by Stephen La Rocque.
A 3-dimensional star 2006-03-15
From Rachel:
I am trying to figure out the name of a figure that consists of 8 (I think) square pyramids. There is a net drawing of this figure somewhere and I can't find it because I don't know the name. When all 8 pyramids are connected, they form a '3-dimensional star'. We did this project when I was 14 years old and I am now a 27 year old teacher and I would love to do this project with my kids.
Answered by chris Fisher.
2x+3y=0 and 3x-y=0 2006-03-14
From Lisa:
my name is lisa I am doing math for work and i have a math problem to solve and i need help with it here is the question 2x+3y=0 and 3x-y=0 this is one question can you help me please
Answered by Penny Nom.
A fifth degree polynomial 2006-03-13
From Forrest:
When f(x) = x5 - 3x4 - 6x3 + 3ax2 - 24 is divided by x2 - 2 the remainder is bx.
Find the values of a and b.
Hence solve the equation f(x) = -8x

Answered by Penny Nom.
f(x) is a polynomial of degree 3. 2006-03-13
From Meadow:
f(x) is a polynomial of degree 3. It leaves a remainder of 10 and 4 when divided by x + 1 and x - 2 respectively. Given also that f(1) = f(-2) = 0, find the remainder when f(x) is divided by x - 3.
Answered by Penny Nom.
On the blueprint, the scale indicates that 2 cm represents 5 ft. 2006-03-11
From Teresa:
On the blueprint, the scale indicates that 2 cm represents 5 ft. What are the actual dimensions of a room that is 4.8 cm by 6 cm on the blueprint?
Answered by Penny Nom.
What is the height of the pot? 2006-03-10
From Michelle:
If a pot is 11 inches in diameter and holds 5.8 litres what is the height of the pot? - assuming it is a cylinder.
Answered by Stephen La Rocque.
The dimensions of a poster 2006-03-08
From Sonu:
poster measures 150cm by 180cm is enlarged in the ratio 8:5 find the length and breath of the enlarged poster.
Answered by Stephen La Rocque.
Heights and shadows 2006-03-06
From Debra:
A person is 7 ft tall and his shadow is 10 ft tall. using the same info what is the shadow of a person who is 5ft tall
Answered by Stephen La Rocque.
1/infinity and 1/0 2006-03-04
From Evan:
I was thinking the other day when i was in math class that when you divide 1 by say n you'll get 1/n. As the value of n increases the smaller the number you get. So if you divide 1/infinity would that equal zero? And if that is true then would 1/0=infinity be true also?
Answered by Penny Nom.
An account that earns 6% annually 2006-03-01
From Christine:
How much do you have to deposit to an account that earns 6% annually if you want to earn $1200 on the account?
Answered by Stephen La Rocque.
The dimensions of a box 2006-02-23
From Ben:
I need to find the dimensions of a rectangular box. They ask for the areas of the top, side and end. If those areas are 120 square units, 96 square units, and 80 square units respectively, what are the dimensions of the box?
Answered by Stephen La Rocque.
Broken-line graphs and histograms 2006-02-16
From George:

1. What is the main difference between a broken-line graph and a histogram? Both represent continuous variables.

2. What is the correct way to read a multiplication array: x-axis first and then y-axis, other way around or it doesn't matter?


Answered by Penny Nom.
A normal distribution problem 2006-02-15
From Mary:
In a certain normal distribution, find the mean when the standard deviation is 5 and 5.48% of the area lies to the left of 78.
Answered by Penny Nom.
the size of a lot is 300 sqm, what is the square feet conversion? 2006-02-11
From JC:
the size of a lot is 300 sqm, what is the square feet conversion?
Answered by Penny Nom.
sinh(i/2) 2006-02-09
From Louis:
How can you set up an equation to find sinh(i/2)
Answered by Penny Nom.
Simplify the variable expression: 3g^2h over 12gh 2006-02-09
From Andrew:
okay the question says simplify the variable expression: 3g2h over 12gh
how would u do this?

Answered by Penny Nom.
Arithmetic progressions 2006-01-31
From A student:
1)the sum to n terms of a particular series is given by Sn=17n-3n2

a)find an expression for the n term of the series
b)show that the series is an arithmetic progression

2)a particular arithmetic progression has a positive common difference and is such that for any three adjacent terms ,three times the sum of their squares exceeds the square of their sum is 375.Find the common difference


Answered by Penny Nom.
Percentage increase in sound intensity 2006-01-29
From David:
I am doing some emotion response mapping. Being a composer I know how to push the heart beat tempo by a % increase of BPM in my music. I want to know how to calculate an increase in intensity of sound. Now I have 60 decibels. I want an increase of intensity by a factor n% with out going over 156 decibels.
Answered by Penny Nom.
How do you find the angles in a triangle? 2006-01-27
From Keith:
How do you find the angles in a triangle if you know the lengths of the sides?
Answered by Chris Fisher and Penny Nom.
The median 2006-01-27
From Wael:
" median"; what does it mean and how do we calculate it?
Answered by Penny Nom.
Composite triples 2006-01-24
From Laeah:
question 1 Find the smallest integer n such that n+1, n+2,and n+3 are all composites.

question 2 If n = 5! +1, show that n+1, n+2, and n+3 are all composite.

question 3 Find the sequence of 1000 consecutive composite numbers.

Answered by Penny Nom.
One boundary of a pond is parabolic in shape. 2006-01-20
From Glenn:
One boundary of a pond is parabolic in shape. The boundary passes through the points A(-20,45), B(40,40) and E(30,35). The equation of the parabola is of the form y=ax2+bx+c. Find the equation of the parabola and the coordinates of the vertex of the parabola. Any assistance you could provide would be greatly appreciated.
Answered by Penny Nom.
I need to order sand for playgrounds 2006-01-18
From Jody:
I need to order sand for playgrounds. I have their square footage and the various depths of sand needed for each, but sand comes in tons.

What is the formula for converting square footage and depth to tons (for example: 6 inches of sand needed for a 200sq ft area = how many tons?)

Answered by Penny Nom.
Polar to Cartesian coordinates 2006-01-15
From Linden:
How do you convert six units at 30 degrees Polar Coordinate into a Cartesian Coordinate?
Answered by Penny Nom.
12100 sq ft is how many acres? 2006-01-14
From Sydney:
If there are 12100 Sq. Ft in property what would that be acre wise?
Answered by Penny Nom.
Convert to decimal form 2006-01-12
From A student:
I would like to know the full answer on how to turn the fraction 3 3/8 into a decimal please.
Answered by Penny Nom.
How do I calculate feet and inches to sq ft? 2006-01-09
From Rebecca:
How do I calculate feet and inches to sq ft?
Answered by Penny Nom.
How many millimeters are in 12 feet? 2006-01-06
From Hunter:
How many millimeters are in 12 feet?
Answered by Penny Nom.
The average of 4 numbers is 5. 2006-01-02
From Sheri:
The average of 4 numbers is 5. I have 3 of the numbers already- 5,8,4,? How do i find out what the last number to give me the average of 5?
Answered by Penny Nom.
Milliliters to grams 2005-12-29
From Kaitlyn:
If I'm using baking soda, how do I convert milliliters to grams?
Answered by Harley Weston.
Cubic metres to cords 2005-12-22
From Duncan:
I have a quote of $2.95 per cubic metre of roundwood but I need the conversion equation to convert the above to a $ per cord.
Answered by Penny Nom.
Simultaneous Equations 2005-12-21
From Matt:
I have these two equations,
336 = 60a + 10b
and
432 = 84a + 6b
Am I right in saying both a and b are 4.8?

Answered by Penny Nom.
Inclusive definitions 2005-12-14
From Layla:

recently the solvable quandary of 5+5+5=550 came up (the question says that you have to put 1 straight line somewhere in the equation to make it true with out turning the "=" into a "not=" sign).

So two answers were put forward:
545+5=550 (the use of a line converting a + into a 4)
AND
5+5+5(less than or equal to)550

There is currently an argument about the second solution. The disagreement is about whether this sign can be used. One person is arguing that the "less than or equal to" sign defines that the number on the left is in the range 550 and below. The other is saying that since the number (which is clearly defined with no variables) can never equal 550, then the "less than or equal to" sign cannot be used in this case.

Which one is the correct definition?


Answered by Walter Whiteley.
how do you convert ml to oz? 2005-12-05
From Sara:
how do you convert ml to oz?
Answered by Penny Nom.
How do you simplify a fraction if one of the numbers is negative? 2005-12-05
From Stephanie:

Question: How do you simplify a fraction if one of the numbers is negative?

Ex. -32/40


Answered by Penny Nom.
Area of a lot 2005-12-03
From Ben:

How do you calculate the area of the following Lot?

I figured the following angles from the deed, which read:

N 86 degrees, 45 minutes E, for 322 feet.

S 10 degrees, 30 minutes W for 113 feet.

N 84 degrees, 30 minutes W for 368 feet.

N 50 degrees, 42 minutes E for 76 feet.

N 40 degrees, 40 minutes E for 15 feet.

There is a discrepancy between two surveyors and I'd like to figure out how to calculate the Area of such a shape.


Answered by Harley Weston.
A sample size estimation 2005-12-03
From Ivonne:
I have to do a research about the behavior of library users. We are going to apply a survey to a population of 1280 students (Management an Economics students) but of course we have to do it to a sample....I need to know the size of my sample.
Answered by Penny Nom.
Express t as a function of d 2005-11-30
From Matthew:

I have a problem on my homework I cant seem to figure out. Basically it gives me this Linear Regression Equation:

ln t=1.5 ln d-0.8954

and it wants be to transform the equation expressing t as a function of d. For some reason I keep getting it wrong. Anyone got an idea of how to do this?


Answered by Penny Nom.
what is 5'7 2005-11-26
From A student:
what is 5'7" inches in cm?
Answered by Harley Weston.
Resizing a polygon 2005-11-26
From Anthony:
I am currently working on a mapping program in which polygons, rectangles, circles, etc... are rotated, moved, and resized. Recently, I used the rotation conversion formula to rotate the items n degrees/radians by using the point and the center point of the shape. Currently, I am working on resizing the shape (length and/or width) while still trying to maintain the integrity of the shape. I basically want to decrease or increase the y value by 1 in order to resize the length and I want to decrease or increase the x value by 1 in order to resize the width. The corresponding x and y values either increase by 1, stay the same, or decrease by 1. Is it possible to use just the point and center of the shape in order to calculate this resizing? What can I use to accomplish this task?
Answered by Chris Fisher.
Common fractions to decimals 2005-11-25
From A student:
how do u turn 8 over23 into a decimal and how do u turn 8 over 29 into a decimal
Answered by Penny Nom.
Four 25-sided dice 2005-11-24
From dan:
Simply put, I am curious to find the likelyhood of each possibility (4-100) if I were to roll 4 25-sided die. It seems to start off simply, as 1, 4, 10, 20...following the inside diagonal of Pascal's triangle. But as soon as i reach the point where the sum is greater than the number of sides, Pascal's no longer seems to work. I would be greatly appreciated if I could be informed of the proper formula to find this answer, or perhaps if my theory of doing it is wrong.
Answered by Andrei Volodin and Shaun Fallat.
Rational expressions 2005-11-15
From Zach:
I can solve easy problems such as (x/2)+3=2+(3x/4). That is easy because the Lowest Common Denominator is 4. But what really gets me stuck is a problem like this one. (6/x-2) = ( 21/(x-2)(x+2) )+ 1.
Answered by Penny Nom.
111111...111 divided by 3 2005-11-14
From Jesus:
If we divide the number 1111...........1( it has got 2004 digits) by 3. How many zeros will we get?. The answer that I have found in the book is 667 but I do not know how to reached it.
Answered by Penny Nom.
fog 2005-11-12
From Janice:
I am having problem with the (fog) function

(fog) (x). Given f(x)= 2x2 ; g(x)= 3-4x

Answered by Penny Nom.
Cubic feet and cubic yards 2005-11-12
From Mike:
I need to know how to calculate square feet into yards of material. Example: 1320 sq. ft. surface area by 1 in. depth. What does this equal in cubic feet and yards. Thanks for any help you can give me.
Answered by Penny Nom.
-120 divided by -15 2005-11-09
From Joshua:
how do you evaluate -120 divided by (-15)
Answered by Penny Nom.
The sides of an octagon 2005-11-02
From Royce:
I understand there is a simple calculation to determine the sides of an octagon when you know the distance across the parallel flats. something like .447 . can you help?
Answered by Penny Nom.
sin(kx) = x 2005-10-20
From David:
What is an integer value for k so that sin(kx) = x has exactly 2005 solutions? How does one arrive at the answer?
Answered by Harley Weston.
Coefficient of variation 2005-10-19
From Jan:
I am currently teaching the coefficient of variation and am wondering if there are some guidelines as to the interpretation of this statistic. I understand that it measures the variation in a variable relative to the mean - but what is the cut off for "too much" variation expressed in this way???
Answered by Andrei Volodin and Penny Nom.
Divisibility by each of the first ten counting numbers 2005-10-17
From Simon:
determine smallest positive integer that is divisible by each of the first ten counting numbers
Answered by Penny Nom.
The coefficient of thermal expansion for steel 2005-10-14
From Jim:

Is the following statement true?

The coefficient of thermal expansion for steel is 0.00000645in/in/deg. Doesn't sound like much but when you run out the numbers it comes to .405504 ft/mile/deg. Still doesn't sound like much, only about 5". Then multiply by 40 degrees and you get a piece of rail that has grown by 16.22 feet in that one mile. It's not at all unusual for the rail temp to go from say, 40 deg to 80 deg on a spring or fall day. Remember that on a sunny day, the rail temp can be significantly higher than the air temp as well."

I ran the math and came up with an answer closer to 16 inches, instead of 16 feet. Which is closer to being correct?


Answered by Penny Nom.
Simultaneous equations 2005-10-13
From Daniel:
5x + 3y = 22 4x - 7y = -20
Answered by Penny Nom.
Whole numbers 2005-10-10
From Kirk:
Can the sum of two or more whole numbers be less than any of those numbers?
Answered by Penny Nom.
Coefficients, constants and like terms 2005-10-05
From Elizabeth:
In the equation -8y+6ab+7-3ab what are the coefficients; the like terms and constants?
Answered by Penny.
Inches and millimeters 2005-09-30
From A student:
how many inches are there in mm? For example 30x40 mm = how many inches.
Answered by Penny Nom.
2x/x^2-9 - 6/x^2+x-12 2005-09-27
From Emily Ann:
2x/x2-9 - 6/x2+x-12
Answered by Penny Nom.
6x+7=8x-13 2005-09-27
From A student:
show me how to do 6x+7=8x-13
Answered by Penny Nom.
14p - 8 = 22 + 20p 2005-09-27
From Tiffany:
14p - 8 = 22 + 20p
Answered by Penny Nom.
How many ounces is 750 cc's? 2005-09-24
From Michael:
How many ounces is 750 cc's?
Answered by Penny Nom.
The length of the side of an octagon 2005-09-22
From Billy:
How can you find the length of the side of an octagon when all that you know are the long sides of one of the eight perfect isc. triangles inside the octagon that share the same center point?
Answered by Penny Nom.
Isn't 1/3 written as a decimal .33? 2005-09-21
From Lisa:
Isn't 1/3 written as a decimal .33? My son informs me I'm wrong, but doesn't know the answer
Answered by Chris Fisher and Penny Nom.
Converting from base ten to base five 2005-09-21
From Julie:
My sister is home schooled and we are trying to convert from base 10 to base 5. We are trying to teach and we have never had this in school. If we could have some examples of some problems and how to do them we would appreciate this.
Answered by Penny Nom.
A 6ft hexagon around a tree 2005-09-20
From Ray:
I am trying to build a 6ft hexagon around a tree in my front yard. I have asked everyone how do I figure the size of each side. I now have several calculations and many theories but still do not know how much lumber and at what angel to cut and what length. I am simply a senior citizen just trying to create something different in my yard.
Answered by Harley Weston.
A 3-dimensional pie shape 2005-09-17
From Bill:
Your site appeared in my search for the name of a 3-dimensional pie shape. 2-d would be a sector of a circle. As it it curved, I don't believe it is in a the polyhedra family. Can you help me find the mathematical term for it?
Answered by Chris Fisher.
I need to know the measurement of 75grams of spice into milliliters. 2005-09-16
From Cyla:
I don't know if this is possible but I need to know the measurement of 75grams of spice into milliliters. Even an approximation would be great.
Answered by Penny Nom.
Converting from grams to milliliters 2005-09-15
From A student:
What if you don't know the liquid that you are converting from grams to milliliters?
Answered by Penny Nom.
A rational arithmetic expression 2005-09-14
From Vijay:
How to simplify following Grade 9 math problem:

(4/-9) X (-21/-32) X (-3/14)

Answered by Penny Nom.
Adding positive and negative numbers 2005-09-03
From Billy:
46 + -24 =
-38 + -22 =
19 - 20 =

Answered by Penny Nom.
Metric conversion 2005-09-01
From Donna:
I know there is an equation to figuring out, for example. .5526 km = _______ mm
Answered by Penny Nom.
Hat size 2005-08-16
From Keshia:
What formula would I use to find the measurement of a hat size? Ex. If the hat was 7 3/4", what formula would I use to get that measurement?
Answered by Penny Nom.
Area of a lot 2005-08-07
From John:
We hope this copy of our survey map will help you to determine the acreage of our land.
Answered by Harley Weston.
Converting rational numbers 2005-08-07
From Joe:

I am helping my son with his converting rational
numbers in the form of A/B where A and B are integers and
B not equal to zero

I think I am doing this right but I am not sure so
below are the problems and our answers are beside
them, please let me know if these answerers are correct


Answered by Penny Nom.
The period of sin(x) + cos(x) 2005-07-21
From A student:
WHAT IS THE PERIOD OF SIN(X)+COS(X)?
Answered by Penny Nom.
Three prime numbers p,q and r, all greater than 3, form an arithmetic progression: 2005-07-18
From Ladis:
Three prime numbers p,q and r, all greater than 3, form an arithmetic progression: p=p, q=p+d and r= p+2d. Prove that d is divisible by 6.
Answered by Chris Fisher.
The area of a lot 2005-07-17
From John:
A plot of land has the following dimensions: 391 ft. x 757 ft. x 208 ft. x 788 ft. Q: How many acres is this?
Answered by Harley Weston.
The value of n*tan*(180/n) tends to pi 2005-07-11
From Daniel:
I am 14 and i have been given a piece of maths coursework whereby a farmer has to fence off a piece of land as large as possible using 1000m of fence. I already know that the formula for working out the area of any shape of a 1000m perimeter = 5002/ n*tan*(180/n), however, after some research I have found out that as the number of sides (n), tends to infinity, the n*tan*(180/n) tends to pi. Why is this?
Answered by Penny Nom.
An integer x when divided by 5 leaves the remainder 3. 2005-06-18
From A teacher:
An integer x when divided by 5 leaves the remainder 3. Prove that when 4x is divided by 5, 2 is the remainder
Answered by Penny Nom.
The size of a lot 2005-06-09
From Bob:
Is it possible to determine the square footage and acreage of a residential lot that's straight on 3 sides, and triangular on the 4th side?

As shown in the crude sketch below, the two legs of the triangle at the back end of the lot are 61' and 62'.

Answered by Penny.
A triangular lot 2005-05-21
From Linda:
I am looking to purchase a piece of property, and I need some help trying to figure out the square footage of the land. I know that normally if I multiply the length and width I could get my answer. However, this piece of land is an odd shape and has 3 measurements. Can you please help me.
The measurements for this piece of land is:

97.8'
201.2'
139.7'

Answered by Penny Nom.
Divisibility of a^2 + b^2 2005-05-16
From Ampa:
given natural numbers a and b such that a2+b2 is divisible by 21, prove that the same sum of squares is also divisible by 441.
Answered by Penny Nom.
What is my acreage? 2005-05-16
From Vonis:
My property is 65.5 feet by 130 feet. what is my acreage?
Answered by Penny Nom.
Angle of incline 2005-05-15
From Kyle:
What is the degree of incline of a 12 foot plank that goes from 10.5 inches on one end to zero inches on the other?
Answered by Penny Nom.
An isosceles triangle...with 2005-04-23
From Shannon:
If given an isosceles triangle...with
Is there any possible way to do this, without knowing a side, if so, please explain in detail.

Answered by Chris Fisher.
Centimeters, millimeters and milliliters 2005-04-19
From A student:
How many milliliters in a centimeter?
Answered by Penny Nom.
Dimensions of a roof 2005-03-18
From A roofer?:
A right triangle (roof of a house) has a base of 7 feet and a 22 degree angle. What is the height of the roof and what is the hypothenus of the triangle.
Answered by Penny Nom.
Biodiesel 2005-03-08
From William:
My son is doing an demonstration for science class about how to make biodiesel. we are having problems with the conversion. for instance we think there are approx 190 liters in 50 gallons. Is this correct? Also if you needed 3.5 grams of lye per liter, would that be 662g of lye.
Answered by Penny Nom.
Spraying roses 2005-03-03
From A student:
A litre and a half of water has 0.045 grams of herbicide added for spraying roses. How many grams of herbicide per litre is this?
Answered by Penny Nom.
How much of a acre is this? 2005-03-02
From Ken:
my property is 11,914 square ft. How much of a acre is this?
Answered by Penny Nom.
Th dimensions of a garden 2005-02-09
From Rebecca:
Nate's garden has a perimeter of 34 meters and an area of 72 square meters. What are the dimensions of the garden?
Answered by Penny Nom.
Conversion of a recipe 2005-02-06
From Peg:

A Hungarian friend gave me an old, old family recipe and it calls for:

20 dg. sugar

15 dg. filbert

I can't find a conversion table anywhere that explains how to get ounces from "dg." Do you know what the equivalent would be in ounces of dry measurement?


Answered by Harley Weston.
A seating arrangement 2005-02-03
From Kay:
If you have 5 married couples, how many arrangements can be made if they must sit across from each other?
Answered by Penny Nom.
Strategies for teaching operations with positive and negative decimal numbers 2005-02-02
From Erin:
I am wondering if you can suggest any strategies for teaching operations with positive and negative decimal numbers that do not rely on rules that must be memorized?
Answered by Diane Hanson.
'zero divided by one' and 'one divided by zero' 2005-01-31
From Katrina:
Please advise how best to explain the distinction between the solution to 'zero divided by one' and 'one divided by zero'. I remember being told in school that one of them is undefined and the other is zero but I'm having difficulty with calculating and explaining it to some highly curious 10 year olds!
Answered by Claude Tardif and Walter Whiteley.
When dividing a 3-digit number by a 1-digit number... 2005-01-22
From A student:
When dividing a 3-digit number by a 1-digit number, for what divisors can you get a remainder of 8?
Answered by Penny Nom.
The length of a chord 2005-01-13
From A parent:
Does anyone have a formula for calculating the chord length for a segment of a circle when you know the radius and the enclosed angle or radian ?
Answered by Penny Nom.
The tide at a boat dock 2005-01-11
From Abraham:
The tide at a boat dock can be modeled by the equation

y = -2cos(pi/6 t) + 8,

where t is the number of hours past noon and y is the height of the tide, in feet. For how many hours between t=0 and t=12 is the tide at least 7 feet?

Answered by Penny Nom.
Acres 2005-01-08
From Jim:
What size is this lot?
And is there a location on the web where you can plug in lot dimensions to figure acreage?

North = 180ft
South = 210ft
West = 336ft
East = 244 ft

Answered by Harley Weston.
ln(hx+n) 2005-01-07
From Anibal:
if I have the eq. y=hx+n and I want to know ln(y), how do I process ln(hx+n)?
Answered by Penny Nom.
An isosceles triangle 2005-01-03
From Abraham:
The question is,"Triangle ABC is not isosceles.Prove that if altitude BD were drawn, it would not bisect AC."My question is If an altitude is drawn wouldn\'t that mean automatically its isosceles because, In a triangle the sides opposite congruent angles(in this case the right angles)are congruent? What am I thinking wrong?
Answered by Harley Weston.
Six digit numbers with at least one 7 2004-12-27
From Behzad:
How many six-digit numbers contain at least one 7 in their decimal expansion?
Answered by Penny Nom.
Modelling monthly temperature with a cosine 2004-12-25
From Regis:
The average monthly temperature for a location in Ontario as a function of month number can be modelled using the equation y = a cos[k(t + b)] + d.
Answered by Harley Weston.
An arithmetic progression 2004-12-24
From A student:
the 4th and 5th term of an arithmetic progration 47 and 52 respactively find
a)d
b)a1
c)a50

Answered by Penny Nom.
Divisibility by 15 2004-12-19
From Lisa:
My son was asked to find divisiblity rules for 15. We have been unable to find the answer. Does it exist?
Answered by Leeanne Boehm and Denis Hanson.
Select a card from the deck. 2004-12-02
From Heidi:
Select a card from the deck. What is the probability that this card will be red? Show the number of expected outcomes versus the number of total possible outcomes. What type of event does this represent?
Answered by Penny.
1/4 tsp salt = ??? grams 2004-11-16
From Julie:
if i have 1/4 teaspoon of salt how many grams does that equal
Answered by Penny Nom.
A standard deviation test 2004-11-09
From Karen:
How do you compare to see if a sample standard deviation is different than the population standard deviation? I know how to compare means, but not standard deviations.
Answered by Penny Nom.
The sides of a hexagon 2004-11-01
From Timothy:
I have a perfect Hexagon in which i know the measurement across is 10m. Not from point to point, but from the middle of one side directly across to the middle of the other. From that measurement, how can I determine the length of 1 side.
Answered by Penny Nom.
Solving triangles 2004-10-30
From Allen:
Solve the following triangles.

Given

1. B = 20 Degrees, a = 25, b = 16
2. A = 35 Degrees, b = 2, c = 3
3. A = 32 Degrees, C = 44, c = 20

Answered by Harley Weston.
Converting coordinates 2004-10-25
From Allen:
I am trying to help my son with his home, but I don't remember these conversions. Please help.

1. Convert the following coordinates Cartesian x = 7, y = -24

2. Convert the following coordinates Polar r = 4, theta = pi / 3

Answered by Penny Nom.
sin(3A) 2004-10-20
From A student:
Express sin3A in terms of sinA and cosA.
Answered by Penny Nom.
A square footage problem 2004-10-19
From Jerome:
I have a peice of property that is almost a triangle but not exactly about 280 frontage with the side lines at 228 and 237

I was told I had about one acre plus a little, a developer says I have 33,000 sq ft. and I would like to know what the real result is

Answered by Penny Nom.
Divisibility by 7 and 11 2004-10-13
From Tammy:
I'm stuck in class in Yr 7 And I'm finding it hard on our new topic Divisibility! When I try to find out what this means on Internet sites i can not understand the used symbols like algebra and so on. I'm stuck on the divisibility rules for the number 11!
Answered by Penny.
An Octagon 2004-10-10
From A parent:
I am trying to determine what the size of each triangle is of a 12 foot circle consisting of 8 triangles would be. I've gotten as far as the radius being 6 feet, but cannot figure the distance.
Answered by Penny Nom.
Visitor wins or home wins 2004-09-29
From Paavn and Laddi:
I have a math question i am stumped on for one of my classes. The question states that there are 14 baseball games with 2 possible outcomes for each game, visitor win or home win. How many different total combinations are possible from the 14 games?
Answered by Penny Nom.
Grams to milliliters 2004-09-25
From Emily:
I am trying to convert 19.5g into mL assuming that it's water; density=1g/mL. I don't know where to begin.
Answered by Penny Nom.
Rational expressions 2004-09-24
From A student:
In general, I understand rational expressions except when it comes to solving problems such as:

x+y/2x-y - 2x/y-2x or m-4/3m-4 + 3m+2/4-3m

I am confounded by the issue of having to find a common denominator. For example, if I tried to solve these problems by multiplying both denominators they would still be uncommon.

Answered by Claude Tardif.
Driving me crazy 2004-09-18
From Jerry:
This is driving me crazy!!! Someone told me of a number that can be divided EQUALLY by each of the following individual numbers: 1,2,3,4,5,6,7,8,9. what is that number?
Answered by Penny.
Division with and without units 2004-09-17
From Kenneth:
What does it indicate when the divisor has units (dollars) but not the dividend, as in 500/$10.00 or 500 is divided by $10.00? Are the units understood to be there? Does 500/$10.00 make sense?
Answered by Penny Nom.
Pizza for Jack? 2004-09-16
From Grace:
Jack is playing pool with Jim for $1 a game. He has only $2 and decides to play until he goes broke or has $5, at which point he will quit and go out for a pizza with Jim(Dutch treat). Jack knows from past experience that he beats Jim 60% of the time. What is the probability that Jack will get to eat pizza? Hints: Let A be the 6x6 matrix defined by A=[aij], where aij is the probability that Jack will have $(i-1)after one game is he starts with $(j-1). For example, a23 - .40 since there is a 40% probability that Jack will end up with $1 after a game is he starts the game with $2 (If Jack wins 605 of the time, he must lose 40% of the time). Also, for example, a52 = 0 since there is no way jack can have $4 after one game if he had $1 at the beginning of the game. Since Jack will stop if he goes broke or accumulates $5, a11 and a66 are both 1.

Let x0 = [0 0 1 0 0 0 ] transposed, which we interpret as saying that initially Jack has $2 with a probability 1. Then Ax0 will represent the porbability of each amount of money, $0-$5, after one game. What is the probability that Jack will be able to eat pizza by computing Akx0 for large k and finding a limiting value.

Answered by Penny Nom.
A schedule for 6 teams 2004-09-10
From Greg:
I have a schedule I need to make of 6 teams. They must play each other once however no team can be the away team twice in a row. There must be 3 home games and 3 away games for each team.
Answered by Penny Nom.
1+3+5+...+(2n+1) 2004-09-10
From Emma:
Prove that 1+3+5+...+(2n+1)= (n+1)2
Answered by Penny Nom.
A travelling salesman 2004-09-09
From Liz:
A salesman traveled due west from city A to city B. The distance he traveled, that is the ditance from A to B, was X miles. He returned from B to A and found that he had traveled half the distance, X/2 miles. How can that be?
Answered by Chris Fisher and Penny Nom.
Crossing a river 2004-09-09
From Barb:
Nine men and two boys, trekking through the jungle, need to cross a river. They have a small inflatable boat and it's easy enough to row it across the river. The boat, however, can hold no more than one man or the two boys. How can they all get across? (Hint. suppose there was only one man and two boys) Does it make math sense and what would the answer be
Answered by Penny Nom and Claude Tardif.
(2x+5y-6) + (3x-4y+12) 2004-09-04
From GG:
I am a grade 9 student and i do not understand a question. Plz help! OK here it is........ (2x+5y-6) + (3x-4y+12)
Answered by Penny Nom.
Cubic meters 2004-08-31
From Brad:
I have a 10' x 10' x 12" area & need to find the cubic meters, how do I do this?
Answered by Penny Nom.
130 cc. is ? 2004-08-20
From Ayrn:
130 cc. is ? I have to put 130cc. of petroleum in a bucket how much exactly is that? Also 110 cc.? I have to also put 110cc. wood alcohol in a bucket aswell!
Answered by Penny Nom.
A division symbol 2004-08-18
From William:
i was wondering what the mathematical name for this division sign (÷).
Answered by Penny Nom.
Limits and composite functions 2004-08-07
From Sue:
I have two questions, one about a limit and the other about a composite function. If you could help me, I'd really appreciate it.

1. Find the limit:

lim[x->0] (x*csc(x))

I converted csc(x) to cos(x)/sin(x), but I didn't know what to do after that.

2. f(g(x)) = ln(x^2 + 4), f(x) = ln(x^2) and g(x) > 0 for all real x, find g(x):

I'm having trouble with this one because x^2 + 4 isn't a perfect square.

--Sue

Answered by Penny Nom.
Integrating e^sin(x) 2004-08-04
From A student:
I need to know that how to solve the integral " e^sin x",
Answered by Harley Weston.
The law of sines 2004-08-01
From Joy:
How do you solve this? Do you solve this triangle using the law of sines of the law of cosines? (ASA)

A=120DEG. B=40DEG c=35 cm I keep getting different answers.

Answered by Penny Nom.
Water in a cone 2004-07-28
From A student:
A vertically inverted cone( i.e. vertex down) has a radius 7 inches and height 24 inches. Water is filled to one third of its height .Find the ht of water when cone is turned upside down
Answered by Penny Nom.
The third side of a triangle 2004-07-28
From Annette:
How do you find the length of a triangle side if you know two sides?
Answered by Penny Nom.
The p-value 2004-07-25
From Kathy:
Ms. Lisa Monnin is the budget director for the New Process Company. She would like to compare the daily travel expenses for the sales staff and the audit staff. She collected the following sample information.

At the .10 significance level, can she conclude that the mean daily expenses are greater for the sales staff than the audit staff? What is the p-value?

Having problems finding the p-value & unsure of the formula. Kathy

Answered by Penny Nom.
Factoring integers 2004-07-02
From A student:
After looking at all the info I could get about NFS, I still have some questions that are unsolved:

First of all: If someone found an algorithm that has a worst case running time of N*Log(N) to factor an integer n into his divisors, would it be quicker or slower then the number field sieve algorithm?

secondly, what exactly is the time complexity of the Number Field Sieve algorithm, if I would factor an integer n?

Answered by Claude Tardif.
Simplify 2004-06-27
From Tiff:
Simplify:

-3x (x2 + 2)

(x + 2) (x + 3)

Answered by Penny Nom.
An equation with rational terms 2004-06-17
From Louise:
simplify the expression (a2+n2)/2n = a2 + (a2-n2)/2n
Answered by Penny Nom.
The sum of some positive integers 2004-06-07
From A student:
Find the sum of all positive integers not greater than 10000 that are divisible by neither 3 nor 7.
Answered by Penny Nom.
Finding bearings 2004-05-24
From James:
This question is about finding bearings. A boat race starts from point A, goes North to Point B, a distance of 1000 meters. The course is triangular. The bearing from point B to point C is South 70degrees West. The distance from Point B to point C is 1500 meters. Find the course bearing from C to A.
Answered by Penny Nom.
Frieze patterns 2004-05-19
From Christina:
How can I prove that there are only seven unique geometric frieze patterns?
Answered by Penny Nom.
The dimsenions of a room 2004-05-18
From Estella:
I have a room that is rectangular the area is 159.5 feet, the perimeter is 51 feet what would the dimensions of the room.
Answered by Penny Nom.
Angles of elevation and depression 2004-05-18
From Anjum:
what is the difference between an angle of elevation and angle of depression?
Answered by Penny Nom.
A pyramid 2004-05-11
From Quiana:
I am a student and I have the following questions I need to answer for homework:
How many sides, vertices, and edges does a pyramid have?

Answered by Penny Nom.
Three dice 2004-05-10
From A student:
If one has 3, 6 sided dice what is the probability of the numbers that are rolled to total 4 through 10 inclusively?

Subsequent to this, what is the probability to do this consecutively...say 3 times?

Answered by Peny Nom.
3x squared - 27 / x + 3 2004-05-04
From Stef:
3x squared - 27 / x + 3
Answered by Penny Nom.
A 3 digit number divisible by 7 2004-05-03
From A student:
We need to arrange 1,3 and 6 to form a 3 digit number that is divisible by 7.
Answered by Penny Nom.
square root 125 x 16 2004-04-23
From Mike:
how do you simplify a square root of "- square root 125 x16" when x is a variable... i don't really get how to do this....
Answered by Penny Nom.
A probability density function 2004-04-14
From A student:
The label on a bottle of liquid detergent shows contents to be 12 ounces per bottle. The production operation fills the bottle uniformly according to the following probability density function:

f(x) = 8 for 11.975 <= x <= 12.10
and
f(x) = 0 elsewhere

a. What is the probability that a bottle will be filled with 12.02 or more ounces? b. What is the probability that a bottle will be filled between 12 and 12.05 ounces? c. Quality control accepts production that is within .002 ounces of number of ounces shown on the container label. What is the probability that a bottle of this liquid detergent will fail to meet the quality control standard?

Answered by Penny Nom.
Rational expressions 2004-04-06
From Shivani:
I am in 11th grade Honors Algebra II. We are currently learning about adding and subtracting rational expressions. I looked at the other questions that were similar to that of mine, but they weren't quite answering my question. I have been working on many problems to practice and see if I can come up with the correct answer, but I can't, I got like 2 out of every 7 right on my school homework. I get the easy ones, where when you have to find the LCD, it's just the other denominator. But, on others you have to really think about it. Those are hard. Here are some of the problems that I cannot get at all: (x2 - 3x + 1)/(x2 - 4) - (x2 + 2x +4)/(2 - x) - (x - 4) /(x - 2)

The correct answer is: (x3 + 4x2 + 7x + 17)/(x2 - 4)

Answered by Penny Nom.
Algebraic simplification 2004-03-31
From Chris:
Simplifying Exponents

Question

b^5d^2/b^3d^8

Answered by Penny Nom.
425 cc's of a saline solution 2004-03-30
From Julie:
425 cc's of a saline solution is equal to how much in lbs-weight??
Answered by Harley Weston.
A new baby 2004-03-14
From Jessica:
I just had a baby 3weeks ago, he is in the hospital and he can come home when he is 1800 grams, well i was wondering how many pounds that is?
Answered by Penny Nom.
The area of a lot 2004-03-09
From Dave:

If our lot is 192.71 x 138.79 x 158.74 x 103.08

Question is what percentage of an acre is our lot?


Answered by Penny Nom.
A fifteen sided polygon 2004-03-05
From Joann:
WHAT IS THE NAME OF A FIFTEEN SIDED SHAPE?
Answered by Penny Nom.
A geometry problem 2004-03-04
From Jennifer:
I need help with this problem: Square ABCD has side length 2. A semicircle with diameter AB is constructed inside the square, and the tangent to the semicircle from C intersects side AD at E. What is the exact length of CE?o
Answered by Chris Fisher.
Cosine of 35 degrees 2004-03-03
From Jason:
How do you find the exact solution to cosine 35 degrees.
Answered by Chris Fisher.
Rearranging an expression 2004-02-24
From Corey:

use the following formula to answer the question F=9c divided by 5 plus 32

rearange the formula to solve for c
how many degrees fahrenheit would it be it it was:
0 degrees celcius
-10 degrees celcius
-46 degrees celcius


Answered by Penny Nom.
The dimensions of a rectangle 2004-02-21
From Jane:
A rectangular label has an area of 176cm2 and a perimeter of 54cm. Find the dimensions of the label.
Answered by Penny Nom.
Grams and Milliliters 2004-02-17
From Chantal:
We have to do a "thing", and I need to convert grams into mililitres (g into mL). how can I do this?
Answered by Penny Nom.
Digits in the decimal expansion 2004-02-11
From Leslie:
In the decimal expansion of 1/17 what digit is in the 1997th place?
Answered by Penny Nom.
Geometric sequences 2004-02-03
From Alan:
hello, I am a junior in precalculus. we started working on geometric sequences today, it makes perfect sense on how it works. but why is it called that? if you could send me an answer to why geometric sequences have that name, I would be much appreciative.
Answered by Chris Fisher.
Sin(3x), cos(3x) and tan(3x) 2004-01-28
From Jon:
What is the identity for cos3x, sin3x, and tan3x? In class, we learned double angel identities and were asked to find out the identity to these three trig functions. If you can help, please do. Also, i know that the cos4x- sin4x is the same as cos2x. Is cos8x-sin8x = cos2x also true? Thank you.s
Answered by Chris Fisher.
Ounces into cubic centimeters 2004-01-27
From A student:
how do you convert ounces into cubic centimeters? Ex: how does 12 oz. change into cc.
Answered by Penny Nom.
Polynomial division 2004-01-13
From Efrat:

How do we divide the following polynomials?

(x3 + x2 - x + 2) / (x + 2)


Answered by Penny Nom.
The sides of a circle 2004-01-07
From Helena:
My name is Helena and I am 10 years old. On a resent math exam I was asked the question" How many sides does a circle have?" and I wrote down none. The teacher said the answer was one side.
Answered by Chris Fisher.
Laying decorative bricks 2004-01-07
From Duk:
Loren is laying decorative brick along both edges of the 21-meter walkway up to his house. Each brick is 0.26 meters long. He is placing the bricks end to end. How many bricks does he need to do the job?
Answered by Penny Nom.
Business trip 2003-12-19
From Ameer:
A businnessman drives from Washington, D.C., to Boston, a distance of 442 miles, and then makes the return trip. On the way to Boston, he drives 65 miles per hour, taking an 1-hour rest stop during the drive. After finishing his business in Boston, he make the return trip driving at 60 miles per hour and takes a 45-minute rest stop halfway through the trip. Which leg of the journey, Washington, D.C. to Boston, or Boston to Washington, D.C., takes the longer time?
Answered by Penny Nom.
How far can you see? 2003-12-15
From Judy:

How far apart, assuming no obstacles, can two people stand and still see each other?

i know this deals with the curvature of earth, but i can't figure out the formulas involved.


Answered by Chris Fisher.
A snowball melts 2003-12-11
From Adam:
A spherical snowball has diameter 10cm and density 0.75g/cm3. It is placed in a cylindrical can with diameer 12cm. After the snow melts it turns to water with density 1.0g/cm3. What will be the depth of the water in the can (to 1 decimal place)?
Answered by Penny Nom.
1/0 = ?? 2003-12-09
From Lisa:
One of my students asked me a fractions question. She knows that 0/1 equals 0 but does 1/0 also equal 0?
Answered by Walter Whiteley and Penny Nom.
A fish tank 2003-11-26
From Bill:
How do I work out the volume and weight of water in a fish tank that is 36in x 14in x 12in?
Answered by Penny Nom.
Laws of sines and cosines 2003-11-23
From A parent:
On the one side of a stream lines PA= 586.3 feet, PB = 751.6 feet are measures, angle APB being 167 degrees and 36 min. Q is a point on the opposite side of the stream. Angle PAQ=63 degress and 18 min and PBQ=49 degrees and 24 min. Find PQ.
Answered by Penny Nom.
If 1 is neither prime nor composite... 2003-11-19
From Joshua:
if 1 is neither prime nor composite...what is it called?
Answered by Claude Tardif and Penny Nom.
If 1 is neither prime nor composite... 2003-11-19
From Joshua:
if 1 is neither prime nor composite...what is it called?
Answered by Claude Tardif and Penny Nom.
Divisibility by 7 2003-11-14
From A student:
how do you test a number to see if it is divisible by 7 or not?
Answered by Penny Nom.
A least squares line 2003-11-09
From Michelle:
Hooke's Law asserts that the magnitude of the force required to hold a spring is a linear function of the extension e of the spring. That is, f = e0 + ke where k and e0 are constants depending only on the spring. The following data was collected for a spring;

e: 9 , 11 , 12 , 16 , 19
f : 33 , 38 , 43 , 54 , 61

FIND the least square line f= B0 + B1x approximating this data and use it to approximate k.
Answered by Penny Nom.

Divisibility by 2 or 5 or both 2003-10-30
From Abdu:
How many positive integers less than 1,001 are divisible by either 2 or 5 or both?
Answered by Penny Nom.
Two chords 2003-10-07
From Lori:
Chords AB and CD of circle O intersect at E. If AE=4, AB=5, CE=2, Find ED.
Answered by Penny Nom.
0/0 2003-09-25
From Thomas:
How is 0/0 ever defined.
Answered by Penny Nom.
The Sieve of Eratosthenes 2003-09-18
From Lynn:
My daughter has been asked to find all the prime numbers by using the Sieve of Eratosthenes. I have no understanding what this means.
Answered by Penny Nom and Claude Tardif.
Converting from base 10 to base 5 2003-09-10
From Susy:

My son, who is 9 in grade 5 has been asked to convert base 10 numerals into base 5.

His first question of:

24(10) he has calculated to be 4x5 + 4x1 = 44 (5)

The next question however is the tricky one.Ý We know the answer is supposed to be 100 but we find it difficult to get this in the way he understands it.

25 (10) = _________________ 100 (5)

Can you help us figure out how we reach the answer.


Answered by Claude Tardif and Penny Nom.
Making a square 2003-09-07
From A student:
if I am given any number (say 80 for example), how may I determine the smallest whole number integer which when multipled by it will yield a square number ? In other words if I express this as: 80Ý*ÝnÝ=ÝsquareÝnumber, what is the least value of n which will yield a square.
Answered by Penny Nom.
7,473,000,000 divided by 52000 2003-09-06
From A student:
I am having trouble with a question. 7,473,000,000 divided by 52000. Our calculater gets 147311.5385. I come up with 143711.53846, and then the number starts to repeat itself
Answered by Penny Nom.
Positive, negative and zero 2003-09-04
From Brian:
Can the Value of zero be positive or can it be negative or can it be both.
Answered by Penny Nom.
Newton's binomial theorem 2003-08-30
From William:
According to page 126 of Murtha & Willard's "Statistics and Calculus" (Prentice-Hall, 1973), Newton's binomial theorem can proved inductively. I suppose that was his method, which I would like to see.
Answered by Penny Nom.
Converting to base 10 2003-08-28
From Karen:
My son is in 5th grade.Ý He is learning how to convert numbers from base 2 to base 10, and base 5 to base 10.Ý Can you explain it to me in the simplest terms?
Answered by Penny Nom.
Simplify 2003-08-24
From Jessica:
I have been asked to simplify the expression (5a2)(-2b2)(3b)+(7a2)(2b2)+(-3a)(-5a)(4b) and the answer is apparently 44(a2)(b2). For the life of me I can't figure out how they derive this.
Answered by Penny Nom.
39 consecutive natural numbers 2003-08-19
From A student:
Prove that among any 39 consecutive natural numbers it is always possible to find one whose sum of digits is divisible by 11.
Answered by Penny Nom.
Making a blanket 2003-08-13
From Liz:
Please help me figure this out. I am making a blanket the directions call for a piece of material measuring 72"X45". Instead of one piece I'd like to make a patchwork quilt but the squares need to equal 72"X45". Centered on the bottom of the longest edge is an 18" square.
Answered by Harley Weston.
Water in a cone 2003-08-12
From Adrienne:

Water is poured into a tank in the shape of an inverted right circular cone.ð The height of the tank is 8 m and its radius at the top is 4 m.

a. Draw and label a picture to represent this situation.ð (I know how to do this)

b. Identify all variable quantities. (h = 8m, r = 4m)

c. Find an equation that relates the variable quantities, and reduce the number of variable quantities to two.

I was thinking about the equation V = 1/3 pi r2 h, which is the Volume of a cone, but I am stumped as to how I am supposed to "reduce the number of variable quantities to two." Can you point me in the right direction?


Answered by Penny Nom.
The side length of a regular octagon 2003-08-09
From Rich:
If I know the distance between the parallel lines of an octagon and want to find the length of the sides, can I just divide that distance by 2.414 to reveal the length of the sides? I work construction and just curious if I am figuring it right.
Answered by Penny Nom.
Liters in a pool 2003-08-05
From A parent:
I need to know how many litres in a 12 foot diameter, 3 foot deep pool? Any help much appreciated as I need to add the correct amount of chemicals
Answered by Penny Nom.
An onsale DVD player 2003-07-09
From Kim:
I have a DVD player with a regular selling price of $175 and percentage of discount is 10 percent. How to solve for discount amount and sale price? Also, when you figure out the sale price how to figure out commission amount when percentage of commission is 5 percent?
Answered by Penny Nom.
Volume of water in a pool 2003-07-06
From An other:
How many litres of water are in a pool that is 16 feet in diameter and 35 inches deep?
Answered by Penny Nom.
Odd powers of sine and cosine 2003-06-25
From Antonio:
Can you please tell me how to integrate a trig function involving sine and cosine? I know if the powers of both the sine and cosine are even and nonnegative, then I can make repeated use of the power-reducing formulas. But for the question I have on my hand, the powers of both sine and cosine are odd: ( sin3x + cos7x ) dx.
Answered by Harley Weston.
An inch and a millimeter 2003-06-21
From Greg:
The conversion factor for inches to millimeters 25.4 It strikes me as very odd that it's not some infinite decimal. I could understand this if one system was derived from the other but as I understand it they were derived independently. Is the conversion 25.4 or 25.4000XXXXXXX? If it's just 25.4 then how did that happen?
Answered by Harley Weston.
Two trig problems 2003-06-10
From Bett:

I have this ongoing trouble with trig and solving triangles with laws of cosines and sines!! For example if it asks to solve triangle FGH, given angle G=102.7 , side f=14.2, and h=18.6. Now do I use law of cosines because I don't have the measure of an angle and length of the opposite side??I don't know where to go from here,I am totally confused!!!

I also have a problem with this word problem I have been doing. It asks: An airplane flies 847.5 km at a bearing of 237.3 degrees. How far south and west fo its original position is it? Huh? Please help!


Answered by Penny Nom.
How many gallons are in 83 liters? 2003-06-09
From Anita:
How many gallons are in 83 liters?
Answered by Penny Nom.
The metric and Imperial systems 2003-06-08
From Ashlee:
i am a year 8 student, and i am doing my Maths talent quest on the Metric and Imperial number system.

i was wondering, where did the 2 systems orginate and if possible, a little bit or information.

I also need to know the conversions of ounce, foot, mile..etc..from imperial to metric.

Answered by Penny Nom.
The square footage of a section of land 2003-06-04
From Diane:
We are trying to calculate the square footage of a section of land with the following dimensions:

One end is 100 feet. The other end is 105 feet.. One side is 350 feet and the other side is 372 feet. I calculated the area as 35,875 sq. feet, however, the brochure for the land says the area is 36,200 + sq. feet.

Answered by Leeanne Boehm.
Millimeters to inches 2003-05-28
From Brian:
I am a music instructor just trying to get a horn repaired. I have been trying to get replacement pearls for a keyboard of a horn. The mechanic needs the measurement in inches. The calibers say that the size is .115 mm can you tell me what that is in inches, I tried your formula of 2.54 mm per inch but this is a smaller size and I cannot seem to figure it out.
Answered by Penny Nom.
The weight of a fish tank 2003-05-20
From A ponderer:
I had a question pertaining conversion units..I have a fish tank at home and the dimensions are 24, 44, and 48 inches, I want to find out how much it weighs in pounds when filled up with water...knowing water is 62.4 pounds per cubic feet...do I just divide the 62.4 pounds to 12 because I"m trying to find out through cubic inches.
Answered by Penny Nom.
Subtracting rational expressions 2003-05-10
From Simone:
hi, i'm totally lost. i understand that you need to find a lowest common denominator to subtract two fractions (rational expressions) with different denominators. but what if the denominators are "x-1" and "x". is x the common denominator? if so what happens to the "-1"? do you know of any live online help i can get with the following:

3/(x-1) - (1-2x)/x

i've looked through my notes and have no examples that quite match that i can follow to get through it. please help!
Answered by Penny Nom.

sin theta = 7/8 2003-05-07
From Patty:
If sin0 = 7/8 and 0 is in quadrant 2, find the other five trigonometric functions of 0. (report your answers in radical form)
Answered by Penny Nom.
Write sin(3x) in terms of sin(x) 2003-05-05
From A student:
Write sin 2x in terms of sin x
Answered by Penny Nom.
Some shapes 2003-04-29
From Suzanne:
  1. What shape has 12 edges all the same length?

  2. What shape has 3 surfaces and 1 is curved?

Answered by Penny Nom.
Gallons of rain 2003-04-25
From Jocelyn:
If you have a surface of 6500 square feet and 1 inch of rain falls on the surface, how many gallons of water does this equal?
Answered by Penny Nom.
Doubling the size of an object 2003-04-01
From Dave:
If I have a known surface area and volume of an unknown object and I want to double the size of the object, how do I find the new area and volume?
Answered by Walter Whiteley.
Division names 2003-03-10
From A parent:
what is the answer to a division problem called
Answered by Penny Nom.
Can a square be a rhombus? 2003-03-04
From Beth:
Can a square be a rhombus? Some sources say yes, some say no. Some sources define a rhombus as a quadrilateral and parallelogram with equal sides, but without right angles. Some sources say a square is a special case of a rhombus. Clarity, please!
Answered by Walter Whiteley.
Excluded values 2003-02-22
From Josh:
Why do you think it is necessary to include the "excluded values" when you write your answers to rational expressions?
Answered by Penny Nom.
The number of possible outcomes with 8 games 2003-02-21
From Gianni:
If 8 basketball games being played(no ties), which means a total of 16 teams, what are the total number of possible outcomes that can occur.
Answered by Penny Nom.
Density 2003-02-19
From Rob:
What is the formula for converting cubic feet to pounds? (with any substance)
Answered by Penny Nom.
Three consecutive positive intergers 2003-02-09
From Yew:
Prove that when we multiply any consecutive positive intergers, the result is always divisible by 6.

ex. (7)(8)(9) = 504 = 6 (84)

Answered by Penny Nom.
Some 5th grade problems 2003-01-24
From Sabrina:

[What is the greatest common factor of 20,28, and 36?] next problem is

[5k+7(k+8)= ] next problem

[Curtis simplified 0.8 divided by 0.2 x 0.04 and got 10. Latoya Simplified and got 1.6. is either correct? Can you explaine how you got the answer please.]


Answered by Penny Nom.
6 digit numbers from 0,0,2,2,4,4 2003-01-23
From Amanda:
How many 6 digit numbers can you make from the numbers 0,0,2,2,4,4, giving that 0 cannot come first. The number has to contain 2 twos, 2 fours, and 2 zeros.
Answered by Penny Nom and Claude Tardif.
The cousin of Sally's sister's boyfriend 2003-01-23
From Michael:
Sally went to a farm to buy eggs. Returning home, she gave half of them to her sister who, in turn, gave a third of those she had gotten to her boyfriend. The latter, after eating one third of the eggs that he had gotten, gave the rest to his cousin. Given that each egg weighs 70 grams, that Sally cannot carry more than 2.5kg, and that the eggs were raw, calculate how many eggs the cousin of Sally's sister's boyfriend received.
Answered by Penny Nom.
Rational expressions 2003-01-22
From A student:

// = the main fraction line

1.) a+1/a-1 + a-1/a+1 // a+1/a-1 - a-1/a+1

2.) 2/a(squared)-3a+2 + 2/a(squared)-a-2 // 2/a(squared)-1 + 2/ a(squared)+4a+3


Answered by Penny Nom.
Division by zero 2003-01-19
From David:
I've been to your site and am interested on the subject of division by 0. It's easy enough to give answers, but it's very hard, especially in this case, to make someone understand a mathematical problem.

It's been about a year since my Calc class, and we never went over division by 0, only infinite numbers and such. My question is: How would you work a problem of (real number) / 0 ? As ignorant as it may sound to you, I am having trouble grasping this since I've only begun to understand some of it. Perhaps you could help.


Answered by Harley Weston.
A rectangular prism 2003-01-09
From Julie:
How many faces on a rectangular prism and how many bases? Can the base also be a face and can a face also be a base?
Answered by Diane Hanson.
y = 1 - sin(x + 60) 2002-12-10
From Eman:
Sketch the graph of y = 1 - sin(x+60). for 0 <= x<= 360, giving the coordinates of the maximum and minimum points and the pints where the curves crosses the y axis.
Answered by Penny Nom.
The dimensions of a box 2002-12-03
From Paula:
My son, who is in secondary school, needs to make a box with the inside dimensions of 1.25 cubic feet. It's supposed to be more in the shape of a rectangle than a square.

I would like to know the equation to use to find the inside dimensions of a box if I know how many cubic feet (or inches) I need the inside to be.


Answered by Penny Nom.
Performance reviews 2002-11-21
From Lara:
Perhaps you can help.

He's doing performance reviews at work.

The average performance review is 3.92 on a 5.0 scale. This person would receive a 3.5% raise for next year.

The highest performance review is 4.9 and this person would recieve a 6.0% raise for next year.

How do I solve for the other performance review numbers.


Answered by Penny Nom.
Composites, primes, factors and common factors 2002-11-20
From Connie:
My son can't remember what a composite number is, what a prime number is and the explaination of factors and common factors. Help!
Answered by Penny Nom.
abc,abc 2002-11-20
From Pam:
Prove or disprove that "every number of the form abc,abc (where a, b, and c represent digits) is divisible by 7,11, and 13"
Answered by Penny Nom.
Why is 5 divided by 1/7 greater than 5 divided by 2/3? 2002-11-19
From Elizabeth:
  1. Without performing the division, explain why 5 divided by 1/7 is a number greater than 5 divided by 2/3.
  2. Is the least common multiple of two prime numbers always their product? Why or why not?

Answered by Diane Hanson and Penny Nom.
A house's selling price 2002-11-05
From Tanisha:
a CERTAIN REAL-ESTATE AGENT RECEIVES $6 FOR EVERY $100 OF A HOUSE'S SELLING PRICE. HOW MUCH WAS A HOUSE SOLD FOR IF THE AGENT $10,725._
Answered by Penny Nom.
Clown has eight steps to climb 2002-11-05
From A mom:
I'm a parent of a grade two student. What is the best way to help my son figure out a math comprehension problem?

Clown has eight steps to climb, is shown in a picture with front foot on third step and behind foot on first step. Each time he climbs three steps, he falls back two. How many times did he have to climb up to reach the top.

Should I get him to do a line graph or math equations. On his own he came up with the answer seven. I think the answer is six. I did 0+3=3, 3-2=1, 1+3=4 and so on, In think my why is alittle confusing to him. Is there a simplier why to explain it to him.


Answered by Diane Hanson.
Rounding 27.27 2002-11-03
From A parent:
If you have a problem 27.27 and you need to round to the first 7 the answer would be 27

Would 27.0 be acceptable as well or is it completely wrong.


Answered by Penny Nom.
Mathematics and Music 2002-11-01
From Hannah:
I am looking for a science fair project to compare math and music and how they relate. If you have any project ideas for me, they would be greatly appreciated.
Answered by Walter Whiteley.
A coin is tossed 11 times 2002-10-31
From A student:
There are 2 people that are playing a game in which a coin is tossed 11 times. The first player gets a point for a toss of heads. The 2nd player gets a point for a toss of tails. whoever gets 6 points wins. Suppose that so far the first player has 2 points and the second player has 4 points. What is the probability that the first player wins the game?
Answered by Andrei Volodin and Penny Nom.
Simplifying Variable Expressions 2002-10-28
From Erika:
i would like to ask you a mathematical question on Simplifying Variable Expressions. The question is 2(n-4)+3.
Answered by Penny Nom.
Simplify 2002-10-19
From A student:
well i need to simplify this problems out and i cant do it so here is one
 3x^2+6x-45 ------------ = 3x^2+21x+30 

Answered by Penny Nom.
A functional equation 2002-10-14
From Rob:
Let f be a function whose domain is a set of all positive integers and whose range is a subset of the set of all positive integers with these conditions: a) f(n+1)>f(n)

b) f(f(n))=3(n)

Answered by Claude Tardif.
Two problems 2002-10-14
From Eva:

a) How many different equivalence relations can be defined on the set X={a,b,c,d}?

b)Show that 6 divides the product of any 3 consecutive integers. I know it is true that 6 divides the product of any 3 consecutive integers. However, i have problem showing the proof.


Answered by Leeanne Boehm and Penny Nom.
Pythagoras in three dimensions 2002-10-14
From Miki:
A room is 6m long, 5m wide and 3 m high. Find the distance from the corner of the floor to the opposite corner of the celing.
Answered by Peny Nom.
Tree diagrams 2002-10-12
From Denean:
  1. For lunch, students can choose juice or milk; hamburger or pizza; and a banana or apple. How many lunch combinations can the students make?

  2. In an experiment, Monty can use paper towels or cloth. He can choose red, green, or yellow food coloring. How many combinations are there?

Answered by Penny Nom.
Kilometers to miles 2002-10-09
From A student:
how many miles are in 16 kilometers?
Answered by Leeanne Boehm.
Nixon, Jefferson, and Madison 2002-10-08
From Lisa:
The longest-lived US presidents are John Adams(age90), Herbert Hoover (also90), and Harry Truman (88). Behind them are James Madison, Thomas Jefferson, and Richard Nixon. The lattter three men lived a total of 249 years, and their ages at the time of death form consecutive odd integers. For how long did Nixon, Jefferson, and Madison live?
Answered by Penny Nom.
Rational expressions 2002-10-03
From Ashley:
1/x(squared) + 5/xy
Answered by Penny Nom.
7+8+9+...+1000 2002-09-11
From Shirley:
My question is what is the formula for adding up numbers when you don't start with number 1? For example 3 + 4 + 5 + 6 = 18. But how could you arrive at the answer without adding all the numbers?
Answered by Penny Nom.
1/8 in decimal form 2002-09-04
From A student:
explain how you put 1/8 in a decimal form
Answered by Penny Nom.
Divisibility of 5 2002 2002-08-25
From Simon:
I need to ask you a question if 5 2002 and 3 2002 are divisible by 26.
Answered by Penny Nom.
Gallons in a cylindrical tube 2002-08-21
From Heidi:
IF I HAVE A 12" DIA TUBE WITH 1/4" WALL AND 72" IN LENGTH HOW MANY GALLONS OF WATER WILL THIS HOLD
Answered by Penny Nom.
Distance in 3-space 2002-08-16
From David:
The question is: how do I figure out the distance of one object in 3D space to another object in 3D space? I have an object at say x = 5.872, y = 2.876, and z = 7.290; and the other object is at x = 1.129, y = -8.213, and z = -11.127. I have been suggested to use the pythagorean theory on this, but since there are three variables, I don't understand how.
Answered by Penny Nom.
Variable expressions 2002-08-15
From Brittany:
I am just learning variable expressions. My homework is to write a variable expression for each word phrase;

example :

the number of minutes in s seconds
the cost of x cartons of juice at $.75 each
And etc.

I don't understand it

Answered by Penny Nom.
Two equations 2002-07-26
From Derek:
1. 3x + 2y = 4
2. -7x + 2y = 24

finding x and y.


Answered by Penny Nom.
Musical Scales 2002-07-24
From Terence:
Given that there are 12 notes in a musical octave, what is the maximum number of musical scales possible within that octave, if each scale has a minimum of 5 notes and a maximum of 9 and we start all the scales from the same note?

In case you don't know anything about music, a scale is a progression of notes where you start on a specific note and end on that same note an octave higher. There are twelve different notes between these two similar notes. Which notes you choose to play determine the sound of the scale. Anything less than five notes would not make for a very interesting scale. Anything more than nine and you would be playing almost 'every' note in the scale, not leaving much room for distinction in how you organize these notes.

I assume you first have to figure out the maximum number of variations possible in a 5-note scale (with 12 notes at your disposal). Then do the same for a 6-note scale, then a 7-note, then an 8-note, and so on. Then add up the results. How to find this maximum number of variations for each scale size though is what I don't know.


Answered by Leeanne Boehm.
Division by zero 2002-07-18
From James:
Why is division by zero undifined (i have to proof that division by zero is undifined)
Answered by Chris Fisher.
Ounces and cubic centimeters 2002-05-30
From Martin:
How many ounces are in 600cc of liquid?
Answered by Penny Nom.
How far apart are the transmitters? 2002-05-18
From Jeff:
A ship at sea is 70 miles from one transmitter and 130 miles from another. The measurement of the angle between the signals is 130 degrees. How far apart are the transmitters?
Answered by Penny Nom.
On which assignment did I do better 2002-05-17
From Denise:
I have 2 writing assignments in class. The first assignment, which had a mean of 10 and a standard deviation of 2, I got a score of 12. The second assignment had a mean of 18 and an s.d. of 3, I got a 21. I need to know which assignment did I do better, relative to my classmates?
Answered by Andrei Volodin.
Some algebra 2002-05-11
From A student:

/x - 2 ^4/x = 8

/ = square root sign
^ = 4th root of x

also, dividing
2x3 -3 -6x
__________
4 + 2x

the 2x3 is 2x to the third


Answered by Penny Nom.
The law of cosines and obtuse angles 2002-05-09
From Bryant:
The question that I am pondering is that I need to derive the law of cosines for a case in which angle C is an obtuse angle.
Answered by Penny Nom.
Trains and cracker boxes 2002-04-29
From Lindsay:
1)Two trains are 250 miles apart on the same track heading towards eaxh other at 7AM- The first train travels at 55mph, the second at 75mph. At what time would they crash?

2)A cracker box measures 12 by 2 by 18 inches. The company wants to double the amount of crackers, but keep the shapes the same[similar] Find the new length[nearest tenth of an inch]


Answered by Penny Nom.
Conversion of units 2002-04-29
From Matt:
I am having trouble finding a workable equation for the following problem, particularly the conversion of two systems of measurement.

The question states that there are 27 grams of salt in a kilogram of seawater. And it asks how many tonnes (1 tonne = 1000 kilograms) of salt are there in one cubic kilometer of ocean. It also says that the density of seawater is 1.1 g/cm3.

What is the equation and how do I figure out how many kilograms there are in a cubic kilometre?


Answered by Harley Weston.
0/4 = ? 2002-04-28
From Danielle:
I am embarrased to be asking this question, but... is it possible to have a fraction with a zero? For example, 0/4. This does not make sense to me and I do not know what it would be representing, other than nothing! Is it proper to express such a fraction?
Answered by Peny Nom.
Arithmetic progressions 2002-04-24
From David:
I have been searching everywhere for the formula to mathamatical progression.
Answered by Penny Nom.
How many gallons of water in a cubic foot? 2002-04-14
From Richard:
I am tring to find out how many gallons of water in a cubic foot.
Answered by Penny Nom.
n +1, n+2, n+3, and n+4 are all composite 2002-04-09
From Jonathan:
Find the small integer n such that n +1, n+2, n+3, and n+4 are all composite
Answered by Penny Nom.
Composition of functions 2002-04-06
From Yvonne:
In our new text book, the following question occurs: State the domain and range of g(f(x))given that f(x) = -x2 - 4 and g(x) = sqrt(x)

The range of f(x), x<=-4, is the domain of g(x). BUT, there is no solution in the Real numbers for g(f(x))= sqrt(-x2 - 4).

In the solutions it says that this is not a function and therefore does not have a domain or range. Is it a relation? Is it anything?


Answered by Claude Tardif.
Pairs of equations 2002-04-04
From A student:

high school level
student is asking

y=4x x=-4y 
x+y=5 3x+2y=20   

y=x-1 3x-y=4 
x+y=3 2x-3y=-9   

x+5y=4 
3x+15y=-1 

. . . 

Answered by Penny Nom.
Two triangles 2002-04-03
From Scott:
Consider 2 triangles: Triangle PMB and Triangle PLA.

Triangle PLA is contained within Triangle PMB.

Side LA is parallel to Side MB.

Point L is located on Side PM. Point A is located on Side PB.

If the ratio PL:LM = 5, then what is PB:PA ??


Answered by harley Weston.
A trigonometric identity 2002-03-22
From Debby:
I am stuck on a problem and wondering if you can help?? It is: Prove the following:
sec2(X)+csc2(X) = sec2(X)csc2(X)

Answered by Harley Weston.
Interest owing 2002-03-17
From Dean:
I am an investor. If I lend to someone on January 1, 1999, $1000 to be paid back interest at 2% per year (simple interest, ie not compounded), and on July 15, 1999, I lend the same person $1,500 more (total loan is now $2,500) but at 18%, and once again on October 15, 1999, I lend to the same person another $1,500, and the person pays me back the principle($4,000) and interest on January 1,2001: What is the total (principle plus interest) due me? The tougher question is: what was my blended rate of return on my total outlay of money? Is there a formula you can give me so that I can perform my own calculations?
Answered by Harley Weston.
Testing a hypothesis 2002-03-14
From A student:
A large distriutor of cosmetics has kept his outstanding accounts receivable to a mean age of 18 days over the past year. This average is considered a standard by which to measure the efficiency of the credit and collections department. Management wishes to check if receivables in the current month is over standard and will do this at a significance level of 0.50. A random sample of 100 accounts yields an average of 20 days with a standard deviation of 9 days. what should management conclude?
Answered by Andrei Volodin.
My 1978 Chevy has a 350 cubic inch engine 2002-03-11
From Sharon:
If my 1978 Chevy has a 350 cubic inch engine, about what size engine would that be in liter?
Answered by Penny Nom.
The square root of 2 2002-03-05
From Roger:
Does two (2) have a square root or do the numbers just keep going? Are there any other numbers that behave like two when it comes to extracting the square root?
Answered by Penny Nom.
Simplify this expression 2002-02-26
From Francine:
2X power 5 Y power 3 times 18 X power -5 Y power -3 divided by X power 1/3 Y power 1/4.

I came up with
     36 ---------- X 1/3 Y 1/4 
The book says 36 x1/3 y1/4.
I don't get it.

Answered by Penny Nom.
sin 2x = cos 3x 2002-02-25
From Allan:
solve:

sin 2x = cos 3x

Primary question: how do you handle the cos 3x?


Answered by Paul Betts and Chris Fisher.
The substitution method 2002-02-24
From Joe:
whats the answer to this question? 3x+y=11
x+2y=-3

its substitution method i am having alot of trouble figuring it out. send the answer as soon as possible. thank you


Answered by Penny Nom.
An octagon-shaped deck 2002-02-20
From An instructor:
How can you solve for finding the side measurements of an octagon-shaped deck that is 10 feet long and 10 feet wide.
Answered by Penny Nom.
The slopes of the sides of the Great Pyramid 2002-02-09
From Christina:
The Great Pyramid is the largest of the Egyptian pryamids. When it was built, it was 481 feet tall and had a square base with 755-foot sides. The pyramid has two different slopes-one along its sides and the other along its edges. Which slope is steeper?
Answered by Penny Nom.
The number of hidden cubes 2002-02-05
From Katie:
This problem is about finding the number of cubes visible and hidden in a cube.

In a cube that is 3x3, 19 cubes can be seen. 8 are hidden.
In a cube that is 4x4, 37 cubes can be seen. 27 are hidden.
In a cube that is 5x5, 61 cubes can be seen. 64 are hidden.
In a cube that is 6x6, 91 cubes can be seen. 125 are hidden.

The question is:
Explain how you could find the number of small cubes that are visible and hidden in a cube of any size.


Answered by Paul Betts and Penny Nom.
Number conversions and averages 2002-02-03
From A student:
A table has the measurements of 1.6m by 2m, how much material is needed to cover table? I converted to cm then multiplied but got an answer of 32000cm ,so I divide by 100 to get to metres - the answer was 320 which I know is wrong - please explain!!

Also I need to explain in my own words what an 'average' is and I am struggling.


Answered by Penny Nom.
Baseball and sit ups 2002-01-31
From A student:
A baseball team won 8 more games than it lost. If it played 56 games, how many games did it win?

On the first day of practice, Paul did 3 sit ups. On the second day he did 5, and on the the third day he did 8, and on the fourth day he did 12. If this pattern continued, how many sit-ups did ihe do on the sixth day of practice?


Answered by Penny Nom.
The size of a lot 2002-01-26
From Claudia:
I own a piece of property that I need to know the square feet for assessment purposes. The figure they came up with is wrong. They measured from one point to another and halved the sums but that means I own the cul de sac and we don't. My lot is 55 feet wide and one side is 108.96 feet and the other side is 146.04 that extends all the way to a circle. The front of the lot on the cul de sac is stated on the survey like this. 78.21 feet where R=40 feet. This large arc is taken off the size of our land. How many square feet is our lot.
Answered by Harley Weston.
Miles and kilometers 2002-01-22
From Dennis:
I'm in the middle of purchasng a vehicle and it was built in Canada. Therefore it is in kilometers and not in miles. I forgot how many kilometers are in a mile. It registers as 183,049 kilometers.
Answered by Penny Nom and Judi McDonald.
Successive differences 2002-01-20
From Linsey:
what is the rule for working out the formula for a sequence with three lines of difference? eg.
                      
                     1, 3, 7, 15, 29 
1st line differences:  2, 4, 8, 14 
2nd line differences:    2, 4, 6 
3rd line differences:     2, 2 

Answered by Penny Nom.
Linear regression 2002-01-16
From Murray:
If you have a set of coordinates (x[1],y[1]),(x[2],y[2]),...,(x[n],y[n]),find the value of m and b for which SIGMA[from 1 to m=n]AbsoluteValue(y[m]-m*x[m]-b) is at its absolute minimum.
Answered by Harley Weston.
Adding vectors 2002-01-12
From Lena:
how do you add vectors together?
If you are given the length and angles of both vectors and are asked to add/subtract them, how do you do it? I know you are supposed to do the head to tail method, but whenever i try it i get the wrong answer. I need help setting it up.

example: A is 2.7cm, and 60 degrees, B is 1.6cm and 135 degrees, find the magnitude and amplitude


Answered by Penny Nom.
The tangent function 2002-01-12
From Justine:
if you know that sin45degress = cos45degrees, how do you know that tan45degrees = 1?
Answered by Penny Nom.
A simplification problem 2002-01-10
From A Parent:
FIND

(A / B) . (C / D)
where
A = Y squared + 3 . Y cubed
B = Y squared + 4
C = 2 . Y + Y squared
D = Y + 4 Y squared + 3 Y cubed


Answered by Penny Nom.
Nickles, dimes, quarters and fifty cent pieces 2002-01-08
From A parent:
The total for all coins counted is $4,564.50 The last coin added to the pile is a 50 cent piece There are 8 times as many 50 cent pieces as there are quarters There are 6 times as many dimes as nickles How many of each are there?
Answered by Penny Nom.
Simultaneous equations 2001-12-17
From Matthew:
4x + y = 4
2x - 3y = 5

what is x and y


Answered by Penny Nom.
Composition of functions 2001-12-16
From Paula:
  1. if f(x)= 3x-1 and g(x)= 1/2x + 3 find fog(2)

  2. find the values of x for which tanx=0

Answered by Penny Nom.
Algabra 2001-12-13
From Stephanie:
My name is Stephanie. My grade 7 class is doing some strange form of algebra which I've never done nor seen before. We are doing equations like these: 3y - 6 + 7 - 4y = 6y - 6. I don't really understand them and was wondering if you could help me do my homework and explain how to at the same time. It would be really great if you could!!!!! Thank you so much! -Stephanie
  1. -9x - 5 - 8 + x = ?

  2. 5 + 6x - 3y + x + 8y = ?

  3. -x - 3 + 5x + 6y + 8x - 9= ?

Answered by Penny Nom.
Recalling the basic facts 2001-12-12
From A parent:
My son is in 4th grade and is a very bright student. He is in the gifted program and makes straight A's on his report card and has through out school so far. My question is how can I teach him to memorize his basic math facts? He does well in math, but when he is placed in a times situation for completing math fact sheets he freezes up. He can not recall the basic facts when questioned at any other time either. He will calculate the answer in his head, like 6 + 3, but he can't just come out with the answer quickly. How can I help him?
Answered by Claude Tardif.
Division symbol 2001-12-12
From Amanda:
What is the name of the symbole used in a long divison problem that seperates the two factors? It looks like a little house.
Answered by Penny Nom.
Two equations in two unknowns 2001-12-04
From Courtney:
y = 3x + 2
y = 4x - 5

solve for x

Answered by Penny Nom.
Dual spaces 2001-11-26
From Hoda:
In the book "Linear Functional Analysis" from Epstein there is a proof (page 116) of the following theorem:

The dual space of Lp is equal to Lq.

Later, it is stated as a theorem without proof that

for 1 <= p < +infinite, The dual space of lp is equal to lq.


Answered by Doug Farenick.
Matrix 2001-11-21
From Hoda:
I have a question about matrix multiplication; well, actually, matrix division. I am assuming that matrices are divided the same as they are multiplied; that is, row by column. But what happens if you have to divide by zero? How does this affect the resulting matrix?
Answered by Patrick Maidorn and Penny Nom.
Box and Whisker plots 2001-11-19
From Rod:
In our Prealgebra course, we have been studying Box and Whisker plots. Recently, we learned how to decide whether a data point is an outlier or not. The book (Math Thematics, McDougall Littell) gave a process by which we find the interquartile range, then multiply by 1.5. We add this number to the upper quartile, and any points above this are considered to be outliers. We also subtract the number from the lower quartile for the same effect.

My question: where does this 1.5 originate? Is this the standard for locating outliers, or can we choose any number (that seems reasonable, like 2 or 1.8 for example) to multiply with the Interquartile range? If it is a standard, were outliers simply defined via this process, or did statisticians use empirical evidence to suggest that 1.5 is somehow optimal for deciding whether data points are valid or not?


Answered by Penny Nom.
How do you get to 100 by using 6 nines? 2001-11-17
From A student:
How do you get to 100 by using 6 nines?
Answered by Claude Tardif.
Third side of Acute Triangle 2001-11-09
From Don:
I gave this problem to my children, 9th and 12th grade and they are not(me too!) sure of the answer. I am trying to determine the length of the third side of an acute triangle. I know two of the lengths but I do not know the angles. Is there a proper formula to use to find the length of the third side? Thank you.
Answered by Chris Fisher and Penny Nom.
A 3 dimensional 5 pointed star 2001-11-08
From Kent:
I am looking for a formula that will give me a layout for a 3 dimensional 5 pointed star. I want to form it out of sheet metal, using 5 polygons and soldering them at the apex. Can you please help me with this? I would like to be able to give the formula the height of the star from the bottom two points to the top point and also how deep the star is. Thank you very much!
Answered by Judi McDonald.
2=the square root of (2 + the square root of (2 + the square root of (2 +...))) 2001-11-05
From Cynthia:
justify algebreically, that:

2=the square root of 2 + the square root of 2 + the square root of 2 + the square root of 2 + the square root of 2 + and so on, .......


Answered by Penny Nom.
Division by zero 2001-10-31
From Stacy:
I have a question about dividing with zero. When the numerator is zero, the answer is zero. But when the denominator is zero, the answer is undefined. Why? I don't understand
Answered by Claude Tardif.
A sample size problem 2001-10-28
From Charles:
The U.S Transportation Dept. will randomly sample traffic reports to estimate the proportion of accidents involving people over the age of 70. The Dept. has no advance estimate of this proportion. how many reports should the dept select to be atleast 97% confident that the estimate is within .01 of the true proportion?
Answered by Harley Weston.
Negative times negative is positive 2001-10-26
From Mary:
I have a question about adding and multiplying positive and negative numbers. When we add two negative numbers the answer is negative BUT when we multiply two negative numbers the answer is positive. I don't understand. Why?
Answered by Penny Nom.
Metric units 2001-10-25
From Kelly:
i am helping my kids out with school work and don't remember meters,centimeters decimeters and all those. We have to change meters into cm.....and so on..........is there a place on the web to look for some kind of help in these questions
Answered by Penny Nom.
A line in 3 dimensions 2001-10-17
From Murray:
I'm working on a complicated proof and i need the equation for a line in 3 dimensions.
Answered by Claude Tardif.
Dimensions of a frame 2001-10-16
From Rachel:
A rectangular picture frame has a perimiter of 44.2 cm.The width of the frame is seven tenths of its length what are the dimensions of the frame?
Answered by Penny Nom.
Six nines 2001-10-09
From A mom:
My middle schooler (sixth) has to calculate the integers 0-20 using only 6 nines. We have done all but the integer 14. He can not use decimals or double the nine like 99 or 19. the fraction 9/9 is okay. Keep in mind of course the order of operations.
Answered by Claude Tardif.
Simplify the expression 2001-10-08
From Natalia:
simplify expression

4(x-7)+(-5x)=-10


Answered by Leeanne Boehm.
A polygon with five sides 2001-10-03
From A student:
I was asked a question in 5th grade today that goes: i am a polygon with five sides and 2 right angles what do i look like and what is my name
Answered by Penny Nom.
Inverting a function 2001-09-30
From Brandie:
Could you please tell me what is the basic guideline for inverting a function

Example:
S(R)=2PiRal
V(R)=PiR(squared)bl
R(V)=?


Answered by Claude Tardif.
Milliters and grams 2001-09-20
From Betty-jo:
500 ml is what in grams
Answered by Pnny Nom.
The square root of 20 2001-09-18
From Dianna:
How do you simplify a square root? My daughter tells me that the square root of 20 simplified is 5root4
Answered by Penny Nom.
Arithmetic sequences 2001-09-10
From Rachel:
I can't seem to figure out a problem that deals with arithmetic sequencing. This is the question: The 5th term in an arithmetic sequence is 1/2, and the 20th term is 7/8. Find the first three terms of the sequence. I attempted this problem with the formula: An = a + (n-1)d (where the n represents the nth term, a is the first term, and d represents the common difference) I keep getting -9.5 for the first number, and then 3/120 as the common difference between the numbers. But as I have figured it, the sequence is getting greater and greater, and my data does not go with the terms given.
Answered by Penny Nom.
Similar triangles 2001-09-08
From Dave:
I am standing on the bank of a river ( whose banks are parallel here) directly opposite a boathouse, B, on the opposite bank. I walk along the bank of the river past a signpost, S, until I reach a point C distant 60 metres from where I started walking. I then walk away from the bank, at right angles to the bank, until I reach a shady tree at D. Attached to teh tree is a sign stating that this spot is 45 metres from the signpost. C is 36 metres beyond S and B and S are in line from D.

(a) How far did I walk away from the bank of the river??

(b) Calculate the width of the river?


Answered by Penny Nom.
Divisibility rules 2001-09-07
From A student:
Why is it that when you add the digits of a number you can tell what the multiples of that number are. Example: 12131313111,

1+2+1+2+1+3+1+1+1=18,

therefore 12131313111 is divisble by 2, 9, 18, & 3 because those numbers are divisble by 18.

Answered by Penny Nom.
Linear regression 2001-08-01
From A student and a mother:
My mom is taking a correspondence course in Simple Linear Regression and Correlation Analysis and we've been arguing about the relationship between the correlation -rxy and standard estimate of error-Sy.x. I took statistics last year in high school and I remember something about the Sy.x being proportionate to the r. Are they inversely related, directly related, not related, or can only range from 0 to 1.00? Her book doesn't say very much and I believe they are inversely related. She says they're directly related.
Answered by Chris Fisher and Penny Nom.
The best linear model 2001-07-29
From Rebeca:
I am a student (senior) summer hire working for Dept of the Army Resource Management. I've been assisting the Analyst in developing a model to predict training costs. The technique we used was simple linear regression. Now I've been asked for my opinion in choosing the best model I feel is better. I've chosen #1. Garrison Training but I want a second opinion before I give my choice. I've taken basic statistics last year in high school and will probably be taking more this year. Could you look over the information and give me a reason my choice would not be best? The dep variable-y, is Training costs in dolars per day.

ModelInd Variable- Xabrsy.x
1.Garrison Training126.310.9820.9055117.2
2.Local Training212.161.370.7937142.7
3.Man Rig Area116.382.470.6481175.6
4.Major Training346.124.670.7280167.2

Answered by Andrei Volodin.
e^pi > pi^e 2001-07-27
From Dusty:
What book(s) contain a proof that ePi > Pie? I think it might be in Problems in Analysis published by Springer-Verlag but I have not been able to check.
Answered by Chris Fisher.
A trig identity 2001-07-27
From Jeff:
prove this identity and show steps
tan(x/2+pi/4)=secx+tanx

Answered by Harley Weston.
The speed of the boat 2001-07-12
From Sharon:
A motor boat is travelling in a southeasterly direction in water that is flowing from the south at 2km per hour. Show that the speed of the boat is (6 times the square root of 2) km per hour, given that it can travel at 10km per hour in still water.
Answered by Penny Nom.
A confidence interval 2001-06-28
From Murray:
An investigator wants to find out of there are any difference in "skills" between full and part time students. Records show the following:
Student            Mean Score         Std Dev        Number
----------      -----------------     ----------     -----------
Full time              83                   12          45
Part time               70                   15          55
Compute a 95% confidence interval for the difference in mean scores.

Answered by Andrei Volodin.
Securing a plank in four dimensions 2001-06-06
From Becca:
If it takes two nails to secure a plank from rotating on a wall in the third spatial dimension, how many nails would it take if you were attempting to secure the board from rotating in the fourth spatial dimension?
Answered by Walter Whiteley.
Mutually exclusive 2001-06-05
From Marje:
What does the mathmatical term "mutually exclusive" mean. Pleas diagram if possible.
Answered by Penny Nom.
Where is the other $1? 2001-06-05
From Samantha:
3 people go to a motel. The room is $30. They split it $10 each. While in the room the Mgr. tells the clerk the room was only $25, so he gives her $5 to take back to the 3 people. On the way she cant decide how to split the $5 between the three people so she puts $2 in her pocket and gives them $1 each.....Therefore, the room now only cost the three people $9 each, which is $27 plus the $2 the clerk pocketed,making a total of $29.....Where is the other $1?
Answered by Leeanne Boehm.
Common solution 2001-06-02
From Samantha:
  1. Solve for common solution: x+y=6 2x-3y=2

  2. Solve for y in terms of x: 3x-y=4

Answered by Penny Nom.
How many sides does a circle have? 2001-05-16
From Georganne:
How many sides does a circle have?
My son answered "infinate" on a test and was corrected. The school insists the answer is 0.

Answered by Chris Fisher and Denis Hanson.
Toy bricks 2001-05-13
From Sarah:
A regular-sized brick weighs 4 kilograms. How much will a similar toy brick, made of the same material, but all of the same dimensions of which are four times smaller, weigh?

The 300-metre tall Eiffel tower is made of steel and weighs 8 000 000 kg. An exact replica, made of the same material, weighs 1 kg. How high is the replica


Answered by Penny Nom.
Simplifying algebraic expressions 2001-05-12
From A student:
How do you simplify algebra expressions?
Answered by Penny Nom.
The angles in a triangle 2001-05-11
From Nikki:
Find the measure, to the nearest degree, of each angle of a triangle with sides of the given lengths.

26, 35, 40


Answered by Penny Nom.
Marginal Propensity to Consume 2001-05-09
From Esther:
The derivative of the national consumption function C = f(y) is called the ____________________.
Answered by Judi McDonald.
Dividing fractions 2001-05-09
From Rina:
I just wanted to ask if you could help me in math. See I'm having a test soon and its on Dividing Fractions and I just don't get it. My math teacher says that I'll be just fine but I failed my math quiz. I went to ask eric but they could help me so they told me to go to you. So here I am asking you if you could help me.
Answered by Penny Nom.
Neither prime nor composite 2001-05-08
From Marc:
what 2 numbers are neither prime nor combitative?
Answered by Penny Nom.
Adding and subtracting rational expressions 2001-05-03
From Donna:
Adding and subtracting Rational expressions. I am in grade 10 and I am a student here is an example of the questions:

1/(x+1) - 1/(x-1) = ?


Answered by Penny Nom.
Population growth 2001-05-01
From Gina:
Suppose the population of a country increases at a steady rate of 3% a year. If the population is 50 million at a certain time, what will it be 25 years later? Define the recurrence relation that solves this problem.
Answered by Penny Nom.
A sequence defined recursively 2001-05-01
From A student:
A sequence s is defined recursively as follows:

s0=1
s1=2
sk=2sk-2 for all integers

- Compute s2,s3,s4... to guess an explicit formula for the sequence sk.
Answered by Penny Nom.

A sequence of even terms 2001-04-29
From A student:
A sequence c is defined recursively as follows:

c0 = 2
c1 = 4
c2 = 6

ck= 5ck-3 for all integers

Prove that cn is even for all integers.


Answered by Leeanne Boehm and Penny Nom.
Two boats 2001-04-19
From Pat:
Two boats head directly toward each other, one of them traveling 12 miles per hour and the other traveling 17 miles per hour. They begin at a distance of 20 miles from each other. How far apart are they one minute before they collide?
Answered by Penny Nom.
The sides of an octagon 2001-04-13
From Craig:
How do I figure the length of the sides of an octagon when all I know is the diameter (4.375).
Answered by Penny Nom.
Where do the lines y=2x-4 and y=x-1 intesect? 2001-04-06
From Bryce:
solve the following problem by setting them equal to each other. Solve for x and y. Where do the lines y=2x-4 and y=x-1 intesect?
Answered by Penny Nom.
The unit circle and trigonometry 2001-04-05
From Ashley:
"My teacher wants us to find out what a unit circle is, which I found out, a circle with the radius of 1, but the problem is he wants us to show the relationship between the unit circle and the sine(30,45,60 degrees), cosine(30,45,60 degrees),and tangent ratios(30,45,60 degrees). I need help with this and my teacher will not help us out. Thanks very much ...
Answered by Penny Nom.
Sigma 2001-03-31
From A student:
What does the Sigma symbol mean in mathematics?
Answered by Penny Nom.
A box with an interior of 1.25 cubic feet 2001-03-26
From Gabe:
I need a box with an interior of 1.25 cubic feet. I am using 3/4 inch material to make the box. What would the exterior dimensions be in inches?
Answered by Harley Weston.
A suspension bridge 2001-03-24
From Janna:
The cables of a suspension bridge hang in a curve which approximates a parabola. The road bed passed through the vertex. If the supporting towers are 720m apart and 60m high, find:

a) an equation of the parabola (it's y = 1/2160x2)

b)the height of the cables at a point 30m from the vertex. I substituted 30 in for the x value and got 0.42 and the answer is 42. What did I do wrong?
Answered by Denis Hanson and Claude Tardif.

Two problems 2001-03-23
From James:
Casey is a finanical consultant for ABC finance. She earns $456.21 weekly. Her annual group medical coverage costs $1942, of which ABC pays 65%. How much is deducted weekly for medical coverage?

and:

Eleanor demonstrates microwaves at the national home show. She is paid $7 for each of the first 7 demonstrations and $11 for each demonstration after 7. What is her commission for a day if she makes 12 demonstrations?

Answered by Penny Nom.
A famous landmark 2001-03-23
From Corinne:
A family is traveling due west on a road that passes a famous landmark. At a given time the bearing to the landmark is N 62 degrees W, and after the family travels 5 miles farther the bearing is N 38 degrees W. What is the closest the family will come to the landmark while on the road?
Answered by Harley Weston.
Why exactly can't you divide fractions? 2001-03-22
From Dennis:
Why exactly can't you divide fractions?
Answered by Penny Nom.
The repetend in repeating decimals 2001-03-21
From Sharon:
What is the name for the bar over the repetend in repeating decimals? Also, what is the name of the long division "house"?
Answered by Penny Nom.
Systems of equations 2001-03-16
From joy:
How do u solve problems using systems of equations?
~ finding x and y~

ex:

26 = 3x - 2y
42 = 4x + y


Answered by Penny Nom.
The substitution method 2001-03-05
From A student:
Solve each system of equations by the substitution method. Show your work.
  1. y = 8
    7x = 1 - y

  2. y = x - 1
    4x - y = 19

Answered by Penny Nom.
How tall is the tree? 2001-03-02
From Ronda:
a tree's shadow is 42 ft. long. There is a stop sign that is right next to it and it is 18 ft. tall and it's shadow is 12 ft. long. How tall is the tree?
Answered by Penny Nom.
Converting percents to decimals 2001-03-01
From Elliot:
My math question is about Converting Percents to Decimals, here it is-

Convert 37 1/2 = ?


Answered by Penny Nom.
The law of cosines in the real world 2001-02-21
From Hope:
Do you have any examles and/or labs that show how the law of cosines is used in the real world?
Answered by Harley Weston.
Law of cosines 2001-02-20
From Emily:
I missed a few days of class and I can't figure out how to solve Law of Cosines problems. I have the notes and formulas but I can't figure out how to do the math involved to answer the problems. I am also getting confused about how to use degrees and seconds in the Law of Sine and Cosine. I can't seem to get the correct answers and I don't really know how to solve them and enter them into my TI 86.
Answered by Harley Weston.
Dividing polynomials 2001-02-19
From Janna:
I have two questions involving dividing polynomials by polynomials.

Here's the first one:

Two factors of 12a4 -39a2 + 8a - 8a3 + 12 are a - 2 and 2a + 1. Find the other factors. The other question I'm stuck on is: When 10x3 + mx2 - x + 10 is divided by 5x-3, the quotient is 2x2 + nx - 2 and the remainder is 4. Find the values of m and n.


Answered by Penny Nom.
Working on commission 2001-02-17
From Jay:
Ron the realtor is offered a job directly out of real-estate school. He has a choice as to which way he will receive his salary the first year.

Salary Plan 1: He would receive a base pay of $2000 per month plus a 3% commission on each sale.

Salary Plan 2: No base pay but a 6% commission on each sale.


Write an equation to determine when it would be better to switch from the first plan to the second plan. Give a one- or two-sentence answer that includes Ron`s sales in a month. Round to the nearest dollar, if necessary.

Answered by Leeanne Boehm and Penny Nom.
A four dimensional object 2001-02-14
From A student:
Can you give me some examples of a four dimensional object you can find around your house?
Answered by Harley Weston.
An order relation 2001-02-12
From Amy:
What is an order relation?
Answered by Claude Tardif.
The "goesinta" box 2001-02-07
From Bridget:
We are looking for the "proper" name of the "box" that is drawn around the dividend on a division problem:

  ___ 3)12 

It has always been a family joke to call it the "goesinta" box, but a search of several reference sources hasn't yielded an answer --

This is a question of curiosity rather than a question for homework.


Answered by Claude Tardif.
23 + 17 * 2 / 8 + 11 = 13 2001-02-02
From David:
My child is working through basic operations, and gave this problem:

23 + 17 * 2 / 8 + 11 = 13

The teacher wants the parenthesis to be put in the right places to come up with 13.
Answered by Claude Tardif.

Finding roots 2001-02-01
From A student:
My math problem is right now we are working on roots. I don't quite understand how to find the answer to the problems, i was wondering what is the easiest, and fastest way to find the answers to roots?
Answered by Penny Nom and Claude Tardif.
Geometric and arithmetic sequences 2001-01-26
From Garry:
what are the equations for geometric and arithmetic sequences?

also, what are the equations for finding the sums of those series?


Answered by Leeanne Boehm and Penny Nom.
Processing speed 2001-01-26
From Zac:
A COMPUTOR IS ADVERTISED AS HAVING A PROCESSING SPEED OF 11 MILLION INSTRUCTIONS PER SECOND. ON THE AVERAGE, HOW LONG DOSE IT TAKE TO PROCESS ONE INSTRUCTION AT SUCH A SPEED?
Answered by Leeanne Boehm.
Rotation 2001-01-25
From A student:
If I have a rectangle 2inches long and 4 inches wide and I rotate the rectangle 45 degrees will the length still be 2 inches?
Answered by Penny Nom.
Conversions 2001-01-24
From Tanya:
I have 2 questions
  1. covert 628 kilometers to miles.(round off your answer to two decimal places.)

  2. a new clerk in your office is to earn $200 per week. If she works 30.5 hours each week, what is her hourly rate? (round off your answer to the nearest hundreth, that is, to the nearest cent.)

Answered by Penny Nom.
Building a circular silo 2001-01-24
From Natasha:
We wish to build a circular silo with internal diameter 10 feet. How much concrete will we need to pour the foundation, if we only need a 1 foot wide and 1 foot deep ring on which the silo walls will sit? Assume the 4 inch thick silo wall rests on the middle of the ring.
Answered by Penny Nom.
Compression by a factor of ? 2001-01-12
From Wayne:
Given an equation like y = (1/2) (x-3)2 + 6, when describing the distortion from the base curve y = x2 is it correct to say there is a vertical compression by a factor of 2, or by a factor of 1/2 ? Which is more correct? I have seen both versions in different high school texts, and I am now unsure which is more correct.
Answered by Harley Weston.
cot(arcsin 3/5) 2001-01-07
From Jason:
Find value. Assume that all angles are in Quadrant 1.

cot(arcsin 3/5)


Answered by Harley Weston.
The city transit system 2001-01-07
From Jacky:
The city transit system carries an average of 9450 bus riders a day, for a fare of $1.75 each. The city wants to reduce car pollution by increasing ridership and to maximize the transit system's revenue at the same time. A survey indicates that the number of riders will increase by 150 for every $0.05 decrease in fare.
Answered by Harley Weston.
Derfs, Enajs and Sivads 2001-01-07
From John and Norman:
All Derfs are Enajs. One-third of all Enajs are Derfs. Half of all Sivads are Enajs. One Sivad is a Derf. Eight Sivads are Enajs. The number of Enajs is 90. How many Enajs are neither Derf nor Sivad?
Answered by Penny Nom.
The laws of sines and cosines 2001-01-02
From Faydene:
Can the sine /cosine rule be applied to a right -angle triangle to find a particular solution or are these 'rules' applied only when the triangle is not right angled?
Answered by Penny Nom.
A lampost and its shadow 2000-12-24
From Laura:
A lamppost line EC casts a shadow line AC. A 30 cm ruler line DB has been moved from A so that it's shadow falls just within the shadow of the lamppost.
  1. Suppose the length of the ruler's shadow is 42 cm. What is the slope of the imaginary line AE?

  2. Suppose the lamppost's shadow is 15 m. long. How tall is the lamppost?

Answered by Penny Nom.
What is the difference between Mean and Average? 2000-12-13
From Julie:
What is the difference between Mean and Average? My thinking is the average, is the equal to the sum of all numbers divided by the number of numbers added together. But the mean, I think should be calculated by adding the largest and smallest numbers in the set and them dividing by 2. (which is the point where 1/2 the numbers are higher and 1/2 the numbers are lower)
Answered by Patrick Maidorn and Penny Nom.
Divisibility by 16 2000-12-12
From Shiling:
A number can be divided by 16 if and only if its 1st four digits can be divided by 16. How can you prove that?
Answered by Penny Nom.
Optical illusions 2000-12-06
From Jessica:
Hi, my name is Jessica, 7th grade, and Im doing a Math Fair project on optical illusions. As one of the required factors, we need a "mathmatical significance" paragraph. Unfortunatley, I can only think of one way that optical illusions have to do with math, and thats time because some optical illusions tell you to look at the picture for a certain amount of time.
Answered by Penny Nom.
The aspect ratio of a rectangle 2000-12-04
From Ron Delavigne:
The aspect ratio of this rectangle is 4:3. That is A to B is 3. And B to C is 4. If I know the lenght of A to C is 19 inches, how can I find the length of A to B, and B to C.
Answered by Penny Nom.
Triangles and trigonometry 2000-11-30
From Mose:
If I have a right triangle, and I know the lengths of all three sides, is there a formula that will allow me to determine the measurements of the 2 non right angles?
Answered by Harley Weston.
Trig identity crisis 2000-11-29
From Rhiannon:
I have tried many times to find the answer to these problems but I can't I am in grade 12
  1. tan(x)=csc2(x)-cot2(x)
  2. cos(x)/csc(x)-2sin(x)=tan(x)/1-tan(x)
  3. cos(x)[ tan2(x)1-1]/cos2(x)+sin2(x)=sec(x)

Answered by Harley Weston.
Vector Problem 2000-11-27
From Ben:
An aircraft can fly 260km/h in still air and the wind is blowing at 70km/h towards the West. In what direction should the aircraft head so that its actual velocity is on a bearing of 030 degrees?
Answered by Harley Weston.
Order of operations 2000-11-26
From Margaret Pratt:
My daughter has a math question and I am afraid I am of no help. Can you help? 2x5/2+1-5= She arrives at 8 as the answer and has been told this is incorrect. Any help you can give would be appreciated.
Answered by Penny Nom.
Bridges and parabolas 2000-11-18
From Lauren:
My name is Lauren, and Im a secondary school student in Ontario. For my gr11 advanced math class I have to find out how and why parabolics are used in arch bridges and write 3 paragraphs on it. People who cohse satelites and whatnot are lucky - I've found a ton of info, but for arch bridges there seems to be nothing.
Answered by Harley Weston.
Find the 3D angle 2000-11-18
From Jacky:
Included is the diagram. I am trying to find out the angle of ABC. Is it possible? How?
Answered by Penny Nom.
Dividing with decimals 2000-11-15
From Alex:
I teach math and my class and I were discussing the dividing of decimals. I explained that if there is a decimal in the divisor, it needs to be moved and so does the decimal in the dividend. My question is why is it necessary to move the decimal point in the divisor before dividing?
Answered by Penny Nom.
Static electricity on an airplane 2000-11-12
From John:
What discharges an airplane of static electricity in a electric storm when it is flying. I assume the charge is on the out side of the plane but when does it get discharged, or does it occur at landing?
Answered by Ted Mathie.
Crown molding mitre cuts 2000-11-06
From Jim Tomfohrde:
My question has to do with making mitre cuts when installing crown molding. Crown molding is the trim that is put up at the top of walls with one edge on the wall and the other edge on the ceiling. To make a mitre cut on your mitre saw for a 90-degree corner you can lay the molding flat on the saw base, set the bevel of the blade to 34 degrees and the mitre to 31.5 degrees (these may be slightly appoximate). Of course depending on which piece of molding you're cutting you will cut one end or the other, or use the left or right end. These angles allow the cuts to line up and form a seamless corner when they're put in place on the wall/ceiling at 90 degrees. My question is this - is there some mathematical formula from which the 34 degrees and 31.5 degrees are derived. I want to know this because in many cases the corner is not 90 degrees but can be more or less, and in these instances I would like to know if I can calculate the bevel and mitre to use based on the angle of the corner.
Answered by Harley Weston.
Length of a shadow 2000-11-01
From Jessy:
A man who is six feet tall is walking away from a street light that is fifteen feet tall. How long is the man's shadow when he is ten feet away from the light?
Answered by Penny Nom.
False Positives 2000-10-29
From James:
A rare disease infected 1 in 1000 people in the population. A test for the disease is accurate 99% of the time when given to an infected person and also when given to a heathy person.
  1. Fill out a two-tier tree diagram and find the probability of the false positive(i.e the conditional probabily of being healty even when tested postive by the test) Comment on the result?

  2. ...

Answered by Penny Nom.
Square roots 2000-10-29
From Pamela:
HERE GOES(I WILL USE Q AS THE SYMBOL FOR SQUARE ROOT):

8(Q2) - 5(Q2) + Q2

SECOND PROBELM IS

(1 + Q2)2

LAST ONE

3/(2-Q5)


Answered by Claude Tardif.
Divisibility by 9 2000-10-24
From Kelera:
If the sum of the digits of a number is divisible by 9, then the number itself it divisible by 9. Why is that? How do you explain this?
Answered by Penny Nom.
Tapes and CD's 2000-10-22
From Arnold:
Nicole has 25 CDs and tapes altogether. When she tries to pair them up, she has 4 CDs left over. How many tapes does Nicole have?
Answered by Penny Nom.
Dividing fractions 2000-10-18
From Paula:
Why do you have to change the division sign to a multiplication sign and invert the fraction that follows the division sign in order to get the answer to a division problem when you're working with fractions?
Answered by Penny Nom.
Hendecagon 2000-10-09
From Jillian:
I need an example of a object that is in the shape of a hendecagon. I know what the shape is but I cannot come up with a real life example of an object that is this shape.
Answered by Chris Fisher.
Mode 2000-09-22
From James Barton:
I have always been told that a mode is the "one" number that appears most in the set of numbers: ex.{1,3,4,6,3,2} the mode is 3. What if you have {1,1,3,4,5,5}is there a mode. I was taught long ago that there is no mode, Not i am having to teach there is two modes. 1 and 5. If this is the case if we have {1,1,2,2,3,3,4,4,5,5} that every number is the mode. True or false. This is being ambigiuous if we say all are the mode. Because no one number is used more than the others.
Answered by Claue Tardif and Harley Weston.
A cycloid in Cartesian form 2000-09-20
From Billy:
The parametric equation of cycloid is given:
x=r(t-sint)
y=r(1-cost)

How to eliminate t?


Answered by Harley Weston.
Two linear equations 2000-09-14
From David Dean:
2a + 1b = 3.39 3a + 3b = 6.59
What formula do I use to find what a = ?

Answered by Harley Weston.
7 - - - - 77 2000-09-13
From Peter:
Does anyone know how to solve the following:
7 _ _ _ _ 77 ?

I have to find the missing values.


Answered by Chris Fisher and Walter Whiteley.
Cubic yards and cubic feet 2000-08-26
From Rhonda:
have 2.47 cubic yards. what could the dimensions, in feet, be to equal this?
Answered by Penny Nom.
Six typists 2000-08-19
From Paula Duncan:
If it takes a typist 4.5 hours to type 860 words, how many hours would it take 6 typist to do 16800 words?
Answered by Penny Nom.
Some trigonometry 2000-08-11
From Angela:
I have some PreCal questions. I am a student at the secondary level. I would be very grateful for your help.

Solve the equation for theta (0 <= theta < 2pi).

tan2(theta) = 3 I know sec2(theta) -1 = tan2(theta)

.
.
.
Answered by Harley Weston.

LOG(LN(x)) = 1 2000-07-28
From An algebra student:
LET F(x)=LOG X AND G(x)= LN X. SOLVE (f *G)(x)= 1
SHOW COMPOSITION AND USE DEFINITION OF LOGS.

Answered by Harley Weston.
(mn + m + n)/(m^2 + n^2) = 1 2000-07-24
From William:
Find all pair of positive integers m and n such that mn + m + n divides m2 + n2 = 1.
Answered by Peny Nom.
Simplify 21+2[3z+5(3z+8)] 2000-07-02
From Dennis Dyer:
My daughter has a math problem that I can't help her with. The problem is 21+2[3z+5(3z+8)] I can get the answer 101 +36z but I can't show her the correct way to write it out. Would you please show the correct way to show her work...
Answered by Penny Nom.
Central Limit Theorem and Law of Large Numbers 2000-06-26
From Jonathan Yam:
The Central limit Theorem states that when sample size tends to infinity, the sample mean will be normally distributed. The Law of Large Number states that when sample size tends to infinity, the sample mean equals to population mean. Is the two statements contradictory?
Answered by Paul Betts and Harley Weston.
Simplifing 2000-06-15
From Angie Herbert:
My daughter is having problems with her algebra homework. She is a year 7 pupil at high school in the UK. She has been given homework on simplifying formulae and she doesn't understand how to do them. Here is an example of one of the questions :

9s+7t+5c+5s-3t

can you help her to understand how to do these and perhaps me too.
Answered by Penny Nom and Claude Tardif.

Using the inverse sine function 2000-05-31
From Nelson Rothermel:
This has me completely baffled. I have to use the laws of sine or cosine to find the angles of a triangle when I have 3 sides, so I can't go 180-x-y when I have 2 angles. Now, I have a triangle with values of 3, 7, and 9. Here are the steps I used (A,B,C are angles; a,b,c are opposite sides):

angle A (16.1951 degrees): cos-1*((b2+c2-a2)/(2*b*c))
angle B (40.6011 degrees): sin-1*(b*sin(A)/a)
angle C (56.7962 degrees): sin-1*(c*sin(A)/a)

If you notice, A+B+C does not equal 180. According to the book, A and B are correct, but C is supposed to be 123.2038 degrees. Why doesn't it work???


Answered by Harley Weston.
Why does division start from the left? 2000-05-24
From Salil Dave:
Addition, subtraction and multiplications start with right most digit and proceed left, but division starts from left-most digits and goes right ... why?
Answered by Harley Weston.
Six letter words 2000-05-15
From Karl Freitag:
An anthroplogist discovers an isolated tribe whose written alphabet contains only six letters (call the letters A,B,C,D,E, and F). The tribe has a taboo against using the same letter twice in the same word. It is never done. If each different sequence of letters constitutes a different word in the language, what is the maximum number of six-letter words that the language can employ?
Answered by Penny Nom.
Multiplying fractions 2000-05-14
From A student:
75/2 x 1/100 = 3/8

Please show how to came up with the answer?
Answered by Penny Nom.

Solve 2sin 3x-1=0 2000-05-11
From Cynthia:
How would you solve 2sin 3x-1=0? I don't know what to do with the 3.
Answered by Penny Nom.
sin(7pi/12) 2000-05-04
From Kristel:
What is the exact value of sin 7pi/12?
Answered by Chris Fisher and Paul Betts.
Graphing cube roots 2000-04-27
From Heather Jones:
What do the graphs of cube roots look like?
Answered by Walter Whiteley.
Paying with silver 2000-04-26
From Luther Jackson:
A silver prospector is unable to pay his March rent in advance. He owned a bar of pure silver, 31 inches long, so he made the following arrangement with his landlady. He would cut the bar, he said into smaller pieces. On the first day of March he would give her and inch of the bar, and on each succeeding day he would add another inch to her amount of silver. She would keep this silver as security. At the end of the month, when the prospector expected to be able to pay his rent in full, she would return the pieces to him.
.
.
.

Answered by Claude Tardif and Penny Nom.
Conversions 2000-04-23
From Tara:
750 lbs = ___ tons

How do I figure this out!? I know that 1 ton is 2000 lbs, how do I figure out this answer?
Answered by Penny Nom.

Side length of an octagon 2000-04-22
From Unknown:
If I have an octagon that is 12 feet across (side to opposite side) how can I find the length of the sides?
Answered by Harley Weston.
Uniting algebra and geometry 2000-04-16
From Beth:
Who is the mathmatician that united algebra and geometry???
Answered by Claude Tardif.
Sample variance 2000-04-16
From Jonathan Freeman:
I was just reading your article entitled "A Note on Standard Deviation" I'm now teaching a unit on s.d. and my students were wondering why one uses a denominator of n for a population and n-1 for a sample. I saw in your article that this is because "[the quantity] tends to underestimate sigma... and other technical reasons." To which my students again asked... "Why?" Could you please elaborate a bit on the "other technical reasons" perhaps in terms a high school senior (or their teacher...) could understand?
Answered by Harley Weston.
Natural numbers 2000-04-12
From Sara:
What is a natural number???
Answered by Penny Nom.
The side length ratios of some triangles 2000-04-04
From Alexis Lockwood:
I am doing a project for my Math 30B class regarding the side length ratios of 45-45-90 degree and 30-60-90 degree triangles. I would really appreciate any assistance in answering the following questions, or even direction to an appropriate web site or resource on the matter.
Answered by Harley Weston.
Division 2000-03-26
From Kelly Allen:
My 8 year old daughter had 12 bracelets which she arranged into 4 piles of 3. Did she divide 12 by 3 or by 4?

Most people I've asked can only offer an opinion. What I'm looking for is a definitive answer, if there is one.

My husband says the number of sets is the answer: 12 divided by 3 = 4 Most others I've asked said that the amount in each set is the answer: 12 divided by 4 = 3


Answered by Harley Weston.
Divisibility by 3 2000-03-24
From Pat Walsh:
W hy does it work when you add the digits of a number then divid by three to see if the number is divisible by three
Answered by Penny Nom.
Binomial coefficients 2000-03-21
From Howard Lutz:
How do you find each successive numerical term in this equation y+dy=(x+dx)5 =x5+5*x4dx+10*x3(dx)2+10*x^2(dx)3+5*x(dx)4+(dx)5 I would appreciate an explanation of the method to find the numeric coefficient in a binomial expansion
Answered by Penny Nom.
Six digit numbers using 1,2,5,6,7, and 9 2000-03-20
From Rachel:
How many different six-digit numbers can you make using the digits 1,2,5,6,7, and 9? How many of these six digit numbers are divisible by six?
Answered by Claude Tardif and Denis Hanson.
The Terror Run 2000-03-18
From Danny Mclean:
A fairground's most popular attraction is a roller coaster ride known as the Terror Run. One stretch of track is called the Missile Path and is in the form of a parabolic curve. B is 180m horizontally from A and the highest point of the curve is 100m above A and B.

A.. The owner works out a quadratic expression to describe the Missile Path. What is the expression He found?

B.. A safety Engineer examined the structure and observed that points A and B were likely to be damaged due to the steepness of the Missile path near these points. The owner can see no way to make the Missile path less steep near A and B and to keep the height of the ride the same. HOW CAN IT BE DONE?


Answered by Harley Weston.
Division 2000-03-16
From Charday:
What is the easiest way to calculate the least number that can be divided evenly by all the numbers from 1 to 10.
Answered by Penny Nom.
Simultaneous equations 2000-03-11
From Laura Molck:
My name is Laura Molck and I am in Year 11 in Australia. Please help me with the following. I know that they are all simultaneous equations which I can do but I have trouble with the formulae to work the equations. Can you please help!!

1. A tent manufacturer produces 2 models, Outback and Bushwalker. From earlier sales records it is known that 20% more of the Outback model is sold than the Bushwalker. A profit of $200 is made on each Outback sold, but $350 is made on each Bushwalker. If during the next year a profit of $177,000 is planned how many of each model must be sold?


Answered by Penny Nom.
Systems of linear equations 2000-03-10
From Ann Marie Devereux:
hi there!!, I guess I have a problem!!!
  • 3x+4y=10 (over)
    4x+y=9

  • 2x=5y+3 (over)
    x=3y+1


Answered by Penny Nom.
Is a sheet of paper 2-dimensional? 2000-03-06
From Jaclyn Evans:
One day, we were discussing 3-dimensional items. Is a sheet of paper considered a 3-dimensional shape? Or is it 2-dimensional as described by 8.5 by 11 size? Also, could you give me an example of a 4-dimensional item or scenario?
Answered by Harley Weston.
Triple angle formula 2000-02-23
From Sara:
Can one derive a triple angle formula for sine and cosine? If so, how?
Answered by Chris Fisher.
Mass of the earth 2000-02-21
From Terry Broughton:
My name is terry i am 15 my question is how much does our planet earth weigh ?
Answered by Chris Fisher and Harley Weston.
A trig identity 2000-02-17
From Eric:

Question:
How do I solve this problem?

 sin3x   cos3x _____ - _____ = 2 sinx cosx 

Answered by Chris Fisher.
Euclidean Math puzzle 2000-01-24
From Margaret Matthews:

(Check out this web-site: Simeon's Triangle Puzzle )

I have tried to figure out how this could be, because everything I know about it tells me it can't be. However, I can't seem to make it NOT work.

Two right angle triangles. They are each cut up into four identical pieces. In the first, all the pieces fit together so that there are NO empty spaces; in the second, presumed to be identical in size to the first, the pieces are slightly rearranged, and now, there IS a space in the triangle.


Answered by Patrick Maidorn and Claude Tardif.
Three dimensional rectangle 2000-01-11
From Dennis Murphy:
I would like to find out the name of a Three dimensional rectangle.
Answered by Harley Weston.
Phone Number Possibilities 2000-01-09
From Hamilton Weston:
How do phone companies arrive at the possible number of phone numbers that can be generated for customers?
Answered by Harley Weston.
A probability experiment 2000-01-05
From Vanessa:
Duels in the town of Discretion are rarely fatal. There, each contestant comes at a random moment between 5 a.m. and 6 a.m. on the appointed day and leaves exactly 5 minutes later, honor served, unless his opponent arrives within the time interval and then they fight. What fraction of duels lead to violence?

There must be a minimum number of 100 trials and things like graphing calculator, dice, spinners, and whatever are allowed.
Answered by Harley Weston.

Factoring ^6 2000-01-03
From Athena:

my name is Athena and I have a question on factoring: how would you figure this out:

(x6-y6) and (x6+y6)


Answered by Penny Nom.
Graph of a sine function 1999-12-23
From Pierre:
Given;
amplitute:1
period: 540
Phase shift: 60 degree,right

I am ask to right the equation: sin 2/3 (value -60degree)

When I am asked to graph the equation, the period is mixing me up.
Answered by Harley Weston.

Passing trains 1999-12-10
From Nancy:
Two trains, each of 80 cars, must pass on a single track which has a deadend siding. How can they pass if the siding can only hold a locomotive and 40 cars?

I do not understand this...help. It will be due monday. I thought it would be simple if the siding could hold 80 cars.
Answered by Penny Nom.

Sines & cosine laws 1999-12-10
From Pierre Boivin:
Triangle LMN, angle L=71 degree , LM= 7.2 , MN=8.3 , ln= 5.9

The questiion was to find angle M. Using the cosine law I found the answer to be 44 degree. It is also the book answer. Using the sines law I found the answer to be 42.2 degree. why can't I use the sines law.
Answered by Chris Fisher.

Cubic meters to gallons 1999-12-09
From Chris Compton:
I am trying to convert cubic meters to gallons. Do you have the conversion formula? The problem I am trying to solve is:

59.62 cubic meters to gallons.


Answered by Harley Weston.

Systems of equations 1999-12-06
From Roger Hornbaker:
I am having problem figuring out x and y solutions.
  1. 5x + y = 4
    3x - y = 4

  2. 3x + 2y = 6
    - 3x + y = 0

Answered by Penny Nom.
How many 3/5 are in 3/4? 1999-12-06
From Whiteny:
How many 3/5 are in 3/4?
Answered by Penny Nom.
Whole numbers 1999-12-02
From Patty:
I'm in a 5th grade class, please explaine what whole numbers are.....
Answered by Penny Nom.
Cos x = -1/2 1999-12-01
From Pierre Boivin:
When I factor[ 2cos (square) - 5cos -3], I get (2cos + 1)(cos - 3). 2cos + 1 = 0, 2cos = -1, cos = -0.5,. Using inv cos on calculator, I get 120 degree related angle.

When I graph I get two values, between 90 and 180 degree and between 180 and 270 degrees. How do I find those two values. How do use 120 degree in relation with the x axis.
Answered by Penny Nom.

Estimating the population mean 1999-11-13
From John Barekman:
Statitistics: Estimating the population mean when the standard deviation is known:

I am not sure which n to use in the formula for the confidence interval equation:

x +/- z*(standard deviation/sqrt(n))

If we have data of ten people, and if we have the data of ten sets of ten people each, what is the difference in the n that we use? What is the difference between the standard deviation and the standard error? Are we using the number of sampling means or just the number of samples?
Answered by Harley Weston.

Body part measurements 1999-11-10
From V Bailey:
I am a kindergarten teacher and mother of a 6th grader who wants to do a science project on the correlations of body part measurements. We heard this on the radio and it sparked our interest, but now I cannot find any information about it. Examples are: your height is supposed to equal the distance from fingertip to fingertip when your arms are outstretched. Your foot size is supposed to equal the distance from your wrist to your elbow.

Please help me find more information on this topic.
Answered by Penny Nom.

Crossing number 1999-11-06
From Christian:
The crossing number of a graph G, denoted cr(G) is defined to be the minimum number of (pairwise) crossings of edges among all drawings of the graph in the plane. For example, cr(K5)=1 and cr(K3,3)=1.
What is cr(K7,7)?

I figured out that the answer is 81.

Now I am trying to figure out if K7,7 can be drawn in the plane with less than 81 crossings?

I'm not sure how to approach this one. Other than actually drawing it out and checking by trial and error, I am not sure how to approach this problem. Please help!
Answered by Denis Hanson.

Division/Fractions 1999-11-06
From Azmat Hussain:
Is there a way to explain division by fractions using patterns? For example 3 divided by one half.
Answered by Chris Fisher and Penny Nom.
Find the missing numbers 1999-10-12
From Bonnie:
I must find the missing numbers and function rules

noutput
710
1220
3940
2430
16b
87c

we are totally confused can someone help
Answered by Denis Hanson.

sin x = x/10 1999-10-07
From Amandeep Grover:
Solve the equation sin x = x/10
Answered by Harley Weston.
Two 12-sided polygons 1999-09-25
From Kelly Boulton:
Two 12-sided polygons are similar. A side of the larger polygon is 3 times as long as the corresponding side of the smaller polygon. wHAT IS the ratio of the area of the larger polygon to the area of the smaller polygon.
Answered by Walter Whiteley.
A fraction problem 1999-09-23
From TruRed:
Seven ninths divided by a negative 3.Could you pleas answer my question and go step by step.
Answered by Penny Nom.
Numbers with the digit 2 in 1...1000 1999-09-20
From Jessica:
Is there a trick to finding out how many numbers containing the digit two is there from 1 to 1000?
Answered by Walter Whiteley.
Stem and Leaf Plot 1999-09-14
From Jeanette Sovick:
My 5th grade son brought home a math paper, the title of which reads, Reading Stem-and-Leaf Plots...can you explain this so I can explain it to him...There is no book, his teacher just sent this practice sheet home for him to complete and I have no clue!
Answered by Penny Nom.
Who is right 1999-09-04
From rf, k, p and uncle a:
Good morning. Could you please, help us resolve and by the way, confirm "who is right"... regarding the solution of this problem: - a piece of metal is 6 in. wide by 2 in. in height and weights 500 pounds... - what is the weight of a piece made of same metal, 5 in. wide by 1 3/4 in in height? Could you please supply the "formula".

Regards, rf, k, p and uncle a.
Answered by Penny Nom.

Is zero positive? 1999-09-03
From Shah:
Is zero a positive integer?
Answered by Penny Nom.
Division by zero 1999-09-02
From Joshua B. Mariano:
Why can't I divide a number by zero. When I put in my calculater 0 divided by 7 it equals 0. But when I put 7 divided by zero my calculater say's error. Why can't this be solved? What is the answer? Is there a paper on it? Help?
Answered by Walter Whiteley.
The Bellhop Problem 1999-08-29
From Gerard O'Neill:
Three fellows going to a Math Conference book into a motel. "That will be $30.00" says the bellhop. Each fellow gives $10.00 and they go off to their room. The manager enters and says "That was a $25.00 room. Go and give them $5.00 back."
The bellhop heads off but thinks....."how am I going to split $5.00 between 3 fellows? Hey I will keep two dollars and give each guy a dollar." He does just that.......each fellow now paid $9.00 and the bellhop kept two. 3 X 9= 27 plus the bellhops two = $29.00 Where is the missing dollar?

Answered by Harley Weston.
A Trigonometry Question 1999-08-28
From Diane Simms:
My question is can the following be factored. I am a teacher who needs the factors to this right away. 2 Sin2X + 2 SinX CosX - 1= 0
Answered by Harley Weston.
Invert and multiply 1999-08-28
From Debbie Walter:
I have another question please, WHY do we invert and multiply when dividing fractions? I know that's what we do but WHY? What is the reasoning behind it?
Answered by Penny Nom.
Division by a negative 1999-08-19
From Sangeeta B.:
56/-8
Answered by Penny Nom.
111...1222...2 1999-08-11
From Brad Goorman:
Let N = 111...1222...2, where there are 1999 digits of 1 followed by 1999 digits of 2.
Express N as the product of four integers, each of them greater than 1.

Answered by Penny Nom.
From an airport control tower 1999-08-04
From Pammy:
Hi I am a 30 yo mature age student doing my HSC but am having difficulty understanding this, if you can help me.

From an airport control tower, a Cessna bears 023 degrees T and is 27km away. At the same time, a Boeing 767 bears 051 degrees T and is 61km from the tower. Both planes are at the same height.

i) What is the size of angle ATB?

ii) Using the cosine rule to calculate the distance the planes are apart, to nearest kilometre.

I figured out and drew the triangular diagram but can't figure out the rest and which formula to use. sorry about this, thankyou kindly
Answered by Harley Weston.

Sin 4A 1999-06-22
From Ryan Cochrane:
If sinA = 4/5, and A is a first quadrant angle, find sin4A
Answered by Harley Weston.
Measuring the fourth dimension 1999-06-07
From Suzanne Hall:
Hi! The other day, two of my students asked a very interesting question: Is the fourth dimension measured with hypercubes? Their reasoning went like this: Lines are 1D and are measured with line segments, which are part of a line. Planes are 2D and are measured with squares, which are part of a plane. Space is 3D and is measured with cubes, which are part of space. So, logically, hypercubes would be used to measure the fourth dimension.
Answered by Chris Fisher.
A rhombicosidecahedron 1999-04-30
From Himmat:
What is a rhombicosidecahedron?
Answered by Harley Weston.
Bill and Sam at the Casino 1999-04-23
From Rham Stewart:
Bill and Sam went off to bet at the casino. Each started with the same number of dollars. At the end of the first hour, Bill had won 20$ and sam had lost 20$. At the end of the second hour, BIll had lost two thirds of his money, and Sam had won the same amount that Bill had lost. At that point, sam had four times as much money as BIll. How much did each one start with?
Answered by Penny Nom.
A ladder problem 1999-04-22
From Michael Blade:
There is a cube box 3feet x 3feet x 3ft resting against a vertical wall on level ground. Resting against the outside corner of the box is a ladder 10 feet tall, this ladder is of course resting on the ground but also against the outside corner of the box and rests on the wall.

The question- the ladder is divided into two unequal section bounded by the box to the ground and the box to the wall. what are those dimensions?
Answered by Penny Nom.

Unit conversions 1999-04-20
From Deanna:
My question has to do with unit conversions with English and metric systems. I need to know how to use conversion factors with powers of 10. Which way do i move my decimal point. ex. .48kl to liters. Thank you.
Answered by Jack LeSage.
Dotted graph paper 1999-04-08
From Bridget Winward:
A teacher at our school is trying to locate dotted graph paper online or in print. His class would like to make three dimensional, geometerical drawings.
Please let us know if you have a good source.

Answered by Jack LeSage.
T-shirts 1999-04-04
From Valerie:
At a discount T-shirt shop the manager marked a special batch of T-shirts that originally sold for $2.00. When all the shirts sold the shop had collected $603.77.

How many shirts did they sell and what was the price per shirt?
Answered by Jack LeSage and Penny Nom.

Finding Excluded Values 1999-04-01
From Lorraine Le Cam:
The question asks me to find the value of the variable that must be excluded in each rational expressionion. The operation is:

a
______
a - 2

How do I work this out?
Answered by Harley Weston.

Cubic feet and gallons 1999-03-26
From Karen Coheley:
How many gallons in a cubic ft. ??
Answered by Harley Weston.
0/11 1999-03-09
From Jacob:
What is the term applied to 0 divided by a number.

In calculating slope problem my answer was 0 divided by 11. I don't know what to do with this answer or how to interpret it.
Answered by Jack LeSage and Penny Nom.

Reversing the direction of an inequality. 1999-03-06
From Mallory White:
If the Problem was -4a plus -5 is less than or equal to 14, why would you change the sign to greater than or equal to?
Answered by Jack LeSage and Harley Weston.
Quotients 1999-02-25
From Brian Healey:
what is a quotient?
what is a divisor?
what is a divident?


Answered by Jack LeSage.

Converting mm's to inches 1999-02-22
From Paul White:
It has been a long time since high school and I do not remember how to convert mm to inches. Could you please tell me what this would convert to in inches? 210 X 254 mm.
Answered by Chris Fisher.
Divisibility by 9 1999-02-21
From Razzi:
I've been having a hard time trying to solve the following problem and I was wondering if you could help me.

For any positive integer a let S(a) be the sum of its digits. Prove that a is divisible by 9 if and only if there exist a positive integer b such that S(a)=S(b)=S(a+b).
Answered by Chris Fisher and Harley Weston.

Finding a rule for a sequence 1999-02-17
From Lindsey Masters:
I'm doing a maths investigation and i have a sequence which goes:-

13,16,25,32,45,56,73.

Our teacher told us we have to find a rule by looking at the differences of the terms until we find a constant. The first differences are:-

3,9,7,13,11,17.

The differences of these are:- ......

Please could you tell me how to work it out so that I could work out the rules of similar sequences.
Answered by Penny Nom.

Dividing Decimals 1999-02-06
From Melanie Campbell:
i ugently need to know how to divide a decimal by a decimal eg:6.0 divided by 2.4 i need to show all working on a basic skills exam please help!!!!

mel
Answered by Jack LeSage.

Simplifying Radicals 1999-01-26
From Mary:
I would like to know how to simplify this question:
   
4 __________________ squareroot7 + squareroot3 
I know the answer is (sqrt7 - sqrt3) but i would really love to know how to get that answer!! Thanks.

Answered by Jack LeSage and Penny Nom.
Simplifying Radicals 1999-01-25
From Randy:
Find the perimeter of a rectangle in simplified radical form L=root80 W=root45. I would appreciate a detailed explination of the process for solving.
Answered by Jack LeSage and Penny Nom.
Fitting a Curve 1999-01-19
From Kirk:
Hello my name is Kirk from Scarborough, Ontario.

I have been out of a formal education system for thirty years. I program microcontrollers in my spare time. I have built a temperature sensing device ready to go but, thermistors are very non-linear. I do know that there is a way to calculate the input condition of the thermistor and display the correct temperature in degrees C. I am sending a file to show my progression so far.
Answered by Harley Weston.

Turning Fractions into Decimals 1999-01-16
From Lynn:
How do you do it step by step
Answered by Jack LeSage.
Indeterminate forms 1998-12-11
From R. Dixon:
What is the correct evaluation of infinity/0 ? I've checked three different math sites. One says definitively, that infinity/0 is "not" possible. Another states that infinity/0 is one of the indeterminate forms having a large range of different values. The last reasons that infinity/0 "is" equal to infinity.
Answered by Walter Whiteley and Harley Weston.
Intersection of planes 1998-11-22
From Dave Rasmussen:
I am a teacher of secondary mathematics with a question about the uses of Three Dimensional Co-ordinate Geometry. I have been teaching my students to write equations of planes and lines, - to find the intersection of these and the distance between them. What I am having difficulty finding are good applications of these techniques to "real world" situations. Can anybody help me?
Answered by Walter Whiteley and Harley Weston.
Fraction division 1998-11-18
From Brett Darrough:
I am needing specific examples that illustrate division of fractions with In-depth understanding. I am enrolled in a CGI (cognitivly guided instruction) course which is trying to develop a method of teaching fraction division with understanding. Most people invert and multiply to obtain a correct answer. Why? I would like help in understanding this method that includes pictures and algorithms that illustrate reasoning and depend on understanding.
Answered by Jack LeSage and Harley Weston.
Arithmetic Progressions 1998-11-12
From Gerry Boser:
It has been years since I was in school and I can't remember if there is a formula for the following problem:

If you deposit $1.00 on the first day of the month, $2.00 on the second day, $3.00 on the third day . . $31.00 on the last day of the month, how much do you have in the bank?

Now will this formula also work if it was, $0.25 (then day two you would deposit 2x $0.25 or $0.50, day three you would deposit 3x $0.25, $0.75. . . ). Will it work with any denomination??

Thank you for your time. I promise I'll write this one down for future reference. . .
Answered by Penny Nom.

Centimetres 1998-11-11
From Vicky Caldwell:
I work in Carpet Manufacturing and I need to know how to figure the calculation for a roll of carpet in centimeters. For example: a 12 x 100 roll of carpet equals 133.34 square yards, which is 100 linear ft. and 1,200 square foot. Can you please tell me how I would figure to get CENTIMETERS???????
Answered by Jack LeSage.
Divisibility by 11 1998-10-28
From Pat Duggleby:
I am an upgrading instructor at a drop-in program in Regina. One of my students is taking General Math 30 through correspondence, and we have run into some confusing instructions. The section is about divisibility rules, and we did just fine up until the rule for Divisibility by 11. The statement is as follows:
If the difference between the sum of the odd-numbered digits and the sum of the even-numbered digits, counted from right to left, is divisible by 11, then the number is divisible by 11.
.
.
.

Answered by Penny Nom.
A Six Digit Number 1998-10-27
From Craig Bedard:
If a certain six-digit number is split into two parts, one constituting the first three digits and the other the last three digits, and the two parts are added and the resultiong sum squared, it is found that the product is the original six digit number. What is the original six-digit number?

At first it seemed liked an impossible question, until it hit me...how long will it take you?
Answered by Jason Stein and Dan Usselman.

Operations Research 1998-10-08
From Lisa Barrett:
What is the history of operations research and the study of linear programming?
Answered by Judi McDonald.
Composite Numbers 1998-10-07
From Greg Murphy:
What type of composite number has an odd number of factors?
Answered by Harley Weston.
Dividing a Class 1998-10-01
From Tom Barker:
My eighth grade niece called with the following homework problem:

A teacher wanted to divide her class into equally numbered groups. She tried to divide the class into groups of two, but was one student short. She tried to divide the class into groups of five, but was one student short. She tried to divide the class into groups of seven and was successful. What is the least number of students that were in her class?

I know the answer is 49, but don't know how to prove it. I must be getting old if I can't solve eighth grade math problems. Your assistance would be appreciated.
Answered by Penny Nom.

A Hexagon 1998-09-22
From Lee Curtis:
Could someone calculate this for me?
If a hexagon is 38 feet 8 inches across, how wide would each of the six sides be?

Thank you,
Lee Curtis
Answered by Chris Fisher and Harley Weston.

Simplifying Roots 1998-09-18
From Lauren Lattimer:
1. 2root27 divided by root 12

2. 3 root 48-root 75
Answered by Penny Nom.

Cutting a Pizza 1998-09-09
From Woody:
what is the greatest number of pieces of pizza you can form if you use five straight cuts to cut the pizza? answer given is 16. please draw a diagram of the answer.
thanks, woody

Answered by Penny Nom.
Frieze Patterns 1998-08-19
From Brian Bairstow:
I am doing a research project on frieze patterns (also called band patterns or border patterns). I know that there are exactly seven different types of frieze patterns, but I have been unable to find a proof for this. If you could tell me this proof, or tell me some internet sites on which I can find material on this, I would be very grateful.
Answered by Chris Fisher.
A Place Value Curiosity 1998-05-25
From Ed:
I was visiting with an elderly gentleman this afternoon. He showed me this curiosity and then asked if I could explain it to him. Can you provide an explanation of why the 9 or multiple of 9 keeps occurring in this procedure? Choose any number, say 125 and add the digits to get 8. subtract the 8 from the 125 and the result is 117. Add the digits in 117 to get 9. Subtract the 9 from the 117 to get 108. Add the digits in 108 to get 9. If this procedure continues a 9 or a multiple of 9 reoccurs. What is the mathematical explanation behind this happening?
Answered by Denis Hanson.
The Fourth Dimension 1998-05-24
From Whitney Page:
Okay, here goes all my effort to try to explain shat I'm trying to ask of you. It's about something I read in a book called A WRINKLE IN TIME, by Madeline L'Engle. It's called tesser, or tesseract.

It talks about first diminsion, a straight line, second diminsion, a flat square, and third diminsion, a square with sides, front and back, top and bottom.

I can picture all of that. Then it says that fourth diminsion is when you square the three diminsional square. It also described the fourth diminsion as time. I can't figure out how that can be. Then it says...
Answered by Chris Fisher.

The Largest Factor 1998-03-15
From Senthuran Nadarajah:
When each expression is evaluated for different values of n, the answers will differ. For each expression, find largest natural number that will divide the result for all natural number values of n.

n^5 - 5n^3 + 4n
Answered by Harley Weston.

The Factor Theorem 1998-01-18
From Dhruv Bansal:
Hi, I'm a ninth grade student doing a math project for school. I'm trying to learn about various ways of solving 3rd and 4th degree polynomial equations using the remainder theorem, the factor theorem, and synthetic division. The books I have all mention x - r, which I know nothing about, not even how to get it.

I would really appreciate any information on this. Thanks.

Dhruv Bansal
Answered by Penny Nom.

Shimin's Geometry Problem 1997-12-02
From Ong Shimin:
ABCD is a rectangle. X and Y are the midpoints of BC and CD respectively. W is a point on AB such that AW : WB = 2 : 1. Z is a point on AD such that AZ : ZD = 2 : 1.

WY intersects XZ at O. If the area of triangle WOZ is 84 centimeters squared, find the area of triangle XOY.
Answered by Chris Fisher.

Division by a fraction. 1997-11-21
From Roger Marchegiano:
I am interested in a solution to the following extra credit problem for my son in Geometry. We have been unable to produce a satisfactory response:

Show a geometric representation of the division of fractions algorithm, (When you divide fractions you multiply the numerator by the reciprocal of the denominator).
Answered by Penny Nom.

Cos(x) Cos(2x) Cos(4x)=1/8 1997-09-24
From Tan Wang:
How many distinct acute angles x are there for which cosx cos2x cos4x=1/8?
Answered by Chris Fisher Harley Weston and Haragauri Gupta.
A Trigonometric Limit 1997-09-18
From Brian Ray:
What is the limit, as x approaches 0, or tan^23x/x^2? (read, tan squared 3x over...)?
Answered by Harley Weston.
Finding the Mine 1997-06-23
From Billy Law:
Tom is gold prospector. On his last trip out from town, he headed 35 degree South of West to a lake where he had lunch. The lake was 24 km out of Town. He then headed due East for 35 km before Doubling back on bearing of 15 degree South of west for 20 km to reach his mine. By converting to Cartesian coordinate before doing vector additions do the following:

a) Calculate the position of the mine from town in term of a distance and a direction.
...
Answered by Harley Weston.

A problem with arccos. 1997-06-09
From Vanessa Chan:
Prove: arc cos4/5 + arc cos (-5/13) = arc cos (-56/65)
Answered by Harley Weston.
The Division Bracket. 1997-04-09
From Judy Riley:
A fellow teacher recently asked if I remembered the exact word for a division bracket (not the symbol with dots, the horizontal line in a fraction, or a solidus). I couldn't. Can you help?
Answered by Walter Whiteley and Harley Weston.
A Presidential Proof 1997-03-18
From Greg Smith:
Which US president developed a proof for the Pythagorean Theorem?

Where can a copy of the proof be located?
Answered by Chris Fisher and Harley Weston.

Ajax, Beverley, Canton and Dilltown 1997-03-14
From S. Johnson:
The following towns are placed on a coordinate system. Ajax at (-x,z), Dilltown at (-10,0), Canton at (0,0) and Beverly at (0,10). The roads from Beverly to Canton and from Canton to Dilltown are perpendiculat to each other and are each 10 miles in length. A car traveling at all times at a constant rate, would take 30 minutes to travel straight from Ajax to Canton, 35 minutes to travel from Ajax to Canton via Beverly, and 40 minutes to travel from Ajax to Canton via Dilltown. What is the constant rate of the car, to the nearest tenth of a mile per hour.
Answered by Chris Fisher and Harley Weston.
Equivalent Expressions 1997-02-26
From Karyn Jones:
Josie says that s=n/(n+1) and s/(1-s)=n are two ways to write the same formula. Describe whether or not you agree with Joise. Explain how you made your decision.
Answered by Walter Whiteley and Penny Nom.
Division by zero 1997-02-14
From Linda Hood:
I am a college student and have been asked to explain and figure out why we can't divide by zero.
Answered by Chris Fisher.
A trig problem 1996-12-13
From S. Johnson:
sin t + cos t = 1/5. Find ALL exact values of cot t, given the original equation.
Answered by Harley Weston.
Height of a Hotel 1996-11-07
From Irene:
"Irene" is to determine the # of floors in a hotel 500 feet up the street. Irene is on the 10th floor of an office building and can measure the angle of elevation to the top of the hotel, 57 degrees. Her view of the entire building is obstructed. If the street rises at an angle of 8 degrees from the office building to the hotel and the average distance between floors is 11 feet, how many floors are on the hotel?
Answered by Penny Nom.
Priority of operations 1996-11-04
From Wallace:
What is the answer to this problem? Let x=-2 and y=3. Evaluate 12x / 2y (if the old division sign is used).
Answered by Chris Fisher.
Could you tell me the name for the bar in a division problem? 1996-10-21
From Linda:
Could you tell me the name for the bar in a division problem. Not the line with dots on either side but the line that divides the two numbers? My name is Linda. I am asking for my niece who is in 8th grade.
Answered by Chris Fisher.
Divisibility of 2n choose n. 1996-09-24
From Kathy Doan:
Can you prove that "2n choose n" is not divisible by 3, 5, and 7 for infinitely many n?
Answered by Penny Nom.
How do you find out if a number is a composite or prime? 1996-09-16
From Trish Feenstra:
Hi my name is Trish Feenstra. In grade 8. How do you find out if a number is a composite or prime if it is big like 37 529? Reply soon got a test on Friday the 13th
Answered by Denis Hanson.
A trig identity 1996-03-11
From Azmat Hussain:
Is there an easier/another way to prove the trig identity cos(a+b) = cos(a)cos(b)-sin(a)sin(b)?
Answered by Penny Nom.
Magic Square 1995-10-20
From Marianne and Carrie:
How can an 8 by 8 square have the same area as a 5 by 13 rectangle?
Answered by Denis Hanson.
deux wagons 2013-04-05
From Cyril:
Bonjour,
je suis confronté à un problème dont je n'arrive pas à trouver la solution. Le voici: "Deux wagons de masses 30 tonnes et 60 tonnes entrent en collision sur une voie ferrée droite horizontale, ne déraillent pas et restent collés ensemble après le choc. Juste après la collision, ils roulent ensemble à 10 km/h vers le nord. Sachant que le wagon léger roulait à 20km/h vers le nord avant la collision, la vitesse du wagon lourd avant la collision = ?"

Je pense qu'il faut employer la formule de la quantité de mouvement "p=mv" mais je ne sais pas comment m'y prendre...

Merci d'avance pour votre aide!
Cyril Biselx

Answered by Claude Tardif.
une écriture plus simple des expressions 2011-01-13
From jessie:

trouver une écriture plus simple des expressions suivantes sachant que : x + y = 3 et x - y =-2
A=x-1+y+2             B=x-1+y-2
C=x+1+y+2            C=x+1+y-2


Answered by Claude Tardif.
Division 2010-03-10
From lucas:
bonjour je suis eleve de 6 eme je bloque sur ce petit probleme

Nat a fait une division,elle ecrit 4568=23*198,608+reste que vaut ce reste?

merci d'avance
lucas

Answered by Claude Tardif.
Un système de plusieurs équations et plusieurs inconnues 2009-01-22
From Cédrick:
J'ai un problème écrit à résoudre est-ce que vous pouvez m'aider ?

Le premier est le tiers de la somme des deux autres.
La somme du premier et du deuxième est 13.
Le produit du deuxième et du troisième est 56.
Réponse: Le premier nombre est___________________
Le deuxième nombre est _________________
Le troisième nombre est _________________

Answered by Pierre-Louis Gagnon, Antoine Letarte at Claude Tardif.
Je voulais savoir d'où provient le signe # ?! 2008-01-17
From Dimitri:
Je voulais savoir d'où provient le signe # ?! On l'appelle diez, sur les téléphones surtout, mais j'aimerai savoir plus précisément à quoi sert ce signe (autre que sur nos appareils) ! Et pourquoi, en anglais, s'il est placé avant un chiffre, on dit 'number' (par exemple #1, 'Number one'). Son origine m'interesse tout autant que son utilité !
Answered by Claude Tardif.
Progression arithmétique 2008-01-08
From parrot:
Bonjour je suis en 6ème et j'ai besoin d'aide pour un problème de math. Donc voila mon problème : Un renard a mangé 100 grains de raisins pendant une periode de 5 jours. Chaque jour, il a mangé six grains le plus que le jour précédent. Quel est le nombre de grains mangés le premier jour ?
Answered by Claude Tardif.
Le symbole ∈ epsilon 2007-10-10
From Cora:
Je veux aider ma fille avec ses devoirs. C'est quoi le symbole qui semble un E, mais c'est un demi-circle? ie:X ∈ N
Answered by Claude Tardif.
les données sont inconsistantes 2007-10-03
From Mikee:
Bonjour, Je me casse la tete depuis ce matin et la je donne ma langue au chat. Je n'arrive pas a resoudre ce probleme. J'ai 30 agents qui appellent dans la journee et realisent 322 questionnaires (long+court) telephonique avec une duree moyenne de conversation 294 secondes. dans ces 322 qestionnaires : -ils realisent 313 questionnaires Long avec une duree moyenne de 317 secondes. -ils realisent 9 questionnaires Court avec une duree moyenne X. je cherche X.

((Interviews réalisées*Durée moyenne de conversation)-(Interviews réalisées avec la question Q2*Durée moyenne de conversation avec la question Q2))/(Interviews réalisées-Interviews réalisées avec question Q2)=Durée moyenne de conversation sans la question Q2

Q2=questionnaire Long

Answered by Claude Tardif.
Illusion géométrique 2007-04-04
From Dominique:
Je n'arrive pas expliquer la modofocation de surface.
Answered by Claude Tardif.
sup {sin n, n est un entier } 2006-12-17
From Madi:
sup {sin n, n est un entier }
Answered by Claude Tardif.
probleme sin cos 2006-02-19
From Thibault:
mon probleme commence par: f(x)=sinx (sinx+1)+ cos²x
donc en le dévellopant on trouve: f(x)= sin²x + sinx + cos²x
et apres ce que je ne comprend pas est que par la suite on trouve: f(x)= 1+sinx
qu'est-ce qui fait que l'on trouve ce resultat??

Answered by Claude Tardif.
des conversions de bases différentes 2003-01-04
From Christian:
Je cherche désespérément la (les) règle(s) pour réussir des conversions de bases différentes : de base x vers base y.
Answered by Claude Tardif.
100 centimètres 2002-11-13
From Un élève:
je voudrais savoir la contenance de 100 centimètres cube en litre?
Answered by Diane Hanson.
Conversion de m2 en cm2 2002-03-10
From Carole et Francis:
Mon enfant est en 6ième année, il avait le problème suivant à faire dans un test:

Trouve l'aire d'un rectangle mesurant 9 m par 4 m; donne ta réponse en m2 et en cm2?

Voici la solution de mon fils:

9m x 4m = 36m2

9m = 900cm 4m = 400cm

donc 900cm x 400cm = 360 000cm2

D'après moi sa réponse est bonne, j'aimerais avoir une confirmation puisque l'enseignante lui dit qu'il y a erreur.


Answered by Claude Tardif.
l'équation d'une rotation dans un graphique cartésien 2001-11-14
From Ghaith:
je d6sir que vous me rafraichissez un peu la mémoire s.v.p. j'aimerai savoir l'équation d'une rotation dans un graphique cartésien merci
Answered by Claude Tardif.
Les tableaux de conversions 2001-05-03
From Jean-Jacques:
Sur quel site est il possible de trouver tous les tableaux de conversions de mètres,kg,litre,m2,m3

Quelle est la relation entre les m3 et les litres 1 m3 = 1000 litres = 1000 kg?


Answered by Claude Tardif.
Calculer la hauteur de chute de cette pierre 2000-12-16
From Marcus:
Une pierre tombe au fond d'un puit, à 5 secondes on entend un son (de la pierre). Calculer la hauteur de chute de cette pierre en tenant compte de la vitesse du son = 340m.s-1

Il faut savoir que pour la chute libre v= g.t ; h = 1/2 g.t2 ; v = 2g.h

g intensité du champs de pesanteur = 10 ; h hauteur de chute.


Answered by Claude tardif.
la dimension fractale 2000-05-01
From François RIVOIRE, Julien CHOLLET et Alexandre DECOSTER:
nous sommes étudiants en première année MPSI en école d'ingénieur ISEN ( Institut supérieur d'électronique du Nord ). Nous avons choisi les FRACTALES ( ou plus précisément sur la dimension fractale et topologique ainsi que sur les suites logistiques) comme sujet de TIPE; serait-il possible que vous nous envoyez des démonstrations mathématiques ou des conseils car jusqu'ici nous avons eût beaucoup de mal à trouver des démonstrations se rapportant à ce sujet,
Answered by Claude Tardif.
 
Page
1/1

 

 


Math Central is supported by the University of Regina and The Pacific Institute for the Mathematical Sciences.

CMS
.

 

Home Resource Room Home Resource Room Quandaries and Queries Mathematics with a Human Face About Math Central Problem of the Month Math Beyond School Outreach Activities Teacher's Bulletin Board Canadian Mathematical Society University of Regina PIMS